PRELIMS MODEL TEST-11 ANSWER KEY AND EXPLANATION. PMT-11 to 15 Explanation.pdfTARGET PRELIMS 2017...

87
NEO IAS 0484-3190310, 9446331522, 9446334122 Page 1 www.neoias.com | www.youtube.com/neoias | www.facebook.com/neoias | www.twitter.com/neoias TARGET PRELIMS 2017 GENERAL STUDIES PAPER - 1 PRELIMS MODEL TEST-11 ANSWER KEY AND EXPLANATION 1. Answer (b) Explanation An ecological niche is the role and position a species has in its environment (ecosystem); how it meets its needs for food and shelter, how it survives, and how it reproduces. Ecological niche is the physical space or position and functional role of a species within the community or ecosystem. A species niche includes all of its interactions with the biotic and abiotic factors of its environment. A habitat will have several ecological niche. A niche is unique for a species while many species share the habitat. No two species in a habitat can have the same niche. This is because of the competition with one another until one is displaced. The description of a niche may include descriptions of the organism's life history, habitat, and place in the food chain. More formally, the niche includes how a population responds to the abundance of its resources and enemies. The abiotic or physical environment is also part of the niche because it influences how populations affect, and are affected by, resources and enemies. 2. Answer (b) Explanation About NoMore50 o The campaign includes a series of videos from celebrities, urging the ministry of environment and forests to increase punishment for animal cruelty in the Prevention of Cruelty to Animals Act, 1960. o Currently, the maximum penalty even for the most heinous form of animal abuse is a petty Rs 50. o NoMore50 is a campaign from the animal rights groups - the Humane Society International/India and the People for Animals. 3. Answer (b) Explanation o Animal Welfare Board of India is a statutory advisory body on Animal Welfare Laws under the Union Ministry of Environment, Forests and Climate Change (MoEF) o Established in 1962 under Section 4 of the Prevention of Cruelty to Animals Act, 1960 1 B 11 D 21 D 31 D 41 C 51 B 61 C 71 C 81 C 91 D 2 B 12 B 22 B 32 D 42 B 52 C 62 B 72 C 82 D 92 C 3 B 13 D 23 C 33 D 43 B 53 B 63 B 73 A 83 C 93 C 4 C 14 D 24 D 34 A 44 C 54 B 64 C 74 D 84 B 94 A 5 A 15 C 25 C 35 C 45 D 55 A 65 A 75 A 85 C 95 D 6 C 16 D 26 C 36 C 46 D 56 C 66 A 76 B 86 A 96 C 7 C 17 C 27 A 37 C 47 B 57 B 67 B 77 D 87 B 97 D 8 B 18 B 28 B 38 B 48 C 58 D 68 C 78 A 88 B 98 A 9 B 19 B 29 D 39 B 49 D 59 B 69 D 79 A 89 D 99 B 10 B 20 C 30 B 40 B 50 C 60 D 70 B 80 D 90 A 100 D Date: 17-04-2017

Transcript of PRELIMS MODEL TEST-11 ANSWER KEY AND EXPLANATION. PMT-11 to 15 Explanation.pdfTARGET PRELIMS 2017...

Page 1: PRELIMS MODEL TEST-11 ANSWER KEY AND EXPLANATION. PMT-11 to 15 Explanation.pdfTARGET PRELIMS 2017 GENERAL STUDIES PAPER - 1 PRELIMS MODEL TEST-11 ANSWER KEY AND EXPLANATION 1. Answer

NEO IAS 0484-3190310, 9446331522, 9446334122 Page 1 www.neoias.com | www.youtube.com/neoias | www.facebook.com/neoias | www.twitter.com/neoias

TARGET PRELIMS 2017 GENERAL STUDIES

PAPER - 1

PRELIMS MODEL TEST-11

ANSWER KEY AND EXPLANATION

1. Answer (b) Explanation An ecological niche is the role and position a species has in its environment (ecosystem); how it meets its needs for food and shelter, how it survives, and how it reproduces. Ecological niche is the physical space or position and functional role of a species within the community or ecosystem. A species niche includes all of its interactions with the biotic and abiotic factors of its environment. A habitat will have several ecological niche. A niche is unique for a species while many species share the habitat. No two species in a habitat can have the same niche. This is because of the competition with one another until one is displaced. The description of a niche may include descriptions of the organism's life history, habitat, and place in the food chain. More formally, the niche includes how a population responds to the abundance of its resources and enemies. The abiotic or physical environment is also part of the niche because it influences how populations affect, and are affected by, resources and enemies.

2. Answer (b) Explanation About NoMore50 o The campaign includes a series of videos from celebrities, urging the ministry of environment and forests to

increase punishment for animal cruelty in the Prevention of Cruelty to Animals Act, 1960. o Currently, the maximum penalty even for the most heinous form of animal abuse is a petty Rs 50. o NoMore50 is a campaign from the animal rights groups - the Humane Society International/India and the

People for Animals.

3. Answer (b) Explanation o Animal Welfare Board of India is a statutory advisory body on Animal Welfare Laws under the Union Ministry

of Environment, Forests and Climate Change (MoEF) o Established in 1962 under Section 4 of the Prevention of Cruelty to Animals Act, 1960

1 B 11 D 21 D 31 D 41 C 51 B 61 C 71 C 81 C 91 D

2 B 12 B 22 B 32 D 42 B 52 C 62 B 72 C 82 D 92 C

3 B 13 D 23 C 33 D 43 B 53 B 63 B 73 A 83 C 93 C

4 C 14 D 24 D 34 A 44 C 54 B 64 C 74 D 84 B 94 A

5 A 15 C 25 C 35 C 45 D 55 A 65 A 75 A 85 C 95 D

6 C 16 D 26 C 36 C 46 D 56 C 66 A 76 B 86 A 96 C

7 C 17 C 27 A 37 C 47 B 57 B 67 B 77 D 87 B 97 D

8 B 18 B 28 B 38 B 48 C 58 D 68 C 78 A 88 B 98 A

9 B 19 B 29 D 39 B 49 D 59 B 69 D 79 A 89 D 99 B

10 B 20 C 30 B 40 B 50 C 60 D 70 B 80 D 90 A 100 D

Date: 17-04-2017

Page 2: PRELIMS MODEL TEST-11 ANSWER KEY AND EXPLANATION. PMT-11 to 15 Explanation.pdfTARGET PRELIMS 2017 GENERAL STUDIES PAPER - 1 PRELIMS MODEL TEST-11 ANSWER KEY AND EXPLANATION 1. Answer

NEO IAS 0484-3190310, 9446331522, 9446334122 Page 2 www.neoias.com | www.youtube.com/neoias | www.facebook.com/neoias | www.twitter.com/neoias

o Established mainly for two purposes: for the promotion of animal welfare generally and for protecting animals from being subjected to unnecessary pain or suffering.

o The parliament of India enacted the PCA Act, 1960 to prevent cruelty to Animals and to prevent the infliction of unnecessary pain or suffering of animals.

o Country‘s apex institution for promoting welfare of animals. o AWBI is Government of India‘s largest funding agency for promotion of animal welfare initiatives. o AWBI provides financial assistance in the form of grants-in-aid to Animal Welfare Organisations.

4. Answer (c) Explanation Species diversity refers to the variety and number of different species in a given time and area. Ecologically, species variety is measured by species richness and relative abundance of species (species evenness). Species richness refers to the number of species in an area or community. Generally, species richness or biodiversity in general tends to increase with the size of the area or from higher to lower latitudes (equator) and higher to lower altitudes. Species richness is the primary estimation of species in an area.

5. Answer (a) Explanation In cities, everything is closely connected so problems tend to multiply. That‘s the bad news. The good news is that solutions can also multiply. And solutions exist already today that have the potential of meeting the demands of urban lifestyles, without exhausting the planet‘s ecological capacity. So WWF created the One Planet City Challenge to highlight these solutions, and to recognize and reward cities that are busy putting them to use. Cities that aim to provide sustainable housing, transportation, and energy for their residents while simultaneously acting as inspirational role models for other cities around the globe. The One Planet City Challenge, previously known as the Earth Hour City Challenge, invities cities in participating countries to report ambitious and innovative climate actions, and to demonstrate how they are delivering on the 2015 Paris Agreement.

6. Answer (c) Explanation About Sangai o Sangai or Brow- Antlered Deer is an endemic species to Manipur. Once distributed throughout much of

Southeast Asia extending from Manipur in North eastern India to Indochina, it is now confined in small patches in Manipur (India), Myanmar, Thailand, Cambodia, Lao Viet Nam and Southern China.

o State animal of Manipur o Its habitat is restricted to the marshy wetland of Keibul Lamjao National Park in the Southeastern fringe in

Loktak Lake (Ramsar Site), Manipur. o About two-third of the surface area of the lake is occupied by a floating Vegetation which is locally called

‗phumdi‘. o While walking on the floating biomass, Sangai often balances itself which looks as if it is dancing on the green

grassland therefore popularly called as ‗dancing deer‘ of Manipur. o The Sangai use phumdis, hillocks and elevated strips of land along the lake. o Found only at Keibul Lamjao National Park, it is the only species from the Northeast, which has been included

in the Centre-sponsored endangered species recovery programme. o It is listed as "endangered" in IUCN Red List and Schedule I of the Indian Wild Life (Protection) Act, 1972. o The Wildlife Institute of India has prepared a plan to secure long-term survival of sangai after extensive

research. The Compensatory Afforestation Fund Management and Planning Authority under the Union

ministry of environment, forests and climate change fund the programme.

7. Answer (c) Explanation 1. Kamlang Tiger Reserve : Arunachal Pradesh 2. Tadoba-Andhari Tiger Reserve : Maharashtra 3. Umred-Karhandla (Umred-Kharngla) wildlife sanctuary : Maharashtra

8. Answer (b) Explanation Fluttering with distinctive red and yellow stripes amongst the verdant green of the Western Ghats, the Southern Bird Wing butterfly was approved to become the ―State butterfly‖ of Karnataka. This makes Karantaka the second State in the country after Maharashtra to adopt a State butterfly. Maharashtra chose the Blue Mormom butterfly as its State butterfly a year ago. The Southern Bird Wing is the largest butterfly in India, with the females growing up to 190 mm in length. Primarily endemic to South India, it is found in abundance in Karnataka. By this move more people become aware of the importance of butterflies in the eco-system. These are important pollinators, and their preservation is necessary for the eco-system.

Page 3: PRELIMS MODEL TEST-11 ANSWER KEY AND EXPLANATION. PMT-11 to 15 Explanation.pdfTARGET PRELIMS 2017 GENERAL STUDIES PAPER - 1 PRELIMS MODEL TEST-11 ANSWER KEY AND EXPLANATION 1. Answer

NEO IAS 0484-3190310, 9446331522, 9446334122 Page 3 www.neoias.com | www.youtube.com/neoias | www.facebook.com/neoias | www.twitter.com/neoias

9. Answer (b) Explanation National Board for Wildlife (NBWL) is a statutory Board constituted on 22nd September 2003 under Section 5 of the Wild Life (Protection) Act, 1972. As per the amendment of the Wildlife (Protection) Act in 2002, a provision was incorporated for the constitution of the National Board for Wildlife, replacing the Indian Board for Wildlife. The NBWL is chaired by the Hon‘ble Prime Minister.

10. Answer (b) Explanation As per Section 62 of the Wildlife Protection Act, 1972, States can send a list of wild animals to the Centre requesting it to declare them vermin for selective slaughter. The Central Government may by notification, declare any wild animal other than those specified in Schedule I and part 11 of Schedule II of the law to be vermin for any area for a given period of time. As long as the notification is in force such wild animal shall be included in Schedule V of the law, depriving them of any protection under that law. Wild boars, nilgai and rhesus monkeys are Shedule II and III members- also protected, but can be hunted under specific conditions in certain regions of India as it is declared as vermin in those areas. Crows and fruit bat fall in Schedule 5, the vermin category. When a species be declared Vermin under the Wildlife Act so that they could be culled. The reason for declaring them as vermin is that, the species may be causing harm to the humans and damaging their agriculture. The main reason for this problem is their number is allegedly increasing or in another word the carrying capacity is increased, so there is less resources for the species, hence they are

migrating to human fields for their survival.

11. Answer (d) Explanation About Loktak lake o About two-third of the surface area of the lake is occupied by a floating Vegetation which is locally called

‗phumdi‘. o A mammal, sangai/dancing deers habitat is restricted to the marshy wetland of Keibul Lamjao National Park

in the Southeastern fringe of in Loktak Lake (Ramsar Site), Manipur. o The habitat in the Park is deteriorating primarily because of the change in water regime due to construction

of Itahi barrage. o The phumdi, which used to settle during lean season and get replenished with soil and nourishment, are now

continuously floating resulting in their thinning. Consequently, so they are unable to bear the weight of deer now.

o After the commissioning of the Loktak Hydro-Power Project in 1983, large agricultural areas at the lake periphery have been submerged which have changed the economic life of the people making them more dependent on the Park.

12. Answer (b) Explanation The world network included Biosphere reserves by UNESCO are Nilgiri, Agasthyamalai, Gulf of Mannar, Great Nicobar, Nanda Devi, Nokrek, Pachmarhi, Achanakmar- Amarkantak, Sunderbans and Similipal.

13. Answer (d) Explanation Biodiversity conservation done in the habitat itself (original place) is known as in-situ conservation. It is considered as the best method of protecting, propagating and developing species and their habitat along with ecological processes with which they have evolved. Eg: Biosphere Reserves, National Park, Tiger Reserve, Wildlife Sanctuary etc. Ex-situ conservation means the conservation of components of biological diversity outside their natural habitats or ex-situ conservation means the wild-life conservation in captivity under human care. Eg: 1. Gene banks; including seed banks, germ banks, field banks 2. Captive breeding

programme 3. Release programmes 4. Zoological garden 5. Botanical garden

14. Answer (d) Explanation The River Cauvery originates at Talakaveri in Kodagu (Coorg) District of Karnataka in Brahmagiri Range of hills in the Western Ghats. Its important tributaries are the Kabini, Bhavani, Noyil, Suvarnavati, Amaravati, Lakshmanatirtha, Harangi, Hemavati, Shimsha, Arkavati, and the Kapila. Coimbatore is located on Noyil. After crossing Tiruchirapalli district, the river Cauvery divides into two parts, the Northern branch being called "The Coleron" and Southern branch remains as Cauvery and from here the Cauvery Delta begins. After flowing for about 16 Kms, the two branches join again to form "Srirangam Island". On the Cauvery branch lies the "Grand Anicut"

said to have been constructed by a Chola King in 1st Century A.D.

Page 4: PRELIMS MODEL TEST-11 ANSWER KEY AND EXPLANATION. PMT-11 to 15 Explanation.pdfTARGET PRELIMS 2017 GENERAL STUDIES PAPER - 1 PRELIMS MODEL TEST-11 ANSWER KEY AND EXPLANATION 1. Answer

NEO IAS 0484-3190310, 9446331522, 9446334122 Page 4 www.neoias.com | www.youtube.com/neoias | www.facebook.com/neoias | www.twitter.com/neoias

15. Answer (c) Explanation If an animal is included in the Vermin category that means the animal can be culled as it cause‘s harm to crops and humans for an area for a given period of time. As long as the notification is in force such wild animal shall be included in Schedule V of the law, depriving them of any protection under that law. Recently the Ministry had issued notifications declaring three wildlife species as vermin in some states on their request. Ministry granted permission to Bihar in the last days of 2015 on December 1 to exterminate nilgai or blue bull and wild boar, declaring them vermin, to cull them for one year. The Bihar government reported harm to life and property due to overpopulation of nilgai and wild pig in areas outside forests. Similarly, along with Bihar, Uttarakhand and Himachal Pradesh also got permission for the scientific management for a limited time for a specific area. On February 2, ministry allowed culling of wild boar in Uttrakhand and killing of rhesus macaque monkey in Himachal Pradesh was allowed by the Centre on May 24.

16. Answer (d) Explanation Tropical evergreen forests are found in the western slope of the Western Ghats, hills of the north-eastern region and the Andaman and Nicobar Islands. Tropical evergreen forests are found in heavy rainfall areas (more than

200cm) with short dry seasons. It has luxuriant vegetation from trees to creepers.

17. Answer (c) Explanation o Researchers of the CV Raman Laboratory of Ecological Informatics, which is part of the Indian Institute of

Information Technology and Management – Kerala (IIITM-K), have developed BIOTA, a biodiversity app. o App is aimed at gathering information on geographical distribution of species for conservation and

educational purposes. o The app also aims at building up a database on different plant and animal species. o The beta version of the app, BIOTA 1.0, was released as part of a research paper in the recently-concluded

National Biodiversity Conference in Thiruvananthapuram

18. Answer (b) Explanation Earth Hour is a charitable organisation based out of Singapore. Their mission is uniting people to protect the planet. WWF is the parent organisation of Earth Hour and started Earth Hour with teams and partners in Sydney, Australia back in 2007. Earth Hour was famously started as a lights-off event in Sydney, Australia in 2007. Earth hour is a global environmental movement by WWF. Earth Hour is a people's movement inspiring individuals from more than 7000 cities across the globe to take a stand against climate change, making it the largest voluntary movement ever witnessed in history. Even though this is an open source campaign, the Earth Hour brand is legally trademarked worldwide and licensed for use by WWF delivering the Earth Hour movement. Rajkot has been declared the National Earth Hour Capital 2016 in WWF‘s global Earth Hour City Challenge (EHCC) for its strong

Page 5: PRELIMS MODEL TEST-11 ANSWER KEY AND EXPLANATION. PMT-11 to 15 Explanation.pdfTARGET PRELIMS 2017 GENERAL STUDIES PAPER - 1 PRELIMS MODEL TEST-11 ANSWER KEY AND EXPLANATION 1. Answer

NEO IAS 0484-3190310, 9446331522, 9446334122 Page 5 www.neoias.com | www.youtube.com/neoias | www.facebook.com/neoias | www.twitter.com/neoias

commitments across key areas, backed by strong low carbon actions and community engagement. The WWF annual EHCC is a year-long competition that highlights and rewards cities for their substantial long-term efforts to combat climate change. While Paris has emerged as the Global Earth Hour Capital for this year (2016), Rajkot was also in the run-up to bag the title alongside 17 National Earth Hour Capitals from across the world.

19. Answer (b) Explanation

Ratapani is a Wildlife Sanctuary in Madhya Pradesh.

20. Answer (c) Explanation Mosses, Ferns and Phytoplanktons are having chlorophyll, so they can produce carbohydrates by the process Photosynthesis; hence they are known as producers of ecosystems. But in the case of fungi it doesn‘t have chlorophyll, and they are consumers and decomposers in ecosystems.

21. Answer (d) Explanation Species diversity refers to the variety and number of different species in a given time and area. Ecologically, species variety is measured by species richness and relative abundance of species (species evenness). Species richness refers to the number of species in an area or community. Species evenness means relative abundance of each species in a community. Relative abundance means the proportion of each species. If a habitat has similar abundance for each species present, the habitat is said to have evenness. Evenness tells whether a particular ecosystem is numerically dominated by one species or whether all of its species have similar abundances. A community with an even species abundance is more diverse than one in which one or two species are abundant. Two communities can have the same species richness, but a different relative abundance.

22. Answer (b) Explanation The new National Air Quality Index (AQI) launched in 2014 can be described as a ‗One Number- One Colour-One Description‘ for the common man to judge the air quality within his vicinity. Air Quality Index (AQI) is a tool for effective dissemination of air quality information to people. The formulation of the index was a continuation of the initiatives under Swachh Bharat Mission. The current measurement index for air quality will consider eight pollutants or parameters (PM10, PM2.5, Nitrogen dioxide (NO2), Sulfur dioxide (SO2), Carbon monoxide (CO), ozone (O3), Ammonia (NH3), and Lead (Pb)) for which short-term (up to 24-hourly averaging period) National Ambient Air Quality Standards are prescribed. There are six AQI categories, namely Good, Satisfactory, Moderately polluted, Poor, Very Poor, and Severe. Central Pollution Control Board (CPCB) along with State Pollution Control Boards (SPCB) has been operating National Air Monitoring Program (NAMP) covering 240 cities of the country. In addition, continuous monitoring systems that provide data on near real-time basis are also installed in a few cities. Presently, air quality data for 10 cities are connected to AQI system which is available on the website of Ministry of Environment, Forest and Climate Change and Central Pollution Control Board.

23. Answer (c) Explanation Eco-sensitive zones are the transition zones around the Protected areas and is guidelined by the Ministry of Environment and Forests (MoEF) by the National Wildlife Action Plan (2002-2016). It is declared as Eco sensitive zone under the Environmental (Protection) Act, 1986. The land falling within 10km of the boundaries of Protected Areas are declared as Eco sensitive zone/ Eco fragile zone, to prevent ecological damage caused due to developmental activities around National Parks and Wildlife Sanctuaries by acting as a shock absorber or transition zone. The guidelines for the Eco sensitive zone were issued in a notification letter dated 19 February, 2011. The methods to manage such zones are by classifying the activities in to Prohibited, Regulated and Permitted.

24. Answer (d) Explanation 1. Dibang Wildlife Sanctuary : Arunachal Pradesh 2. Panna Tiger Reserve : Madhya Pradesh 3. Wayanad Wildlife Sanctuary : Kerala 4. Dandeli - Anshi Tiger Reserve : Karnataka

25. Answer (c) Explanation About Kashmir Stag o The Kashmir Stag or Hangul is a subspecies of elk native to India. o Known for its giant antlers bearing 11 to 16 points.

Page 6: PRELIMS MODEL TEST-11 ANSWER KEY AND EXPLANATION. PMT-11 to 15 Explanation.pdfTARGET PRELIMS 2017 GENERAL STUDIES PAPER - 1 PRELIMS MODEL TEST-11 ANSWER KEY AND EXPLANATION 1. Answer

NEO IAS 0484-3190310, 9446331522, 9446334122 Page 6 www.neoias.com | www.youtube.com/neoias | www.facebook.com/neoias | www.twitter.com/neoias

o The main concentration of the endangered species is in Srinagar‘s famed Dachigam national park. o Once found in the high altitudes of northern India and Pakistan, the animal now only lives in the dense

riverine forests of Dachigam.

Protection Measures o Listed under the Schedule-I of the Wildlife (Protection) Act, 1972 and J&K Wildlife (Protection) Act, 1978. o Listed among the top 15 species of high conservation priority by the Central Government.

26. Answer (c) Explanation Wetlands International (WI) o WI is a global not-for-profit organisation established in 1995, headquartered in the Netherlands o Wetlands International is dedicated to maintaining and restoring wetlands— for their environmental values

as well as for the services they provide to people o They are also supported by government and NGO membership. o They work at the field level to develop and mobilise knowledge, giving respect to traditional values and use

the practical experience to advocate for better policies

27. Answer (a) Explanation Coral bleaching Coral bleaching takes place when the symbiotic relationship between algae (zooxanthellae) and their host corals breaks down under certain environmental stresses. This results in the host expelling their zooxanthellae. In the absence of symbiotic algae, the corals expose their white underlying calcium carbonate coral skeleton and the affected coral colony becomes pale in colour. Coral bleaching can be activated and persist during varied environmental stresses When corals are stressed by changes in conditions such as temperature, light, or nutrients, they expel the symbiotic algae living in their tissues, causing them to turn completely white. Coral reefs are very sensitive ecosystems – even slight changes in water temperature and salinity can lead to coral ―bleaching‖ and even death.

28. Answer (b) Explanation The term community refers to the population of different kinds of organisms living together and sharing the same habitat. It consists of all the species living in an area. It consists of interacting populations. Population consists of all the organisms of one species living in an area. It is the group of interacting and interbreeding organisms.

29. Answer (d) Explanation Home range could be the whole area that can sustain the animal with living conditions such as food, shelter, and mating partners. When the concept of home range is considered, it covers all the individual members of a particular species. Therefore, as an example, it could be stated that the home range of the Asian elephant, Elephas maximus, is South and Southeast Asia including Sri Lanka, India, Thailand, and Burma. It is a concept that describes the real distribution of an animal. The home range of a species shows the actual geographical locations that the particular species naturally inhabits. The modern Geographic Positioning Systems are important in identifying the home ranges of mammals and other animals. The changes of home ranges of a species with time indicate the changes of resourcefulness of those areas. Therefore, home range concept is an indicator of the ecological sustainability of a particular area, country, or ecosystem. Territory is a geographical area or location that a particular population, a social unit, or an individual of a particular species (mostly mammals) occupy in a particular time. That means, the term territory does not imply to the whole species alone, but a territory could be occupied by either one animal or few related ones such as friends and family members. Territoriality is a mechanism to manage the available, limited resources among the animals in the same niche, and it is particularly common among carnivores. The primates and birds are the other territorial animals, and humans are amongst the serious territorial animals. All the territorial animals defend their defined territory against the conspecifics (individuals of the same species). Male lions protect the territory of their pride; primates defend the territory of a troop, and orang-utan keeps the others away from the territory of one individual. The territory is a self-defined area by different types of marking techniques such as urination, defecation, scratching trees, use of scent glands, and use of noise or other vocal effects. The dominant groups or individuals have larger territories compared to the submissive groups. Therefore, a territory provides its best resources to the strongest population or individual, so that the tendency to pass the better genes into the next generation is high.

30. Answer (b) Explanation World Heritage Sites

Page 7: PRELIMS MODEL TEST-11 ANSWER KEY AND EXPLANATION. PMT-11 to 15 Explanation.pdfTARGET PRELIMS 2017 GENERAL STUDIES PAPER - 1 PRELIMS MODEL TEST-11 ANSWER KEY AND EXPLANATION 1. Answer

NEO IAS 0484-3190310, 9446331522, 9446334122 Page 7 www.neoias.com | www.youtube.com/neoias | www.facebook.com/neoias | www.twitter.com/neoias

o A place of importance that is listed by the United Nations Educational, Scientific and Cultural Organization (UNESCO) as of special cultural or physical significance.

o There are natural and cultural sites. For Natural WHS, the site must be of outstanding universal value, the site should be of outstanding example ongoing ecological process, it should contain outstanding unique natural phenomena and beauty, contain exceptional biodiversity. For the Cultural WHS it should represent a masterpiece of human creative, monumental arts, outstanding architecture, traditional settlement their cultural tradition or civilisation, etc

o There are 7 Natural, 27 Cultural and 1 Mixed World Heritage Sites in India. o The 7 Natural sites are; Great Himalayan National Park Conservation Area, Western Ghats, Nanda Devi and

Valley of Flowers National Parks, Sundarbans National Park, Kaziranga National Park, Keoladeo National Park and Manas Wildlife Sanctuary

31. Answer (d) Explanation International Union for Conservation of Nature (IUCN) o IUCN is also known as the World Conservation Union o Former name - International Union for the Protection of Nature (IUPN) o Membership union composed of both government and civil society organisations (NGOs) for the

conservation of biodiversity o Established in 1948, headquarters in Gland, Switzerland o Has official Observer Status at the United Nations General Assembly o Prepare a list called IUCN Red List (Red Data Book), which classify the plants and animals on the basis of

their threat o It provides public, private and non-governmental organisations with the knowledge and tools that enable

human progress, economic development and nature conservation to take place together. o It is a leading provider of conservation data, assessments and analysis. o They run hundreds of field projects around the world to better manage natural environments.

32. Answer (d) Explanation Primary production is defined as the amount of biomass or organic matter produced per unit area over a time period by plants during photosynthesis. It is expressed in terms of weight or energy. Or Primary productivity means the rate of food produced by producers (autotrophs) or the rate of solar energy trapped at first trophic level. Primary productivity is again classified in to two, Gross primary productivity and Net primary productivity. Gross Primary Productivity (GPP) means the total amount of energy produced by producers (autotrophs) at trophic level one. Or in another words it is the rate at which an ecosystem‘s producers convert solar energy into chemical energy as biomass. This gross productivity depends up on photosynthetic capacity of producers and environmental factors including climatic conditions such as temperature, rainfall and total solar radiation and other

nutrient materials of the abiotic environment such as nitrogen, phosphorus and sulphur.

33. Answer (d) Explanation Tributary River Main River 1. Ken : Yamuna 2. Wainganga : Godavari 3. Shyok : Indus

34. Answer (a) Explanation World Heritage Sites o A place of importance that is listed by the United Nations Educational, Scientific and Cultural Organization

(UNESCO) as of special cultural or physical significance. o UNESCO seeks to encourage the identification, protection and preservation of cultural, and natural heritage

around the world considered to be of outstanding value to humanity. This is embodied in an international treaty called the World Heritage Convention concerning the Protection of the World Cultural and Natural Heritage, adopted by UNESCO in 1972.

o There are natural and cultural sites.

For Natural WHS, the site must be of outstanding universal value, the site should be of outstanding example ongoing ecological process, it should contain outstanding unique natural phenomena and beauty, contain exceptional biodiversity.

For the Cultural WHS it should represent a masterpiece of human creative, monumental arts, outstanding architecture, traditional settlement their cultural tradition or civilisation, etc

o There are 7 Natural, 27 Cultural and 1 Mixed World Heritage Sites in India.

35. Answer (c)

Page 8: PRELIMS MODEL TEST-11 ANSWER KEY AND EXPLANATION. PMT-11 to 15 Explanation.pdfTARGET PRELIMS 2017 GENERAL STUDIES PAPER - 1 PRELIMS MODEL TEST-11 ANSWER KEY AND EXPLANATION 1. Answer

NEO IAS 0484-3190310, 9446331522, 9446334122 Page 8 www.neoias.com | www.youtube.com/neoias | www.facebook.com/neoias | www.twitter.com/neoias

Explanation A biome is a large ecological area with a distinct climate and a distinct biological community of plants, animals and other organisms. Or a biome is a large ecological area having same kind of climate, same kind of vegetation and same kind of adaptations. One or more communities of different species interacting with one another and with their no-living environment is called as ecosystem. Or the structural and functional system of communities of living organisms and their environment is called ecological system or in short the ecosystem.

36. Answer (c) Explanation About AWC o The Asian Waterbird Census (AWC), conducted each year in January, is a waterbird and wetland-

monitoring programme initiated in 1987. o AWC is a part of the International Waterbird Census (IWC) which is completing its 50 years of waterbird

monitoring. o The Bombay Natural History Society (BNHS) along with Wetlands International is organizing Asian

Waterbird Census (AWC) to count waterbirds all over India.

Objectives of AWC o The AWC is a citizen science programme that supports the management and conservation of waterbirds

and wetlands. o The data collected through the AWC would also help identify and protect new sites of importance for

waterbirds. o Obtain information of waterbird population on an annual basis o Annual monitoring of the status and condition of wetlands o Obtain information of waterbird population on an annual basis and use it as a basis for monitoring population o The AWC encourages people to count waterbirds in wetlands around them and collect information that would

help promote the designation and management of internationally important sites such as nationally protected areas, Ramsar sites and Important Bird and Biodiversity Areas (IBAs).

37. Answer (c) Explanation Food chain is the sequence of who eats whom in a biological community (ecosystem) to obtain nutrition‖ or in another words the transfer of energy in an ecosystem through a series of organisms, by eating and being eaten Grazing food chain: This food chain starts from producers, passes through herbivores, then to carnivores and decomposers. Detritus food chain: This chain begins with dead organic matter which is eaten by animals which in turn are eaten by other animals. There are certain organisms which depend exclusively on the dead bodies of animals and plants. These organisms are called detritivores. Parasitic food chain: This food chain starts from herbivore but food energy passes from larger to smaller organisms without outright killing as in case of predator. Hence, the larger animals are considered to be the hosts and the smaller animals which fulfil their nutritional requirements from the hosts are considered as parasites.

38. Answer (b) Explanation o India Biodiversity Awards is a joint initiative of the Ministry of Environment, Forest and Climate Change

(MoEFCC), National Biodiversity Authority (NBA) and United Nations Development programme (UNDP). o It Recognise the contribution of stakeholders towards the conservation of biodiversity and excellence in

biodiversity governance. o The first India Biodiversity Awards were jointly announced by the Ministry and UNDP India in 2012 at the high

level segment of the Eleventh meeting of Conference of Parties (COPs) to the Convention on Biological Diversity, during India‘s Presidency of the COPs to the Convention.

o In 2016, the categories of the India Biodiversity Awards have been modified to link them with the provisions of the Biological Diversity Act, 2002, in order to incentivize stakeholders for the conservation of biodiversity and the effective implementation of the Act.

The four categories of awards: 1. Conservation of threatened species 2. Sustainable use of biological resources 3. Successful mechanisms/models for access and benefit sharing 4. Biodiversity Management Committee (BMC)

39. Answer (b) Explanation Energy is not cycling through nature. It is a unidirectional one. Every levels of the food chain the energy will be utilised by the biotic components, so the energy will be reduced in every levels.

Page 9: PRELIMS MODEL TEST-11 ANSWER KEY AND EXPLANATION. PMT-11 to 15 Explanation.pdfTARGET PRELIMS 2017 GENERAL STUDIES PAPER - 1 PRELIMS MODEL TEST-11 ANSWER KEY AND EXPLANATION 1. Answer

NEO IAS 0484-3190310, 9446331522, 9446334122 Page 9 www.neoias.com | www.youtube.com/neoias | www.facebook.com/neoias | www.twitter.com/neoias

40. Answer (b) Explanation The application of Nitrogen fertilizers to crops has caused increased rates of denitrification and leaching of nitrate into groundwater. The additional Nitrogen entering the groundwater system eventually flows into streams, rivers, lakes, and estuaries. In these systems, the added Nitrogen can lead to eutrophication. Increased deposition of Nitrogen from atmospheric sources is also because of fossil fuel combustion and forest burning. Both of these processes release variety forms of Nitrogen through combustion. Livestock release a large amount of ammonia into the environment from their wastes. These all Nitrogen enters the soil system and then the hydrologic system through leaching, groundwater flow, and runoff. Eutrophication is the natural aging of a lake by nutrient enrichment of its water. Pollutants from man‘s activities like effluents from the industries and homes can radically accelerate the aging process. This phenomenon has been called Cultural or Accelerated Eutrophication. During the past century, lakes in many parts of the earth have been severely eutrophied by sewage and agricultural and industrial wastes. The prime contaminants are nitrates and phosphates, which act as plant nutrients. They overstimulate the growth of algae, causing unsightly scum and unpleasant odors, and robbing the water of dissolved oxygen vital to other aquatic life. At the same time, other pollutants flowing into a lake may poison whole populations of fish, whose decomposing remains further deplete the water‘s dissolved oxygen content. In such fashion, a lake can literally choke to death. Excessive quantities of chemicals which get washed from the fields act as nutrients for algae in the nearby water body to flourish (algal bloom - excessive growth of planktonic (free-floating) algae). Algal blooms cause deterioration of the water quality and fish mortality. Once these algae die, they serve as food for decomposers like bacteria. A lot of oxygen in the water body gets used up. This results in a decrease in the oxygen level which may kill aquatic organisms, and becomes dead zones. Some bloom-forming algae are extremely toxic to human

beings and animals.

41. Answer (c) Explanation The gradual and progressive replacement of one community by another, till the development of a stable community in that area, or a process through which ecosystems tend to change over a period of time is called as ecological succession. Succession can be related to seasonal environmental changes, which create changes in the community of plants and animals living in the ecosystem. Hydrarch Succession takes place in wetter areas whereas Xerarch succession takes place in dry areas. Pioneer species in Hydrarch succession are usually the small phytoplankton and that in Xerarch succession are usually lichens.

42. Answer (b) Explanation About Himalayan Brown Bear o It‘s an omnivorous animal and India‘s largest animal (mammal) in the Himalayas. o They exhibit sexual dimorphism. o It is distributed in Nepal, Pakistan, and northern India. o In India they are found in Jammu and Kashmir, Himachal Pradesh and Uttarakhand. o The best place to spot them is the Great Himalayan National Park in Himachal. o A brown bear requires about 100 square kilometer as its territory to survive and any human intervention

disturbs its ecology. Threatened by o They are poached for their fur and claws for ornamental purposes and internal organs for use in medicines. o They are killed by shepherds to protect their livestock. o The tree bearing the state flower of Himachal — buransh — is the favourite hangout of this bear. Due to the

high value of the buransh tree, it is being commercially cut causing further destruction to the brown bear‘s home.

o In Himachal, their home is the Kugti and Tundah wildlife sanctuaries and the tribal Chamba region. o The 1999 Kargil war had caused significant destruction to their habitats, and no Himalayan bear had been

spotted in the region for many years. The Tiger Hill, one of the battle fields, was a prominent brown bear habitat. The war and the movement of troops did impact the animal‘s behavior and ecology.

43. Answer (b) Explanation Primary productivity is the rate of production of biomass by the producers in an ecosystem. During each energy transformations some of the energy is lost because of the respiration, so the annual productivity of the ecosystem will be greater than the annual increase in biomass of the herbivores in the ecosystem.

44. Answer (c) Explanation The speciation occurring when the population becomes separated by geographical barriers is called allopatric speciation. When a population is geographically continuous, the allele frequencies among its members are similar; however, when a population becomes separated, the allele frequencies between the two groups can begin to vary. Thus the chance of interbreeding between these populations is reduced. If the separation between groups

Page 10: PRELIMS MODEL TEST-11 ANSWER KEY AND EXPLANATION. PMT-11 to 15 Explanation.pdfTARGET PRELIMS 2017 GENERAL STUDIES PAPER - 1 PRELIMS MODEL TEST-11 ANSWER KEY AND EXPLANATION 1. Answer

NEO IAS 0484-3190310, 9446331522, 9446334122 Page 10 www.neoias.com | www.youtube.com/neoias | www.facebook.com/neoias | www.twitter.com/neoias

continues for a long period of time, the differences between their alleles can become more and more pronounced due to differences in climate, predation, food sources, and other factors, eventually leading to the formation of a new species. Each separated population acquires mutations by natural selection to adapt to the new environment. A long time, reproductive isolation sets in separating two populations into two species. Allopatric speciation events can occur either by dispersal, when a few members of a species move to a new geographical area, or by vicariance, when a natural situation, such as the formation of a river or valley, physically divide organisms. Allopatric is the most common form of speciation. Sympatric Speciation: It is the formation of two or more species from a single ancestral species all occupying the same geographical area. In this the populations are not geographically separated. Sympatric speciation often occurs through polyploidy. A diploid individual cannot interbreed with tetraploid individual leading to reproductive isolation. This type of speciation is rare and occurs often in plants as self fertilization and polyploidy is common in plants compared to animals.

45. Answer (d) Explanation Irrawaddy dolphin, Humpback dolphin, Bottlenose dolphin, Pantropical spotted dolphin, can be seen in India. The finless porpoise is a small porpoise and lacks a dorsal fin. It is replaced by a ridge with small bumps which runs down the middle of the back. The Indo-Pacific finless porpoise can also be distinguished by its rounded head, which lacks an apparent beak. This small marine mammal has a relatively slender body. Dolphins have longer noses, bigger mouths, more curved dorsal fins, and longer, leaner bodies than porpoises. Dolphins and porpoises have many similarities, one of which is their extreme intelligence. Both have large, complex brains and a structure in their foreheads, called the melon, with which they generate sonar (sound waves) to navigate their underwater world. People use the terms dolphins, porpoises, and whales to describe marine mammals belonging to the order Cetacea (from the Greek work ketos, ―large sea creature‖), and often use them interchangeably. The orca, or killer whale, for example, is actually the largest member of the dolphin family. Dolphins are by far more prevalent than porpoises. Most scientists agree that there are 32 dolphin species (plus five closely related species of river dolphin) and only six porpoise species.

46. Answer (d) Explanation Prosopis cineraria, locally called as Khejri or Jandi is an indigenous tree, which effectively stabilizes sand dunes and can withstand periodic burial. Culturally, the tree holds a very important place in lives Rajasthani people, especially the Bishnois. There is a significant economic and environmental importance too. The Khejri tree plays a vital role in maintaining the ecosystem of the dreary Thar region because of its ability to survive in such tough conditions and the different ways in which it can be used by the farmers. Apart from being a source of firewood and fodder, the Khejri also helps in sustaining the nutrient value of the soil and ensuring a good yield. The khejri tree is adapted to withstand both frost and drought, and survives both high temperature and low. It is known to improve soil fertility; it aids nitrate retention in soil. The decreasing number of Khejri has impacted the culture and economics in some way. It's in April every year that one expects to see the khejri tree, the lifeline of the desert, bearing fruit. This year (2016), however, the trees, a hardy species of the pea family, were afflicted by pests which have left the fruit, locally called sangria, unfit not just for human consumption. Cattle too won't touch it. Dried sangria could earlier be purchased for about Rs200 a kg. These days, it fetches about Rs1,000 a kg - production, it is estimated, has fallen by about 90%.The loss in this year is mainly because of the mites and the resultant gall formation. In 2015, the Central Arid Zone Research Institute (CAZRI) took out a report which stated that the number of Khejri trees per hectare in the 12 dry districts of Rajasthan had dropped to less than 35 per cent. This, the report stated, was happening because of a very high mortality rate owing to factors like decrease in groundwater level, fungal attacks, indiscriminate cutting, etc. The frequent and almost complete lopping off of branches for fodder affects seed production and stresses the trees. Frequent drought, the increased depth of the water table and use

Page 11: PRELIMS MODEL TEST-11 ANSWER KEY AND EXPLANATION. PMT-11 to 15 Explanation.pdfTARGET PRELIMS 2017 GENERAL STUDIES PAPER - 1 PRELIMS MODEL TEST-11 ANSWER KEY AND EXPLANATION 1. Answer

NEO IAS 0484-3190310, 9446331522, 9446334122 Page 11 www.neoias.com | www.youtube.com/neoias | www.facebook.com/neoias | www.twitter.com/neoias

of mechanical ploughs - which injure the roots - have also affected chances of regeneration. One cause of the higher mortality of the khejri tree, scientists speculate, could also be the changing topography, with the Indira Gandhi Canal making water flow through once arid desert.

47. Answer (b) Explanation A biodiversity hotspot is a biogeographic region that is both a significant reservoir of biodiversity and is threatened with destruction. Scientists exploring the forests of the Western Ghats have come across four new species of tiny frogs no bigger than a human thumbnail, which make a distinctive chirping sound comparable to the one of a cricket. These species are among the seven new ‗Night Frogs‘ discovered by a team of researchers from the University of Delhi and the Kerala Forest Department, who spent five years surveying the global biodiversity hotspot. ‗Night Frogs‘ belong to the Nyctibatrachus genus endemic to the Western Ghats and represent an ancient group of frogs that diversified on the Indian landmass approximately 70 to 80 million years ago.

48. Answer (c) Explanation 1. Manas River : Assam 2. Peddavagu river : Telangana 3. Pala Rapu cliff : Telangana

49. Answer (d) Explanation Biodiesel in India is mostly produced from the oils extracted from the seeds of Jatropha, mainly because of the fact that edible oil is scarce and the country already depends on huge quantity of imported oils for edible purposes. Apart from Jatropha, Pongamiapinnata, Mahua, Neem and Castor are also considered as good source of non-edible oil-based biodiesel in India. In Western countries, biodiesel is typically made from vegetable oil (rapeseed oil, sunflower oil and palm oil), animal tallow and used cooking oil. Rapeseed oil has 82 percent of the share of the world‘s biodiesel feedstock followed by sunflower oil, soybean and palm oil.

50. Answer (c) Explanation Neurocalyx calycinus is a medicinal plant endemic to the southern parts of Western Ghats and Sri Lanka. It could offer scientists the key to new herbal formulations and modern drugs for the treatment of cancer and wounds and burns. Scientists at the Jawaharlal Nehru Tropical Botanic Garden and Research Institute (JNTBGRI) here have confirmed the multiple therapeutic properties of Neurocalyx calycinus used by the Cholanaickan tribe, one of the particularly vulnerable groups in Kerala, to treat inflammations and wounds. The researchers have filed for a patent on a novel herbal drug formulation possessing wound-healing, burn-healing, anti-cancer, analgesic, anti-inflammatory, immuno- enhancing, platelet-augmentation and anti-oxidant effects. The presence of high Vitamin E content and potent cytoprotective activity in cell lines in the plant species have also enhanced the prospects of developing an anti-cancer drug.

51. Answer (b) Explanation Aquila is the code name of the unmanned solar-powered drone developed by Facebook. The company plans to use a linked network of the drones to provide internet access to large rural areas. Recently Facebook completed the first successful flight of Aquila. However, as with its Internet.org project, Facebook will not be dealing with customers directly, instead partnering with local ISPs to offer the services.

52. Answer (c) Explanation BharatNet aims to establish, by 2017, a highly scalable network infrastructure accessible on a non-discriminatory basis, to provide on demand, affordable broadband connectivity of 2 Mbps to 20 Mbps for all households and on demand capacity to all institutions, to realise the vision of Digital India, in partnership with States and the private sector. The entire project is being funded by Universal service Obligation Fund (USOF), which was set up for improving telecom services in rural and remote areas of the country. The objective is to facilitate the delivery of e-governance, e-health, e-education, e-banking, Internet and other services to the rural India.

53. Answer (b) Explanation A suspected chemical attack in a town in Syria‘s northern Idlib province killed dozens of people on April 4, 2017. The Britain-based Syrian Observatory for Human Rights monitoring group put the death toll at 58, saying there were 11 children among the dead. The province of Idlib is almost entirely controlled by the Syrian opposition. It is home to some 900,000 displaced Syrians, according to the United Nations. The United States launched a targeted missile

Page 12: PRELIMS MODEL TEST-11 ANSWER KEY AND EXPLANATION. PMT-11 to 15 Explanation.pdfTARGET PRELIMS 2017 GENERAL STUDIES PAPER - 1 PRELIMS MODEL TEST-11 ANSWER KEY AND EXPLANATION 1. Answer

NEO IAS 0484-3190310, 9446331522, 9446334122 Page 12 www.neoias.com | www.youtube.com/neoias | www.facebook.com/neoias | www.twitter.com/neoias

strike on Syria in response to the chemical attack. It was the first direct American assault on the Syrian government and Donald Trump‘s most dramatic military order since becoming President.

54. Answer (b) Explanation Islamic Military Alliance to Fight Terrorism (IMAFT) is a military alliance led by Saudi Arabia to fight terrorism, with a joint operations centre in Riyadh. The 39 member military alliance include most Arab League states, a number of mainly Muslim states mostly in Africa, and Asian countries including Pakistan, Bangladesh and Malaysia. Notable omissions are Iran, Iraq and Syria.

55. Answer (a) Explanation International Energy Agency (IEA) was founded in 1974 to help countries collectively respond to oil supply disruptions. In March 2017, India joined the IEA as an Association country. However India is still not a member of IEA. To be a member country of the IEA, a country must also be a member country of the OECD (Organisation for Economic Co-operation and Development).

56. Answer (c) Explanation The South Asia Subregional Economic Cooperation (SASEC) Program brings together Bangladesh, Bhutan, India, Maldives, Myanmar, Nepal, and Sri Lanka in a project-based partnership that aims to promote regional prosperity, improve economic opportunities, and build a better quality of life for the people of the subregion. SASEC countries share a common vision of boosting intraregional trade and cooperation in South Asia, while also developing connectivity and trade with Southeast Asia through Myanmar, to the People‘s Republic of China, and the global market.

57. Answer (b) Explanation China disputes the international boundary between India and China. Tawang is a disputed territory in the Eastern sector of the India- China boundary. Tawang district is situated in Arunachal Pradesh in Northeastern India. Remarks by a former senior Chinese official on a possible trade-off between Tawang in Arunchal Pradesh, which is claimed by China, and Aksai Chin in Jammu and Kashmir, claimed by India has made the region the focus of Sino-Indian Relations.

58. Answer (d) Explanation Yazidis are a Kurdish religious minority found primarily in Iraq, Turkey, Syria, the Caucasus region, and parts of Iran. The Pashtuns are an ethnic group native to Afghanistan and North-Western Pakistan. The Baloch are a people who live mainly in the Balochistan region of Pakistan, Iran, and Afghanistan, as well as in the Arabian Peninsula.

59. Answer (b) Explanation Organoids is a three-dimensional lung created by researchers in laboratory to study diseases, including idiopathic pulmonary fibrosis (Idiopathic pulmonary fibrosis is a chronic lung disease characterised by scarring of the lungs. The scarring makes the lungs thick and stiff, which over time results in progressively worsening shortness of breath and lack of oxygen to the brain and vital organs). It was created by coating tiny gel beads with lung-derived stem cells and allowing them to self-assemble into the shape of air sacs found in human lungs.

60. Answer (d) Explanation Shenzhou 11 was a manned spaceflight of the Shenzhou program of China, launched on 17 October 2016 (16 October UTC) from the Jiuquan Satellite Launch Centre.

It was China's sixth manned space mission.

Recently, it

docked with the Tiangong-2 space laboratory, which had been launched on September 15, 2016.

61. Answer (c) Explanation PAVA shell is a chilli-based ammunition. It is otherwise called Nonivamide. It aims to find an alternative to pellet guns for crowd control. PAVA can be categorised in the less-lethal munition category and once fired, the shells burst to temporarily stun, immobilise and paralyse the target in more effective ways than a tear gas shell or pepper sprays. On the Scoville scale (the degree to measure the power of chilli), PAVA is categorised as ―above peak‖ meaning it will severely irritate and paralyse humans, but in a temporary fashion. PAVA is biosafe which is better than chilli grenade or tear smoke shell and can also be used in combination with stun and tear shells by security forces facing unruly protestors in place of pellet guns.

Page 13: PRELIMS MODEL TEST-11 ANSWER KEY AND EXPLANATION. PMT-11 to 15 Explanation.pdfTARGET PRELIMS 2017 GENERAL STUDIES PAPER - 1 PRELIMS MODEL TEST-11 ANSWER KEY AND EXPLANATION 1. Answer

NEO IAS 0484-3190310, 9446331522, 9446334122 Page 13 www.neoias.com | www.youtube.com/neoias | www.facebook.com/neoias | www.twitter.com/neoias

62. Answer (b) Explanation Sarathi vessel is an Offshore patrolling (OPV) vessel. It was built by Goa Shipyard Ltd (GSL). It will help meet the increasing requirements of the Indian Coast Guard for policing and patrolling the vast Indian Exclusive Economic Zone. The 105-metre Sarathi is designed to carry one twin-engine light helicopter and five high speed boats. The ship is also capable of carrying pollution response equipment to contain oil spill at sea.

63. Answer (b) Explanation Intellectual disability (ID) is otherwise called mental retardation. It is measured by an intelligence quotient below 70. ID significantly limits an individual‘s intellectual ability and practical skills. Half of all ID cases can be linked to environmental causes like poor nutrition. 50% of ID cases can be linked to unknown genetic factors like Gene mutations.

64. Answer (c) Explanation Yudh Abhyas 2016 is a Joint military training exercise between India and U.S.A. The exercise is aimed at stimulating a scenario where both nations were working together to counter insurgency and terrorism in mountainous terrain under UN charter. It is a series of one of the longest running joint military training and a major ongoing bilateral defence cooperation endeavours between India and the US. It is the 12th edition of the joint military exercise hosted alternately by the two countries.

65. Answer (a) Explanation Barak-8 Missile is a Surface to Air Missile. It was jointly developed by Defence Research and Development Organization (DRDO) and Israel Aerospace Industries (IAI) and Israel's Administration for the Development of Weapons and Technological Infrastructure. It was designed to defend against a variety of short-to-long-range airborne threats, including fixed-wing aircraft, helicopters, drones and projectiles, Barak-8 incorporates a state-of-the-art phased array multi-mission radar, two-way data link, and a flexible command and control system, enabling users to simultaneously engage multiple targets day and night and in all weather conditions.

66. Answer (a) Explanation Three parent technique involves fertilising both the mother‘s egg and a donor egg with the father‘s sperm. Before the fertilised eggs start dividing into early-stage embryos, each nucleus is removed. The nucleus from the donor‘s fertilised egg is discarded and replaced by that from the mother‘s fertilised egg. The resulting egg – with nuclear DNA from the mother and mitochondrial DNA from a donor – was then fertilised with the father‘s sperm. It is legally approved in the UK.

67. Answer (b) Explanation Avian influenza is a disease occurring amongst birds that particularly affects domestic poultry such as chickens, turkeys and ducks. It is caused by H5N1 virus. It is a zoonotic disease. Human infections are primarily acquired through direct contact with infected animals or contaminated environments, but do not result in efficient transmission of these viruses between people. There is no evidence that the avian or zoonotic influenza viruses can infect humans through properly cooked food. Recently, the Department of Animal Husbandry, Dairying and Fisheries in the Ministry of Agriculture and Farmers welfare has declared India free from Avian Influenza (H5N1) from 5th September, 2016.

68. Answer (c) Explanation Quantum radar is a device that uses quantum entanglement photons to provide better detection capabilities than conventional radar systems. It was developed by the Intelligent Perception Technology Laboratory of the 14th Institute in CETC, according to Xinhua news agency. CETC stands for Electronics Technology Group Corporation, a defense and electronics firm. The system was able to detect a target at a range of 100 kilometres in a real-world environment. The device employs single photon detection technology. The method would be useful for tracking targets with a low radar cross section, such as modern aircraft using stealth technology or targets employing active countermeasures to jam or baffle enemy.

69. Answer (d) Explanation Pradhan Mantri Awas Yojna (PMAY) is a mission started with an aim ‗Housing for All‘ (HFA) scheme by the present Government to be achieved by the year 2022, that is when India will be completing its 75 years of Independence. The mission started in June 2015 and will be attained in seven years i.e., during 2015 – 2022. The scheme originally was meant to cover people in the economically weaker sections (EWS - annual income not exceeding ₹3 lakh) and low-income group (LIG - annual income not exceeding ₹6 lakh) sections, but now covers the mid-income

Page 14: PRELIMS MODEL TEST-11 ANSWER KEY AND EXPLANATION. PMT-11 to 15 Explanation.pdfTARGET PRELIMS 2017 GENERAL STUDIES PAPER - 1 PRELIMS MODEL TEST-11 ANSWER KEY AND EXPLANATION 1. Answer

NEO IAS 0484-3190310, 9446331522, 9446334122 Page 14 www.neoias.com | www.youtube.com/neoias | www.facebook.com/neoias | www.twitter.com/neoias

group (MIG) as well. However there are further specifications of people who are targeted as direct beneficiaries: (i) Indian women of all religions and castes: There will be no biased approach depending on the said factors and everyone will be equally eligible, (ii) Scheduled tribes and scheduled castes and (iii) Physically differently abled or senior citizens.

70. Answer (b) Explanation Deen Dayal Upadhyaya Grameen Kaushalya Yojana (DDU-GKY) was launched on 25th September, 2014. The new training programme envisages setting up of at least 1500 to 2000 training centres across the country and will be run on PPP model. DDU-GKY aims to skill rural youth who are poor and provide them with jobs having regular monthly wages or above the minimum wages. It seeks to promote rural livelihoods as part of the Deen Dayal Antyodaya Yojana (DAY-NRLM). Rural youth, under the age group of 15–35 years are eligible to get the benefits of the scheme. It also guarantees placement for at least 75% trained candidates.

71. Answer (c) Explanation Pradhan Mantri Suraksha Bima Yojana is a government-backed accident insurance scheme in India. It provides for accidental Death Insurance. The PMSBY is aimed at covering the uncovered population at a highly affordable premium of just Rs 12 per year. The scheme will be available to people in the age group 18 to 70 years of age with bank accounts.

72. Answer (c) Explanation Pradhan Mantri YUVA Yojana (Yuva Udyamita Vikas Abhiyan) is a centrally sponsored Scheme on entrepreneurship education and training being implemented by the Ministry of Skill Development and Entrepreneurship, Government of India. The Scheme aims at creating an enabling ecosystem for Entrepreneurship development through Entrepreneurship education and training; Advocacy and easy access to entrepreneurship support network and Promoting social enterprises for inclusive growth. The scheme spans over five years (2016-17 to 2020-21). It will also include easy access to information and mentor network, credit, incubator and accelerator and advocacy to create a pathway for the youth.

73. Answer (a) Explanation Swavlamban Health Insurance Scheme was launched on 21st September, 2015. It pertains to provide affordable Health Insurance to the persons with disabilities (PwDs). It aims to improve the general Health condition & quality of life of persons with disabilities. It ensures coverage of any pre-existing condition and a health Insurance cover up to Rs. 2, 00,000 per annum as family floater. Persons with blindness, low vision, leprosy-cured, hearing impairment, loco-motor Disability, mental Retardation and mental Illness are eligible to get the benefits under the scheme.

74. Answer (d) Explanation The National Rural Health Mission (NRHM) was launched on 12th April 2005, to provide accessible, affordable and quality health care to the rural population, especially the vulnerable groups. Under the NRHM, the Empowered Action Group (EAG) States as well as North Eastern States, Jammu and Kashmir and Himachal Pradesh have been given special focus. The thrust of the mission is on establishing a fully functional, community owned, decentralized health delivery system with inter-sectoral convergence at all levels, to ensure simultaneous action on a wide range of determinants of health such as water, sanitation, education, nutrition, social and gender equality. It aims to promote access to improved healthcare at household level through the female health activist (ASHA).

75. Answer (a) Explanation National Mission for Empowerment of Women (NMEW) / Mission Poorna Shakti is an initiative of the Government of India (GOI) for empowering women holistically. It is conceived as an umbrella mission with a mandate to strengthen inter-sectoral convergence and facilitate the process of coordinating all the women‘s welfare and socio economic development programmes across ministries and departments. The salient features of the National Mission for Empowerment of Women (NMEW) are: (i) to ensure economic empowerment of women, (ii) to ensure that violence against women is eliminated progressively, (iii) to ensure social empowerment of women with emphasis on health and education, (iv) to oversee gender mainstreaming of programmes, policies, institutional arrangements and processes of participating Ministries, Institutions and Organizations and (v) to undertake awareness generation as well as advocacy activities to fuel demand for benefits under various schemes and programmes and create, if required, structures at district, tehsil and village level with the involvement of Panchayats for their fulfilment. It aims to setup Poorna Shakti Kendra (PSK) at every village.

Page 15: PRELIMS MODEL TEST-11 ANSWER KEY AND EXPLANATION. PMT-11 to 15 Explanation.pdfTARGET PRELIMS 2017 GENERAL STUDIES PAPER - 1 PRELIMS MODEL TEST-11 ANSWER KEY AND EXPLANATION 1. Answer

NEO IAS 0484-3190310, 9446331522, 9446334122 Page 15 www.neoias.com | www.youtube.com/neoias | www.facebook.com/neoias | www.twitter.com/neoias

76. Answer (b) Explanation Shyama Prasad Mukherji Rurban Mission (SPMRM) was launched on 21st February, 2016. It is to stimulate local economic development, enhance basic services, and create well planned Rurban clusters. It aims at development of 300 rural growth clusters called ‗Rurban Clusters‘ (cluster of Smart Villages). The scheme is targeting to stop rising rural-urban migration. Clusters would be developed by provisioning economic activities, developing skills and local entrepreneurship and providing infrastructure facilities.

77. Answer (d) Explanation Mahatma Gandhi National Rural Employment Guarantee Act was passed in 2005 and scheme was launched in 2006. It aims at enhancing livelihood security of people in rural areas. It provides a legal guarantee of 100 days of work in a financial year. Additional 50 days of work in drought-hit areas to provide a relief to farmers. The work is usually on projects to build durable assets like roads, canals, ponds and wells. Every rural household whose adult members are willing to do unskilled manual work at a minimum wage rate are eligible to get the benefits. One-third of the stipulated work force must be women. Scheme is inclusive — with higher participation of women, SC and ST individuals.

78. Answer (a) Explanation PRAGATI (Pro-Active Governance And Timely Implementation) was launched on 25th March, 2015. It aimed at starting a culture of Pro-Active Governance and Timely Implementation. It also aimed at addressing common man‘s grievances, and simultaneously monitoring and reviewing important programmes and projects of the Government of India as well as projects flagged by State Governments. PRAGATI platform uniquely bundles three latest technologies: (i) Digital data management, (ii) video-conferencing and (iii) geo-spatial technology.

79. Answer (a) Explanation Subramanian panel‘s recommendations are based on MSP procurement operations and are well worth implementing. According to it, the Centre should immediately lift stockholding limits and export bans on pulses, along with launching a ―war effort‖ to procure moong, urad and tur at their announced minimum support prices (MSP).

80. Answer (d) Explanation Coarse cereals are also knowns as millets. Jowar(sorghum), Bajra (Pearl millet/Bull Rush millet) Ragi (Finger millet/Buck wheat) are the important millets grown in India.

81. Answer (c) Explanation Cluster Bean (guar gum): After 2010, India‘s much neglected and little-known Cluster Bean (guar gum) got a big export market. The new market was due to the multibillion shale and oil gas drilling business in U.S, which has found the gel or hydrocolloid produced from cluster beans to be an excellent lubricant for the ‗fracking‘ process. The leaves and beans of the guar plant have traditionally been used as an animal feed and as a vegetable for human consumption. Farmers take advantage of the nitrogen fixing abilities of the guar plant by using it as a green manure crop. Guar gum is used in baking as a dough thickener. It is used to thicken dairy products such as milk, yogurt, cheese, ice cream and sherbet. In processed foods guar gum is used as a thickener in salad dressing, sauces, ketchup, soup and many other products. Guar gum has medicinal uses which include: water-soluble fiber, bulk-forming laxative and creating a feeling of satiety. Industrially it is used in mining, petroleum drilling and textile manufacturing.

82. Answer (d) Explanation Sunflower as an oilseed is a newly introduced crop in the country. Due to source of high quality edible oil, sunflower oil is used as cooking oil. Its importance increases as sunflower oil is considered as a heart friendly oil. Besides oil, almost every part of sunflower has commercial value. This crop has gained importance due to its short duration of maturity, containing of excellent quality of oil, photo-insensitivity, wide adaptability into different kinds of cropping pattern, high-energy hull and drought tolerance. It is a short duration crop and completes its life cycle in about three months. Hence can be incorporated in different type of cropping pattern. Since it is a photo-insensitive crop, it can be grown throughout the year. Sunflower is cultivated as both kharif and rabi seasons but, two-thirds of production comes from rabi season crop.

83. Answer (c) Explanation Crops grown for the purpose of restoring or increasing the organic matter content in the soil are called Green manure crops. Green manuring are low cost and effective technology in minimising cost of fertilizers and

Page 16: PRELIMS MODEL TEST-11 ANSWER KEY AND EXPLANATION. PMT-11 to 15 Explanation.pdfTARGET PRELIMS 2017 GENERAL STUDIES PAPER - 1 PRELIMS MODEL TEST-11 ANSWER KEY AND EXPLANATION 1. Answer

NEO IAS 0484-3190310, 9446331522, 9446334122 Page 16 www.neoias.com | www.youtube.com/neoias | www.facebook.com/neoias | www.twitter.com/neoias

safeguarding productivity. Objectives of green manuring are to add Nitrogen to the companion or succeeding crop and add or sustain organic matter in the soil. Examples of leguminous Green Manures are as follows: Local name- Cowpea, Cluster bean (Guar), Green gram (Mung bean), Sesbania, Dhaincha, Sunhemp, Wild Indigo, Pillipesara, Berseem, Madras Indigo etc.

84. Answer (b) Explanation The Suez Canal is an artificial sea-level waterway in Egypt, connecting the Mediterranean Sea to the Red Sea through the Isthmus of Suez. 2016 marked the 60

th anniversary of Suez Canal Crisis. In 1956, Israeli armed

forces pushed into Egypt toward the Suez Canal after Egyptian president Gamal Abdel Nasser nationalized the canal, initiating the Suez Crisis. The Israelis soon were joined by French and British forces, which nearly brought the Soviet Union into the conflict, and damaged their relationships with the United States. In the end, the British, French and Israeli governments withdrew their troops.

85. Answer (c) Explanation Article 50 of the Treaty of Lisbon gives any EU member the right to quit unilaterally, and outlines the procedure for doing so. It gives the leaving country two years to negotiate an exit deal and once it's set in motion it can't

be stopped except by unanimous consent of all member states.

86. Answer (a) Explanation The Raisina Dialogue is an annual conference held in New Delhi, envisioned to be India‘s flagship conference of geopolitics and geo-economics. The conference is held jointly by Ministry of External Affairs, India and the Observer Research Foundation (ORF), an independent think tank based in India. Its inaugural session was held in March 2016. The second edition of the Raisina Dialogue was held in January 2017 under the theme ‗The New Normal: Multilateralism with Multipolarity‘.

87. Answer (b) Explanation World Food Programme (WFP) is the leading humanitarian organisation fighting hunger worldwide, delivering food assistance in emergencies and working with communities to improve nutrition and build resilience. WFP development projects focus on nutrition, especially for mothers and children, addressing malnutrition from the earliest stages through programmes targeting the first 1,000 days from conception to a child‘s second birthday, and later through school meals. WFP is the largest humanitarian organisation implementing school feeding programmes worldwide. WFP is governed by an Executive Board. It works closely with its two Rome-based sister organizations, the Food and Agriculture Organization of the United Nations and the International Fund for Agricultural Development. WFP‘s efforts focus on emergency assistance, relief and rehabilitation, development aid and special operations. It was established in 1961.

88. Answer (b) Explanation The United Nations Children's Fund (UNICEF) is a United Nations (UN) programme headquartered in New York City that provides humanitarian and developmental assistance to children and mothers in developing countries. It is a member of the United Nations Development Group (The UNDG unites 32 UN funds, programmes, specialized agencies, departments, and offices that play a role in development). It defends the rights of every child.

89. Answer (d) Explanation FIPIC is a multinational grouping for cooperation between India and 14 Pacific Islands nations which include Cook Islands, Fiji, Kiribati, Marshall Islands, Micronesia, Nauru, Niue, Samoa, Solomon Islands, Palau, Papua New Guinea, Tonga, Tuvalu and Vanuatu. It was launched during Indian Prime Minister, Mr. Narendra Modi's visit to Fiji in November 2014.Though these countries are relatively small in land area and distant from India, many have large exclusive economic zones (EEZs), and offer promising possibilities for fruitful cooperation. India's focus has largely been on the Indian Ocean where it has sought to play a major role and protect its strategic and commercial interests. The FIPIC initiative marks a serious effort to expand India's

engagement in the Pacific region.

90. Answer (a) Explanation Diego Garcia, which is a part of Chagos archipelago in the Indian Ocean, was leased by Britain to the US for the military base in 1966 for 50 years (two years before Mauritius independence). Nearly 2,000 islanders were driven out and settled in Mauritius and Seychelles. London administers it as the British Indian Ocean Territory. The lease expired in December 2016, but has been renewed by London until December 2036. London also announced that the islanders will not be allowed to return. In 2010, Britain created a Maritime Protection Area in the Chagos island

Page 17: PRELIMS MODEL TEST-11 ANSWER KEY AND EXPLANATION. PMT-11 to 15 Explanation.pdfTARGET PRELIMS 2017 GENERAL STUDIES PAPER - 1 PRELIMS MODEL TEST-11 ANSWER KEY AND EXPLANATION 1. Answer

NEO IAS 0484-3190310, 9446331522, 9446334122 Page 17 www.neoias.com | www.youtube.com/neoias | www.facebook.com/neoias | www.twitter.com/neoias

allegedly to stop return of islanders. Recently, Britain and the United States have approached India to leverage its influence with Mauritius on the contentious issue of Diego Garcia.

91. Answer (d) Explanation Ram Sethu is a stretch of limestone shoals running from Pamban Island near Rameswaram in South India to the Mannar Island near the northern coast of Sri Lanka. While there are geological theories on its natural formation, many devout Hindus believe that it was built by the army of Lord Ram to go to Lanka to wage war with its king, Ravan. The Indian Council of Historical Research (ICHR) is set to undertake an archaeological exploration into whether the Ram Sethu is a natural or man-made phenomenon.

92. Answer (c) Explanation

Page 18: PRELIMS MODEL TEST-11 ANSWER KEY AND EXPLANATION. PMT-11 to 15 Explanation.pdfTARGET PRELIMS 2017 GENERAL STUDIES PAPER - 1 PRELIMS MODEL TEST-11 ANSWER KEY AND EXPLANATION 1. Answer

NEO IAS 0484-3190310, 9446331522, 9446334122 Page 18 www.neoias.com | www.youtube.com/neoias | www.facebook.com/neoias | www.twitter.com/neoias

93. Answer (c) Explanation Dafla Hills – Arunachal Pradesh (Importance: Pakke Wildlife Sanctuary and Tiger Reserve) Nallamalai hills – Andhra Pradesh (Importance: Nagarjunsagar-Srisailam Tiger Reserve) Agasthyamala – Kerala (Importance: Agasthyamala Biosphere Reserve was included in the World Network of Biosphere Reserves of UNESCO). Maikala range – Madhya Pradesh and Chhattisgarh (Importance: Kanha National Park)

94. Answer (a) Explanation At the 39th session of the International Civil Aviation Organization (ICAO) Assembly in 2016, ICAO‘s Member States adopted a global carbon offsetting scheme for international aviation. ICAO‘s Carbon Offset and Reduction Scheme for International Aviation (CORSIA) is set to commence with a voluntary period (2021-2026) after which it will become mandatory. Under the Carbon Offsetting Scheme for International Aviation (CORSIA), aircraft operators will be required to purchase offsets, or ―emission units‖, for the growth in CO2 emissions covered by the scheme. CORSIA aims to address any annual increase in total CO2 emissions from international civil aviation above 2020 levels. In order to address the concerns of developing States and to take into account the special circumstances and respective capabilities of States, CORSIA will be implemented in phases.

95. Answer (d) Explanation Gravitational waves are 'ripples' in the fabric of space-time caused by some of the most violent and energetic processes in the Universe. Albert Einstein predicted the existence of gravitational waves in 1916 in his general theory of relativity. The strongest gravitational waves are produced by catastrophic events such as colliding black holes, the collapse of stellar cores (supernovae), coalescing neutron stars or white dwarf stars, the slightly wobbly rotation of neutron stars that are not perfect spheres, and the remnants of gravitational radiation created by the birth of the Universe itself. The milestone of detecting gravitational waves was achieved by Laser Interferometer Gravitational Wave Observatory (LIGO), a pair of giant laser detectors in the U.S located in Louisiana and Washington State. The detection at once improves our understanding of the workings of the universe and, more important, throws open a big opportunity to study it from completely new angles. It opens the way to get information about the evolution of galaxies and black holes.

96. Answer (c) Explanation Natural gas hydrates are a naturally occurring, ice-like combination of natural gas and water found in oceans and polar regions. The amount of gas within the world‘s gas hydrate accumulations is estimated to greatly exceed the volume of all known conventional gas resources. They are considered as vast resources of natural gas and are known to occur in marine sediments on continental shelf margins. Gas hydrate resources in India are estimated at 1,894 trillion cubic meters and these deposits occur in Western, Eastern and Andaman offshore areas. The gas hydrate deposits were discovered in coarse-grained sand-rich depositional systems in the Krishna-Godavari Basin and is made up of a sand-rich, gas-hydrate-bearing fan and channel-levee gas hydrate prospects.

Page 19: PRELIMS MODEL TEST-11 ANSWER KEY AND EXPLANATION. PMT-11 to 15 Explanation.pdfTARGET PRELIMS 2017 GENERAL STUDIES PAPER - 1 PRELIMS MODEL TEST-11 ANSWER KEY AND EXPLANATION 1. Answer

NEO IAS 0484-3190310, 9446331522, 9446334122 Page 19 www.neoias.com | www.youtube.com/neoias | www.facebook.com/neoias | www.twitter.com/neoias

This discovery is the result of the most comprehensive gas hydrate field venture in the world to date, made up of scientists from India, Japan and the United States.

97. Answer (d) Explanation UDAY is a Central scheme providing for the financial turnaround and revival of State electricity distribution utilities. Under the scheme, States will take over 75% of the debt of their respective DISCOMs. The scheme envisages:

Financial Turnaround

Operational improvement

Reduction of cost of generation of power

Development of Renewable Energy

Energy efficiency & conservation

BENEFITS TO PARTICIPATING STATES Reduction in Cost of power through Central Support

Increased supply of domestic coal

Allocation of coal linkages at notified prices

Coal price rationalization

Coal linkage rationalization & allowing coal swaps

Supply of washed & crushed coal

Additional coal at notified prices

Faster completion of Interstate Transmission lines

Power purchase through transparent competitive bidding Additional priority funding under Deen Dayal Upadhyaya Gram Jyoti Yojana (DDUGJY), Integrated Power

Development Scheme (IPDS), etc.

98. Answer (a) Explanation The 1987 landmark Montreal Protocol was designed to close the growing hole in the ozone layer by banning ozone-depleting coolants called chlorofluorocarbons, or ―CFCs‖. Meanwhile, in Kigali, negotiators from 197 countries agreed to amend the Montreal Protocol to phase down super potent greenhouse gas emissions known as hydrofluorocarbons or ―HFCs.‖ HFCs are estimated to have a thousand times more heat trapping potency than CO2 and are commonly used in refrigeration and air conditioning. With global temperatures rising, increasing populations and higher rates of urbanization there is a growing demand for air-conditioning and the use of high-emitting coolants. It is expected that the measures of the Kigali Amendment will help prevent up to 0.5 degrees Celsius of global warming by the end of the century. The amended Montreal Protocol will bind countries to their HFC reduction schedules from 2019. There are also penalties for non-compliance as well as clear directives that

developed countries provide enhanced funding support estimated at billions of dollars globally.

The Amendment will enter into force on 1 January 2019, provided that it is ratified by at least 20 parties to the Montreal Protocol. If that condition is not met by that date, the amendment will become effective on the 90

th day

following the date of ratification by the 20th party.

99. Answer (b) Explanation Jawaharlal Nehru Port – Maharashtra Kamarajar Port or Ennore Port – Tamil Nadu Paradip – Odisha Mormugao – Goa

100. Answer (d) Explanation The 9.2 km-long road tunnel that bores through the belly of the lower Himalayas between Chenani in Udhampur district and Nashri in Ramban district is an achievement of engineering that incorporates India‘s first fully integrated mechanism to externally control everything from the movement of vehicles to the inflow and outflow of air, and even the evacuation of passengers or vehicles in distress. It is not only India‘s longest highways tunnel but also Asia‘s longest bi-directional highways tunnel. The 9.2 km tunnel, which will bypass snow- and landslide-prone Kud, Patnitop and Batote on National Highway 44, marks significant roadbuilding first in India, including an unprecedented stress on user safety.

Page 20: PRELIMS MODEL TEST-11 ANSWER KEY AND EXPLANATION. PMT-11 to 15 Explanation.pdfTARGET PRELIMS 2017 GENERAL STUDIES PAPER - 1 PRELIMS MODEL TEST-11 ANSWER KEY AND EXPLANATION 1. Answer

NEO IAS 0484-3190310, 9446331522, 9446334122 Page 20 www.neoias.com | www.youtube.com/neoias | www.facebook.com/neoias | www.twitter.com/neoias

TARGET PRELIMS 2017 GENERAL STUDIES

PAPER - 1

PRELIMS MODEL TEST-12

ANSWER KEY AND EXPLANATION

1. Answer (a) Explanation LAF is a tool used in monetary policy that allows banks to borrow money through repurchase agreements. This arrangement allows banks to respond to liquidity pressures and is used by governments to assure basic stability in the financial markets. It consists of repo and reverses repo operations. Buying and selling of the government securities are part of the Open Market Operations.

2. Answer (c) Explanation Central Public Sector Enterprises (CPSE) ETF, which tracks the Nifty CPSE Index, is a concentrated portfolio of 10 stocks, whose main aim is to help the Government of India (GOI) in disinvesting its stake in a few CPSEs via the ETF route. The portfolio comprises of 10 Maharatna and Navaratna CPSEs whose weightage in the index are given below.

1 A 11 B 21 A 31 C 41 B 51 A 61 C 71 B 81 B 91 C

2 C 12 B 22 D 32 D 42 D 52 D 62 A 72 D 82 C 92 A

3 D 13 C 23 D 33 C 43 C 53 C 63 D 73 B 83 B 93 D

4 D 14 D 24 A 34 D 44 C 54 D 64 C 74 C 84 D 94 D

5 C 15 D 25 D 35 D 45 C 55 A 65 D 75 B 85 C 95 B

6 C 16 A 26 A 36 A 46 A 56 A 66 A 76 C 86 D 96 C

7 C 17 C 27 D 37 A 47 A 57 A 67 B 77 C 87 C 97 A

8 A 18 D 28 C 38 C 48 D 58 D 68 B 78 C 88 B 98 C

9 C 19 A 29 B 39 A 49 C 59 B 69 C 79 D 89 D 99 A

10 D 20 B 30 C 40 A 50 B 60 A 70 C 80 D 90 A 100 D

Date: 23-04-2017

Page 21: PRELIMS MODEL TEST-11 ANSWER KEY AND EXPLANATION. PMT-11 to 15 Explanation.pdfTARGET PRELIMS 2017 GENERAL STUDIES PAPER - 1 PRELIMS MODEL TEST-11 ANSWER KEY AND EXPLANATION 1. Answer

NEO IAS 0484-3190310, 9446331522, 9446334122 Page 21 www.neoias.com | www.youtube.com/neoias | www.facebook.com/neoias | www.twitter.com/neoias

3. Answer (d) Explanation The Indian banking system suffers from the inefficient monetary transmission due to following reasons: 1. Deposit rates of the small savings schemes are higher compared to the bank deposits. Banks find it more difficult to cut lending rates immediately after a cut in the policy rate because the cost of deposits does not adjust commensurately and immediately, given the fixed nature of deposit contracts. However, banks can raise lending rates far more quickly after a policy rate hike, because loans are mostly at variable rates, and can be re-priced faster. There is a competitive constraint of reducing deposit rates too much and too quick as well. If bank deposit rates are lower than small savings rates which are administratively fixed, there could be some deposit flight to small savings schemes providing higher returns. 2. Twin balance sheet problem The banking system continues to be under stress due to rising non-performing assets (NPAs), which along with restructured assets are estimated at about 10 per cent of total assets currently. The stressed loans are concentrated across five key sectors - infrastructure, iron and steel, textiles, aviation and mining - which constitute 24 per cent of total advances and 52 per cent of total stressed advances. Given this backdrop, it is natural to expect banks to be risk-averse and show reluctance in lending further to these sectors. 3. High Statutory Reserve Requirements Some commercial banks have argued that a cash reserve ratio (CRR) cut is required to make the monetary transmission effective. Currently, commercial banks have to maintain four per cent CRR with RBI, on which they do not earn any interest. A meaningful cut in the CRR will not only help inject additional liquidity into the system, thereby freeing resources to lend, but also help banks to pass on rate cuts without taking a hit on their net interest margin.

4. Answer (d) Explanation STRUCTURAL UNEMPLOYMENT It is the unemployment caused by structural changes like rapidly growing population; fall in the rate of capital formation; technological change etc., in the economy.

5. Answer (c) Explanation The GII is an inequality index. It measures gender inequalities in three important aspects of human development—reproductive health, measured by maternal mortality ratio and adolescent birth rates; empowerment, measured by proportion of parliamentary seats occupied by females and proportion of adult females and males aged 25 years and older with at least some secondary education; and economic status, expressed as labour market participation and measured by labour force participation rate of female and male populations aged 15 years and older. The GII is built on the same framework as the IHDI—to better expose differences in the distribution of achievements between women and men. It measures the human development costs of gender inequality. Thus, the higher the GII value the more disparities between females and males and the more loss to human development.

6. Answer (c)

Explanation Based on the GDP (Purchasing Power Parity $) in decreasing order the BRICS countries can be arranged as China-India-Russia-Brazil-South Africa. The GDP in market rates follows the arrangement China-India- Brazil-Russia-South Africa.

7. Answer (c) Explanation Purchasing Power Parity (PPP) is an economic theory that compares different countries' currencies through a market "basket of goods" approach. According to this concept, two currencies are in equilibrium or at par when a market basket of goods (taking into account the exchange rate) is priced the same in both countries. PPP is

Page 22: PRELIMS MODEL TEST-11 ANSWER KEY AND EXPLANATION. PMT-11 to 15 Explanation.pdfTARGET PRELIMS 2017 GENERAL STUDIES PAPER - 1 PRELIMS MODEL TEST-11 ANSWER KEY AND EXPLANATION 1. Answer

NEO IAS 0484-3190310, 9446331522, 9446334122 Page 22 www.neoias.com | www.youtube.com/neoias | www.facebook.com/neoias | www.twitter.com/neoias

determined in each country based on its relative cost of living and inflation rates. Purchasing power plus parity ultimately means equalizing the purchasing power of two differing currencies by accounting for differences in inflation rates and cost of living. Purchasing power parity is used worldwide to compare the income levels in different countries. PPP thus makes it easy to understand and interpret the data of each country. As a light-hearted annual test of PPP, The Economist has tracked the price of McDonald's Corp.‘s Big Mac burger in many countries since 1986.The highly publicized Big Mac Index is used to measure the purchasing power

parity (PPP) between nations, using the price of a Big Mac as the benchmark.

8. Answer (a) Explanation Under the Repo or Repurchase Option the banks borrow money from the RBI to meet the short term needs by selling the securities to RBI with an agreement to repurchase the same at the predetermined rate and date.

The rate charged by the RBI is called the repo rate.

9. Answer (c) Explanation Most of the States in the country need to call a special session of their legislature to pass the State GST bills before May end. All the 29 states and 2 Union Territories with legislatures i.e. Delhi and Puducherry need to pass the State GST bills within almost 70 odd days, if the GST regime is to be rolled out by July end. Telangana is the first state to pass the State GST bills.

10. Answer (d) Explanation

The periods 1966-69, 1979-80 and 1990-92 were the deviation from Five Year Plans under the Indian Economy.

11. Answer (b) Explanation The Human Development Index (HDI) is a summary measure of average achievement in key dimensions of human development: a long and healthy life, being knowledgeable and have a decent standard of living. The HDI is the geometric mean of normalized indices for each of the three dimensions. The health dimension is assessed by life expectancy at birth; the education dimension is measured by mean of years of schooling for adults aged 25 years and more and expected years of schooling for children of school entering age. The standard of living dimension is measured by gross national income per capita. The HDI uses the logarithm of income, to reflect the diminishing importance of income with increasing GNI. The scores for the three HDI dimension indices are then aggregated into a composite index using geometric mean.

The Multidimensional Poverty Index (MPI), published for the first time in the 2010 Report, complements monetary measures of poverty by considering overlapping deprivations suffered by individuals at the same time. The index identifies deprivations across the same three dimensions as the HDI and shows the number of people who are multidimensionally poor (suffering deprivations in 33% or more of the weighted indicators) and the number of weighted deprivations with which poor households typically contend with. About 1.5 billion people in the 102 developing countries currently covered by the MPI—about 29 percent of their population — live in multidimensional poverty.

12. Answer (b)

Explanation

Page 23: PRELIMS MODEL TEST-11 ANSWER KEY AND EXPLANATION. PMT-11 to 15 Explanation.pdfTARGET PRELIMS 2017 GENERAL STUDIES PAPER - 1 PRELIMS MODEL TEST-11 ANSWER KEY AND EXPLANATION 1. Answer

NEO IAS 0484-3190310, 9446331522, 9446334122 Page 23 www.neoias.com | www.youtube.com/neoias | www.facebook.com/neoias | www.twitter.com/neoias

India slipped down one place from 130 to 131 among the 188 countries ranked in terms of human development, says the 2016 Human Development Report (HDR) released by the United Nations Development Programme (UNDP). India‘s human development index (HDI) value of 0.624 puts it in the ―medium human development‖ category, alongside countries such as Congo, Namibia and Pakistan. It is ranked third among the SAARC countries, behind Sri Lanka (73) and the Maldives (105), both of which figure in the ―high human development‖ category. While India‘s HDI value increased from 0.428 in 1990 to 0.624 in 2015, it still had the lowest rank among BRICS nations. However, its average annual growth in HDI (1990-2015) was higher than that of other medium HD

countries.

13. Answer (c) Explanation The Human Development Index (HDI) is a summary measure of average achievement in key dimensions of human development: a long and healthy life, being knowledgeable and have a decent standard of living. The HDI is the geometric mean of normalized indices for each of the three dimensions. The HDI simplifies and captures only part of what human development entails. It does not reflect on inequalities, poverty, human security, empowerment, etc. The HDRO offers the other composite indices as broader proxy on

some of the key issues of human development, inequality, gender disparity and poverty.

14. Answer (d) Explanation Though the appreciation of the Indian currency against the international currencies shows the strength of the Indian economy, it would affect the export competitiveness of the country in the global market.

15. Answer (d) Explanation The N.K. Singh panel to review India‘s fiscal discipline rules has recommended a debt to GDP ratio of 38.7% for the central government, 20% for the state governments together and a fiscal deficit of 2.5% of GDP (gross domestic product), both by financial year 2022-23. The panel has recommended enacting a new Debt and Fiscal Responsibility Act after repealing the existing Fiscal Responsibility and Budget Management (FRBM) Act, and creating a fiscal council. The proposed three member fiscal council will prepare multiyear fiscal forecasts for the central and state governments (together called the general government) and provide an independent assessment of the central government‘s fiscal performance and compliance with targets set under the new law. The committee favours a debt to GDP ratio of 60% for the general government by 2022-23, 40% (38.74%) for the central government and 20% for state governments. Within the framework, the committee has recommended adopting fiscal deficit as the key operational target consistent with achieving the medium term debt ceiling, at 3% of GDP for three years, between 2017-18 and 2019-20. Revenue deficit to GDP ratio has been envisaged to decline steadily by 0.25 percentage points each year from 2.3% in 2016-17 to 0.8% in 2022-23. However, to deal with unforeseen events such as war, calamities of national proportion, collapse of agricultural activity, far reaching structural reforms, and sharp decline in real output growth of at least 3 percentage points, the committee has specified deviation in fiscal deficit target of not more than 0.5 percentage points. Similar to the escape clause, this buoyancy clause can be invoked by the government after formal consultations and advice of the fiscal council. ―If there is a sharp increase in real output growth of at least 3 percentage points above the average for the previous four quarters, fiscal deficit must fall by at least 0.5 percentage points below the target,‖ the committee said.

16. Answer (a) Explanation RBI has created a specialised cell (C-SITE) within its supervision department to conduct detailed IT examination of banks‘ cyber security preparedness, to identify the gaps and to monitor the progress of remedial measures. More than 30 major banks were to be covered by the detailed IT examination in 2016-17 and all banks

are to be covered by 2017-18.

17. Answer (c) Explanation Currency chests are branches of selected banks authorised by the RBI to stock rupee notes and coins.

18. Answer (d) Explanation India and Singapore have amended the DTAA for the avoidance of double taxation and prevention of fiscal evasion with respect to taxes on income, by signing a Third Protocol. This is in line with India‘s treaty policy to prevent double non-taxation, curb revenue loss and check the menace of black money through automatic exchange of information, as reflected in India‘s recently revised treaties with Mauritius and Cyprus and the joint declaration signed with Switzerland.

Page 24: PRELIMS MODEL TEST-11 ANSWER KEY AND EXPLANATION. PMT-11 to 15 Explanation.pdfTARGET PRELIMS 2017 GENERAL STUDIES PAPER - 1 PRELIMS MODEL TEST-11 ANSWER KEY AND EXPLANATION 1. Answer

NEO IAS 0484-3190310, 9446331522, 9446334122 Page 24 www.neoias.com | www.youtube.com/neoias | www.facebook.com/neoias | www.twitter.com/neoias

The Protocol for amendment of the India-Mauritius Convention signed on 10th May, 2016, provides for source-based taxation of capital gains arising from alienation of shares acquired from 1st April, 2017 in a company resident in India. Simultaneously, investments made before 1st April, 2017 have been grandfathered and will not be subject to capital gains taxation in India. Where such capital gains arise during the transition period from 1st April, 2017 to 31st March, 2019, the tax rate will be limited to 50% of the domestic tax rate of India. However, the benefit of 50% reduction in tax rate during the transition period shall be subject to the Limitation of Benefits Article. Taxation in India at full domestic tax rate will take place from financial year 2019-20 onwards. As the Singapore treaty is coterminous with the Mauritius treaty, the above provisions are applicable to it also. The revised treaty signed by India and Mauritius in November 2016 gives India the right to tax capital gains from sale of shares on investments made by Cyprus-based companies after 1 April 2017. However, it does not provide for a transitionary period like the one provided by the India-Mauritius treaty, wherein only half the capital gains tax rate will be applicable between 2017 and 2019. New Revised Double Taxation Avoidance Agreement (DTAA) between India and Republic of Korea comes into force with effect from 12th September, 2016; it will have effect in India in respect of income derived in fiscal years beginning on or after 1st April, 2017. The revised DTAA inserts new Limitation of Benefits Article i.e. anti-abuse provisions to ensure that the benefits of the Agreement are availed only by the genuine residents of both the countries.

19. Answer (a) Explanation Goods and Service Tax is a major economic reform in India after the 1991 economic liberalisation. GST, proposed to be rolled out from 1

st July 2017, will have a four tier structure with rates ranging from 5% to 28%. But regrettably,

a large chunk of the economy, which includes real estate, electricity, alcohol and petroleum products, is out of GST.

20. Answer (b) Explanation At the Central level, the following taxes are being subsumed:

(a) Central Excise Duty, (b) Additional Excise Duty, (c) Service Tax, (d) Additional Customs Duty commonly known as Countervailing Duty, and (e) Special Additional Duty of Customs.

At the State level, the following taxes are being subsumed: (a) Subsuming of State Value Added Tax/Sales Tax, (b) Entertainment Tax (other than the tax levied by the local bodies), Central Sales Tax (levied by the Centre

and collected by the States), (c) Octroi and Entry tax, (d) Purchase Tax, (e) Luxury tax, and (f) Taxes on lottery, betting and gambling.

21. Answer (a) Explanation The Insolvency and Bankruptcy Code 2016

The Code creates time-bound processes for insolvency resolution of companies and individuals. These processes will be completed within 180 days. If insolvency cannot be resolved, the assets of the borrowers may be sold to repay creditors.

The resolution processes will be conducted by licensed insolvency professionals (IPs). These IPs will be members of insolvency professional agencies (IPAs). IPAs will also furnish performance bonds equal to the assets of a company under insolvency resolution.

Information utilities (IUs) will be established to collect, collate and disseminate financial information to facilitate insolvency resolution.

The National Company Law Tribunal (NCLT) will adjudicate insolvency resolution for companies. The Debt Recovery Tribunal (DRT) will adjudicate insolvency resolution for individuals.

The Insolvency and Bankruptcy Board of India will be set up to regulate functioning of IPs, IPAs and IUs.

22. Answer (d) Explanation International Monetary Financial Committee The IMFC advises and reports to the IMF Board of Governors on the supervision and management of the international monetary and financial system, including on responses to unfolding events that may disrupt the system. The IMFC usually meets twice a year. The size and the composition of the IMFC mirrors that of the Executive Board. The IMFC has 24 members who are central bank governors, ministers, or others of comparable rank and who are usually drawn from the governors of the Fund‘s 189 member countries. A number of international institutions, including the World Bank, participate as observers in the IMFC‘s meetings.

Page 25: PRELIMS MODEL TEST-11 ANSWER KEY AND EXPLANATION. PMT-11 to 15 Explanation.pdfTARGET PRELIMS 2017 GENERAL STUDIES PAPER - 1 PRELIMS MODEL TEST-11 ANSWER KEY AND EXPLANATION 1. Answer

NEO IAS 0484-3190310, 9446331522, 9446334122 Page 25 www.neoias.com | www.youtube.com/neoias | www.facebook.com/neoias | www.twitter.com/neoias

Group 10 The Group of Ten (G10) refers to the group of countries that have agreed to participate in the General Arrangements to Borrow (GAB), a supplementary borrowing arrangement that can be invoked if the IMF‘s resources are estimated to be below a member‘s needs. G10 includes 11 members. The following international organizations are official observers of the activities of the G10: the Bank for International Settlements (BIS), the European Commission, the IMF, and the OECD.

G10 Members

Belgium Netherlands

Canada Sweden

France Switzerland

Germany United Kingdom

Italy United States

Japan

Group 15 The Group of Fifteen (G15) was established at the Ninth Non-Aligned Summit Meeting in Belgrade, then Yugoslavia, in September 1989. It is composed of countries from Latin America, Africa, and Asia with a common goal of enhanced growth and prosperity. The G15 focuses on cooperation among developing countries in the areas of investment, trade, and technology. The membership of the G15 has since expanded to 17 countries but the name has remained unchanged.

G15 Members

Algeria Indonesia Nigeria

Argentina Iran Senegal

Brazil Jamaica Sri Lanka

Chile Kenya Venezuela,

Egypt Malaysia Zimbabwe

India Mexico

Group 77 The Group of 77 is the largest intergovernmental organization of developing countries in the United Nations, which provides the means for the countries of the South to articulate and promote their collective economic interests and enhance their joint negotiating capacity on all major international economic issues within the United Nations system, and promote South-South cooperation for development.

23. Answer (d) Explanation The India-Singapore DTAA at present provides for residence based taxation of capital gains of shares in a company. The Third Protocol amends the DTAA with effect from 1st April, 2017 to provide for source based taxation of capital gains arising on transfer of shares in a company. This will curb revenue loss, prevent double non-taxation and streamline the flow of investments.

24. Answer (a) Explanation The Reserve Bank of India Act, 1934 (RBI Act) has been amended by the Finance Act, 2016, to provide for a statutory and institutionalised framework for a Monetary Policy Committee, for maintaining price stability, while keeping in mind the objective of growth. The Monetary Policy Committee would be entrusted with the task of fixing the benchmark policy rate (repo rate) required to contain inflation within the specified target level. As per the provisions of the RBI Act, out of the six Members of Monetary Policy Committee, three Members will be from the RBI and the other three Members of MPC will be appointed by the Central Government. The committee is headed by the Governor of the RBI. The Members of the Monetary Policy Committee appointed by the Central Government shall hold office for a period of four years and there will not be eligible for reappointment. Decisions in MPC are taken based on voting with each member having a single vote and the Chairman having casting vote in case of tie.

25. Answer (d) Explanation The Index of Industrial Production (IIP) is an index which shows the growth rates in different industry groups of the economy in a stipulated period of time. The IIP index is computed and published by the Central Statistical Organisation (CSO) on a monthly basis. Currently IIP figures are calculated considering 2004-05 as base year. IIP data is based on the 3 sectors viz. mining, manufacturing and electricity with maximum weightage to manufacturing sector. The data compilation is received from the 16 different agencies across the country.

26. Answer (a) Explanation

Page 26: PRELIMS MODEL TEST-11 ANSWER KEY AND EXPLANATION. PMT-11 to 15 Explanation.pdfTARGET PRELIMS 2017 GENERAL STUDIES PAPER - 1 PRELIMS MODEL TEST-11 ANSWER KEY AND EXPLANATION 1. Answer

NEO IAS 0484-3190310, 9446331522, 9446334122 Page 26 www.neoias.com | www.youtube.com/neoias | www.facebook.com/neoias | www.twitter.com/neoias

The new ₹500 notes in the Mahatma Gandhi (New) Series are different from the present series in colour, size, theme, location of security features and design elements. The size of the new note is 66mm x 150mm. The colour of the notes is stone grey and the predominant new theme is Indian heritage site - Red Fort. The Reserve Bank of India is introducing new design banknotes in the denomination of ₹2000 as part of Mahatma Gandhi (New) Series. The new denomination has motif of the Mangalyaan on the reverse, depicting the country's first venture in interplanetary space. The base colour of the note is magenta. The note has other designs, geometric patterns aligning with the overall colour scheme, both on the obverse and the reverse. The size of the new note is 66mm x 166mm. RBI has opened an educative microsite ‗Paisa boltahai‘ which includes a film for public awareness of bank notes and other details of the same. The RBI can issue bank notes in the denomination of ₹1000, ₹5000 and ₹10,000 or any other denominations that central government may specify.

27. Answer (d) Explanation The Open Market operations refer to the sale and purchase of securities, bills and bonds of government as well as the private financial institutions by the central bank.

28. Answer (c) Explanation During the tight money policy, the Central bank tries to reduce the money supply in the system and it may increase the repo rate to achieve the desired result.

29. Answer (b) Explanation The Index of Industrial Production (IIP) is an index which shows the growth rates in different industry groups of the economy in a stipulated period of time. The IIP index is computed and published by the Central Statistical Organisation (CSO) on a monthly basis. Currently IIP figures are calculated considering 2004-05 as base year.

30. Answer (c) Explanation Inflation means the sustained increase in the general level of prices for goods and services. Inflation negatively affects the debenture or bond holders and as well as savers.

31. Answer (c) Explanation Reflation means the deliberate action of the government to increase the rate of inflation in the economy. It is usually done to redeem the economy from the deflationary situation.

32. Answer (d) Explanation The demand pull and cost push inflation are affected by forces of demand and supply. Factors affecting the demand are:

1. Increase in money supply 2. Increase in disposable income. 3. Cheap money policy. 4. Increase in public expenditure. 5. Repayment of public debt.

Factors affecting supply: 1. Shortage of factors of production 2. Industrial dispute 3. Natural calamities 4. Artificial scarcities 5. Increase in exports 6. International factors like oil price hike

33. Answer (c) Explanation The monthly CPI (combined), with base 2012=100 is now taken as the measure of headline inflation and is tracked by the RBI to anchor its monetary policy.

34. Answer (d) Explanation The Indian Performing Right Society Limited (IPRS) came into existence on 23rd August 1969. The IPRS is a representative body of Owners of Music, viz. The Composers, Lyricists (or Authors) and the Publishers of Music and is also the sole Authorized Body to issue Licences permitting usage of Music within India by any person. Composers are those who are better known as Music Directors, Authors are better known as Lyricists, Publishers of Music are the Producers of Films and Music Companies, or those who hold Publishing Rights of the

Page 27: PRELIMS MODEL TEST-11 ANSWER KEY AND EXPLANATION. PMT-11 to 15 Explanation.pdfTARGET PRELIMS 2017 GENERAL STUDIES PAPER - 1 PRELIMS MODEL TEST-11 ANSWER KEY AND EXPLANATION 1. Answer

NEO IAS 0484-3190310, 9446331522, 9446334122 Page 27 www.neoias.com | www.youtube.com/neoias | www.facebook.com/neoias | www.twitter.com/neoias

Musical Works. The Society is a non-profit making Organization and is a Company Limited by Guarantee and Registered under the Companies Act, 1956. The business of IPRS is to issue Licences to users of music and collect Royalties from them, for and on behalf of its Members i.e. the Authors, the Composers and the Publishers of Music and distribute this Royalty amongst them after deducting its administrative costs.

35. Answer (d) Explanation There are two Public Sector Shipyards viz. Cochin Shipyard Limited (CSL), Kochi and Hooghly Dock & Port Engineers Limited (HDPEL), Kolkata under the administrative control of Ministry of Shipping and four Public Sector Shipyards under the administrative control of Ministry of Defence viz. Mazagon Dock Limited (MDL), Mumbai, Garden Reach Shipbuilders & Engineers Limited (GRSEL), Kolkata, Hindustan Shipyard Limited (HSL), Visakhapatnam and Goa Shipyard Limited (GSL), Goa. Cochin Shipyard Ltd (CSL), India‘s largest public sector shipyard, is planning to raise about Rs. 1,400-1,500 crore through an initial public offering (IPO) and has filed initial papers with the market regulator. On completion of a successful IPO, CSL will become the first of five state-owned shipbuilding companies to be publicly listed. In November 2015, the Cabinet Committee on Economic Affairs has approved the share sale of CSL.

36. Answer (a) Explanation Copyright (or author‘s right) is a legal term used to describe the rights that creators have over their literary and artistic works. Works covered by copyright range from books, music, paintings, sculpture, and films, to computer programs, databases, advertisements, maps, and technical drawings. Works commonly protected by copyright throughout the world include:

literary works such as novels, poems, plays, reference works, newspaper articles;

computer programs, databases;

films, musical compositions, and choreography;

artistic works such as paintings, drawings, photographs, and sculpture;

architecture; and

Advertisements, maps, and technical drawings. Copyright protection extends only to expressions, and not to ideas, procedures, methods of operation or mathematical concepts as such. Copyright may or may not be available for a number of objects such as titles, slogans, or logos, depending on whether they contain sufficient authorship. There are two types of rights under copyright:

economic rights, which allow the rights owner to derive financial reward from the use of his works by others; and

moral rights, which protect the non-economic interests of the author. Economic rights have a time limit, which can vary according to national law. In those countries which are members of the Berne Convention, the time limit should be equal to or longer than 50 years after the creator‘s death. Longer periods of protection may however be provided at the national level.

37. Answer (a) Explanation The focus areas of NDB are: INFRASTRUCTURE

Infrastructure development is the key driver of economic and social growth. In the context of developing nations, infrastructural deficiencies are a matter of concern. NDB strive to identify the gaps between ‗needs‘ and ‗funding‘. NDB‘s mission is to bridge these gaps and be a partner in bringing about truly holistic development. SUSTAINABLE DEVELOPMENT

The 21st century has brought with it tremendous development. However, this progress has been skewed, insufficient & often harmful to environment. NDB committed to be a partner in bringing about sustainable development. NDB is looking forward to partner with initiatives that drive growth and employment while ensuring environmental protection.

38. Answer (c) Explanation Debentures are long-term financial instruments which acknowledge a debt obligation towards the issuer. Some debentures have a feature of convertibility into shares after a certain point of time at the discretion of the owner. The debentures which can't be converted into shares or equities are called non-convertible debentures (or NCDs). Non-convertible debentures are used as tools to raise long-term funds by companies through a public issue. To compensate for this drawback of non-convertibility, lenders are usually given a higher rate of return compared to convertible debentures.

39. Answer (a) Explanation

Page 28: PRELIMS MODEL TEST-11 ANSWER KEY AND EXPLANATION. PMT-11 to 15 Explanation.pdfTARGET PRELIMS 2017 GENERAL STUDIES PAPER - 1 PRELIMS MODEL TEST-11 ANSWER KEY AND EXPLANATION 1. Answer

NEO IAS 0484-3190310, 9446331522, 9446334122 Page 28 www.neoias.com | www.youtube.com/neoias | www.facebook.com/neoias | www.twitter.com/neoias

India is planning to introduce a new monetary-policy tool in the financial year 2017-18 to better manage a banking system swimming in excess cash. The so-called Standing Deposit Facility, or SDF, will help the Reserve Bank of India absorb surplus funds without having to provide lenders collateral in exchange. In 2014, a panel led by RBI Governor Urjit Patel -- who was at that time a deputy -- had proposed the introduction of the SDF as part of measures to improve the monetary-policy framework.

40. Answer (a) Explanation SBI merged with its 5 associated banks on 1 April 2017. These are State Bank of Bikaner and Jaipur, State Bank of Travancore, State Bank of Mysore, State Bank of Hyderabad and State Bank of Patiala. SBI also absorbed Bharatiya Mahila Bank on the same date. The merging of State Bank of Saurashtra with SBI was in 2008. State Bank of Indore merged to SBI in 2010.

41. Answer (b) Explanation With a view to improve the governance of Public Sector Banks (PSBs), the Government of India has set up an autonomous Banks Board Bureau(BBB). The Bureau will recommend for selection of heads - Public Sector Banks and Financial Institutions (even from the private sector, if need be) and help banks in developing strategies and capital raising plans. It will also advise banks how to go ahead with mergers and acquisitions. Banks Board Bureau has three ex-officio members and three expert members in addition to Chairman. All the Members and Chairman will be part time. The appointments have been made for a period of two years. BBB, which came to life in 2016, as part of the seven-point Indradhanush plan will constantly engage with the boards of all 22 public sector banks to formulate appropriate strategies for their growth and development. It would also be responsible for selection of non-executive chairman and non-official directors on the boards. Presently BBB is chaired by the former CAG Vinod Rai.

42. Answer (d) Explanation National Investment and Infrastructure Fund (NIIF) is a major initiative by the Government of India to give fillip to Infrastructure sector and spur economic growth. Its creation was announced in the Union Budget 2015-16. The objective of NIIF would be to maximize economic impact mainly through infrastructure development in commercially viable projects, both green field and brown field, including stalled projects. It could also consider other nationally important projects, for example, in manufacturing, if commercially viable. The Cabinet Committee on Economic Affairs (CCEA) on 27th January, 2005 had approved the constitution of a National Investment Fund (NIF). The Purpose of the fund was to receive disinvestment proceeds of central public sector enterprises and to invest the same to generate earnings without depleting the corpus. The earnings of the Fund were to be used for selected Central social welfare Schemes. This fund was kept outside the consolidated fund of India. Exchange Traded Funds (ETFs), as their name denotes, are mutual funds that are traded on stock exchanges and bought and sold like stocks. They mostly track an underlying index and are passive investment tools, that is, the fund manager has no role to play in ETFs. The expense ratio is, therefore, much lower in ETFs. On 14 March 2017, the Government of India launched the third tranche of Central Public Sector Enterprises Exchange Traded Fund (CPSE-ETF) worth ` 2,500 crore. It was operated by Reliance Mutual Fund. The offering was subscribed nearly four times. CPSE-ETF, which tracks the Nifty CPSE Index, is a concentrated portfolio of 10 stocks (stocks of both Navaratna and Maharatna companies), whose main aim is to help the Government of India (GOI) in disinvesting its stake in a few CPSEs via the ETF route. There is an upfront discount of 5 per cent

on the Further Fund Offer (FFO) Reference Market Price to the stocks included in the index.

43. Answer (c) Explanation The eligibility criteria laid down by the Government for grant of Maharatna, Navratna and Miniratna status to Central Public Sector Enterprises (CPSEs) are following: Criteria for grant of Maharatna status:- The CPSEs fulfilling the following criteria are eligible to be considered for grant of Maharatna status. (i) Having Navratna status. (ii) Listed on Indian stock exchange with minimum prescribed public shareholding under SEBI regulations. (iii) Average annual turnover of more than Rs. 25,000 crore, during the last 3 years. (iv) Average annual net worth of more than Rs. 15,000 crore, during the last 3 years. (v) Average annual net profit after tax of more than Rs. 5,000 crore, during the last 3 years. (vi) Should have significant global presence/international operations. Criteria for grant of Navratna status:- The Miniratna Category – I and Schedule ‗A‘ CPSEs, which have obtained ‗excellent‘ or ‗very good‘ rating under the Memorandum of Understanding system in three of the last five years, and have composite score of 60 or above in the six selected performance parameters, namely, (i) net profit to net worth, (ii) manpower cost to total cost of production/services,

Page 29: PRELIMS MODEL TEST-11 ANSWER KEY AND EXPLANATION. PMT-11 to 15 Explanation.pdfTARGET PRELIMS 2017 GENERAL STUDIES PAPER - 1 PRELIMS MODEL TEST-11 ANSWER KEY AND EXPLANATION 1. Answer

NEO IAS 0484-3190310, 9446331522, 9446334122 Page 29 www.neoias.com | www.youtube.com/neoias | www.facebook.com/neoias | www.twitter.com/neoias

(iii) profit before depreciation, interest and taxes to capital employed, (iv) profit before interest and taxes to turnover, (v) earning per share and (vi) inter-sectoral performance. Criteria for grant of Miniratna status:- The CPSEs which have made profits in the last three years continuously and have positive net worth are eligible to be considered for grant of Miniratna status. Presently, there are 7 Maharatna, 16 Navratna and 71 Miniratna CPSEs. 7 Maharatna companies are: Steel Authority of India Ltd. Oil and Natural Gas Corporation National Thermal Power Corporation Indian Oil Corporation Ltd. Coal India Ltd. Bharat Heavy Electricals Ltd. Gas Authority of India Ltd.

44. Answer (c) Explanation In the first monetary policy of new fiscal year 2017-18, the Monetary Policy Committee of RBI has reduced the gap of repo and reverse repo to 25 bps from existing 50 bps. The MSF rate is fixed 25 bps above the repo rate.

45. Answer (c) Explanation Goods and Services Tax Network (GSTN) is a Section 8 (under new companies Act, not for profit companies are governed under section 8), non-Government, private limited company. It was incorporated on March 28, 2013. The Government of India holds 24.5% equity in GSTN and all States of the Indian Union, including NCT of Delhi and Puducherry, and the Empowered Committee of State Finance Ministers (EC), together hold another 24.5%. Balance 51% equity is with non-Government financial institutions. The Company has been set up primarily to provide IT infrastructure and services to the Central and State Governments, tax payers and other stakeholders for implementation of the Goods and Services Tax (GST). The Authorised Capital of the company is ₹10, 00, 00,000 (Rupees ten crore only). The present equity composition of GSTN is given below:

46. Answer (a) Explanation ‗NASDAQ‘ is a global electronic marketplace for buying and selling securities, as well as the benchmark index for U.S. technology stocks. Nasdaq was created by the National Association of Securities Dealers (NASD) to enable investors to trade securities on a computerized, speedy and transparent system, and commenced operations on February 8, 1971. The term ―Nasdaq‖ is also used to refer to the Nasdaq Composite, an index of more than 3,000 stocks listed on the Nasdaq exchange that includes the world‘s foremost technology and biotech giants such as Apple, Google, Microsoft, Oracle, Amazon, Intel and Amgen.

47. Answer (a) Explanation Goods and Services Tax Network (GSTN) is a Section 8 (under new companies Act, not for profit companies are governed under section 8), non-Government, private limited company. It was incorporated on March 28, 2013. The Government of India holds 24.5% equity in GSTN and all States of the Indian Union, including NCT of Delhi and Puducherry, and the Empowered Committee of State Finance Ministers (EC), together hold another 24.5%. Balance 51% equity is with non-Government financial institutions. The Company has been set

Page 30: PRELIMS MODEL TEST-11 ANSWER KEY AND EXPLANATION. PMT-11 to 15 Explanation.pdfTARGET PRELIMS 2017 GENERAL STUDIES PAPER - 1 PRELIMS MODEL TEST-11 ANSWER KEY AND EXPLANATION 1. Answer

NEO IAS 0484-3190310, 9446331522, 9446334122 Page 30 www.neoias.com | www.youtube.com/neoias | www.facebook.com/neoias | www.twitter.com/neoias

up primarily to provide IT infrastructure and services to the Central and State Governments, tax payers and other stakeholders for implementation of the Goods and Services Tax (GST). Several measures of strategic control of Government over GSTN have been envisaged. These are explained below: 1. Strategic control through Board of Directors (BOD):

The Articles of Association (AOA) of GSTN provide that matters of strategic importance will be decided by the Board of Directors and that the Chairman of the Board will have casting/ second vote where Directors are equally divided over any issue in Board meeting.

2. Strategic Control through Special Resolution: The AOA of GSTN provides that certain matters of strategic importance shall be decided only through Special Resolution (i.e. three fourth (3/4) of the shareholders voting must vote in favour of such matters). Government's 49% shareholding will ensure that it retains effective control over such matters.

3. Strategic control through Shareholders Agreement: An agreement amongst all shareholders of GSTN SPV may provide that till the time Government holds certain threshold of shares in GSTN SPV, specific matters of strategic importance shall not be decided upon without the affirmative vote of the Government.

4. Placement of personnel on deputation in the GSTN SPV: Strategic Control is being ensured also through deputation of Government officials in the GSTN, at both leadership as well as operational levels. Services Division of GSTN, which is responsible for defining business processes, approving the modules and monitoring the outcomes is managed primarily by Government officers drawn from Central and State Tax Departments. GSTN has two officers of CBEC, eight officers from State Commercial Tax Departments, one officer of Indian Audit and Accounts Service and one officer from central government working on deputation. Some more officers are likely to join in future.

5. Agreements between Government and GSTN SPV: Control over strategic matters could be exercised by Government by incorporating suitable provisions in the Agreement governing service delivery to be executed between Government and GSTN.

In order to reinforce these measures further, following provisions have been included in the AOA of GSTN: 1. The Chairman of the Company will be nominated through a joint approval mechanism of Central and State

Governments. 2. Out of 14 Directors envisaged in the AOA, seven (including the Chairman) are nominees of the Central and

State Governments whereas private share-holders having 51% equity can nominate only 3 Directors. 3. The quorum for a meeting of the Board is four (4) Directors, out of which one Director must be the nominee

of the Central Government, one Director the nominee of State Governments and one Director should be other than nominees of Central and State Governments. Further, the quorum for a meeting of the Board is not complete unless fifty per cent (50%) of the Directors present in any meeting of the Board are nominees of Central and State Governments.

With above mentioned structuring, Government exercises an effective strategic control over decision-making in the GSTN Board, in which all powers of the Company are vested.

48. Answer (d) Explanation India is planning to introduce a new monetary-policy tool in the financial year 2017-18 to better manage a banking system swimming in excess cash. The so-called Standing Deposit Facility, or SDF, will help the Reserve Bank of India absorb surplus funds without having to provide lenders collateral in exchange. In 2014, a panel led by RBI Governor Urjit Patel -- who was at that time a deputy -- had proposed the introduction of the SDF as part of measures to improve the monetary-policy framework.

49. Answer (c) Explanation The India-Singapore DTAA at present provides for residence based taxation of capital gains of shares in a company. The Third Protocol amends the DTAA with effect from 1st April, 2017 to provide for source based taxation of capital gains arising on transfer of shares in a company. This will curb revenue loss, prevent double non-taxation and streamline the flow of investments. In order to provide certainty to investors, investments in shares made before 1st April, 2017 have been grandfathered subject to fulfilment of conditions in Limitation of Benefits clause as per 2005 Protocol. Further, a two year transition period from 1st April, 2017 to 31st March, 2019 has been provided during which capital gains on shares will be taxed in source country at half of normal tax rate, subject to fulfilment of conditions in Limitation of Benefits clause. A derivative is a contract between two parties which derives its value/price from an underlying asset. The most common types of derivatives are futures, options, forwards and swaps. Derivatives traded in India remain exempt from the new capital gains tax under the revised treaty.

50. Answer (b) Explanation PM Garib Kalyan Yojna is a voluntary income declaration scheme under which those with unaccounted income were allowed to come clean by paying 50% of their untaxed wealth. During the second window of PMGKY, popularly known as Income tax Declaration Scheme-II that did wind up on 31

st March 2017, the govt. could mop

up an income of ₹4600 Crore, which is only about 10 per cent of the previous income declaration scheme.

Page 31: PRELIMS MODEL TEST-11 ANSWER KEY AND EXPLANATION. PMT-11 to 15 Explanation.pdfTARGET PRELIMS 2017 GENERAL STUDIES PAPER - 1 PRELIMS MODEL TEST-11 ANSWER KEY AND EXPLANATION 1. Answer

NEO IAS 0484-3190310, 9446331522, 9446334122 Page 31 www.neoias.com | www.youtube.com/neoias | www.facebook.com/neoias | www.twitter.com/neoias

Central Board of Direct Tax is the agency responsible for the implementation of the scheme.

51. Answer (a) Explanation SGBs are government securities denominated in grams of gold. They are substitutes for holding physical gold. Investors have to pay the issue price in cash and the bonds will be redeemed in cash on maturity. The Bond is issued by Reserve Bank on behalf of Government of India. The Bonds bear interest at the rate of 2.75 per cent (fixed rate) per annum on the amount of initial investment. There is no bar on investment by banks in Sovereign Gold Bonds as they will qualify for SLR. The

bonds can also be traded and they can be used as collateral for loans.

52. Answer (d) Explanation Small saving schemes in India are one of the country‘s most viable financial options. Their main aim is to provide financial help to the economically underprivileged section of the Indian population, a group that makes up a huge portion of the country‘s population. They become instrumental for generation of money to be spent on development. The providers of small savings schemes want to achieve this by providing the common people with investment options that offer high returns without high risk. The major savings scheme options are

Sukanya Samridhi Yojna

Kisan Vikas Patra

National Savings Certificate

Public Provident Fund

Senior Citizens Savings Scheme etc.

53. Answer (c) Explanation An anti-dumping duty is a protectionist tariff that a domestic government imposes on foreign imports that it believes are priced below fair market value. Dumping is a process where a company exports a product at a price lower than the price it normally charges on its own home market. To protect local businesses and markets, many countries impose stiff duties on products they believe are being dumped in their national market. In cases foreign producers attempt to subsidize the goods being exported by them so that it causes domestic production to suffer because of a shift in domestic demand towards cheaper imported goods, the government makes mandatory the payment of a countervailing duty on the import of such goods to the domestic economy.

54. Answer (d) Explanation WTO members' Mode 4 commitments have been undertaken on a horizontal basis, i.e. applicable without distinctions to all sectors inscribed in a member's schedule. In most instances, members have scheduled an initial ―unbound‖ (i.e. no binding of access conditions) and then qualified it by granting admission to selected categories of persons, with a marked bias towards persons linked to a commercial presence (e.g. intra-corporate transferees) and highly skilled persons (managers, executives and specialists). In addition to limiting access to certain categories of persons, other restrictions frequently inscribed in schedules include: defined duration of stay; quotas, including on the number or proportion of foreigners employed; ―economic needs tests‖ (a test that conditions market access upon the fulfilment of certain economic criteria) or ―labour market tests‖, generally inscribed without any indication of the criteria of application; pre-employment conditions; residency and training requirements. Recently Singapore has been denying visas to Indian IT professionals citing the Economic Needs Test though India Singapore CECA clearly says that that there will be no ENT or quotas on agreed services.

55. Answer (a) Explanation National Investment and Infrastructure Fund (NIIF) is a major initiative by the Government of India to give fillip to Infrastructure sector and spur economic growth. Its creation was announced in the Union Budget 2015-16. The objective of NIIF would be to maximize economic impact mainly through infrastructure development in commercially viable projects, both greenfield and brownfield, including stalled projects. It could also consider other nationally important projects, for example, in manufacturing, if commercially viable. The proposed corpus of NIIF is ₹40,000 Crores (around USD 6 Billion). The initial authorized corpus of NIIF would be ₹20,000 crore, which may be raised from time to time, as decided by Ministry of Finance. Government can provide upto ₹20000 crore per annum into these funds. Government's contribution/share in the corpus will be 49% in each entity set up as an alternate Investment Fund (AIF) and will neither be increased beyond, nor allowed to fall below, 49%. The whole of 49% would be contributed by Government directly. Rest is open for contribution from others.

Page 32: PRELIMS MODEL TEST-11 ANSWER KEY AND EXPLANATION. PMT-11 to 15 Explanation.pdfTARGET PRELIMS 2017 GENERAL STUDIES PAPER - 1 PRELIMS MODEL TEST-11 ANSWER KEY AND EXPLANATION 1. Answer

NEO IAS 0484-3190310, 9446331522, 9446334122 Page 32 www.neoias.com | www.youtube.com/neoias | www.facebook.com/neoias | www.twitter.com/neoias

Governing Council of NIIF is an advisory body - chaired by the Finance Minister. It shall comprise of government representatives, eminent economists, professionals from the Infrastructure Sector and representatives from the non-government contributors. Government is in the process of setting up the two sub funds under the NIIF- one in clean energy fund and other in focus on highways projects.

56. Answer (a) Explanation As per Constitution (One Hundred and First Amendment) Act, 2016, the President of India has constituted the Goods and Services Tax Council. The Goods and Services Tax Council shall consist of the following members‘ viz. Union Finance Minister as the Chairperson, Union Minister of state in charge of Revenue as member and the minister in charge of finance or revenue or any other minister nominated by each state government as other members. The decisions of the GST Council are made by three-fourth majority of the votes cast. The centre has one-third of the votes cast, and the states together have two-third of the votes cast. Each state has one vote, irrespective of its size or population. Apart from the above:

The Secretary (Revenue) will be appointed as the Ex-officio Secretary to the GST Council.

The Chairperson, Central Board of Excise and Customs (CBEC), will be included as a permanent invitee (non-voting) to all proceedings of the GST Council.

One post of Additional Secretary to the GST Council in the GST Council Secretariat (at the level of Additional Secretary to the Government of India) will be created.

Four posts of Commissioner in the GST Council Secretariat (at the level of Joint Secretary to the Government of India) will also be created.

57. Answer (a) Explanation GST is one indirect tax for the whole nation, which will make India one unified common market. GST is a single tax on the supply of goods and services, right from the manufacturer to the consumer. Credits of input taxes paid at each stage will be available in the subsequent stage of value addition, which makes GST essentially a tax only on value addition at each stage. The final consumer will thus bear only the GST charged by the last dealer in the supply chain, with set-off benefits at all the previous stages. Since the share of GST is distributed to the state where goods or services are finally consumed, it is called as destination based tax.

58. Answer (d) Explanation GST is one indirect tax for the whole nation, which will make India one unified common market. GST is a single tax on the supply of goods and services, right from the manufacturer to the consumer. Assam is the first state to ratify the 122

nd constitutional amendment bill on GST on 12 August 2016.

France is the first country in the world to introduce the GST system in 1954.

59. Answer (b) Explanation Credit rating is an assessment of the creditworthiness of a borrower in general terms or with respect to a particular debt or financial obligation. A high credit rating indicates a high possibility of paying back the loan in its entirety without any issues; a poor credit rating suggests that the borrower has had trouble paying back loans in the past, and might follow the same pattern in the future. Credit ratings apply to businesses and government, while credit scores apply only to individuals. Similarly, sovereign credit ratings apply to national governments, and corporate credit ratings apply solely to corporations. The Credit ratings are mentioned as ‗AAA‘, ‗AA‘, ‗A‘, ‗BBB‘, ‗BB‘, ‗B‘, ‗C‘ and ‗D‘. These rating show the timely servicing of debt obligation. The highest rating ‗AAA‘ means the highest degree of safety and lowest credit risk. The Sovereign rating of India is ‗BBB-‘ which is just above the junk bonds.

60. Answer (a) Explanation The Economic Survey 2016-17, noted that India‘s ratings have remained stuck at the much lower level of BBB, despite the country‘s ―dramatic‖ improvement in growth and macro-economic stability since 2014. In November, 2016, the Standard & Poor ruled out the scope of a ratings upgrade for India, mainly on the grounds of its low per capita GDP and relatively high fiscal deficit. India has no history of default loans

61. Answer (c) Explanation A revised Agreement between India and Cyprus for the Avoidance of Double Taxation and the Prevention of Fiscal evasion (DTAA) with respect to taxes on income, along with its Protocol, will replace the existing DTAA that was

Page 33: PRELIMS MODEL TEST-11 ANSWER KEY AND EXPLANATION. PMT-11 to 15 Explanation.pdfTARGET PRELIMS 2017 GENERAL STUDIES PAPER - 1 PRELIMS MODEL TEST-11 ANSWER KEY AND EXPLANATION 1. Answer

NEO IAS 0484-3190310, 9446331522, 9446334122 Page 33 www.neoias.com | www.youtube.com/neoias | www.facebook.com/neoias | www.twitter.com/neoias

signed by two countries on 13th June 1994. This step follows the recent amendment of the Double Taxation Avoidance Agreement with Mauritius. As in the case of Mauritius, the treaty with Cyprus had provided for residence-based taxation of capital gains. With the revision of the treaty now approved by the Cabinet, capital gains will be taxed in India for entities resident in Cyprus, subject to double tax relief. In other words, India will have the right to tax capital gains arising in India. Though this will be effective prospectively (investments made before 1

st April 2017 are grandfathered), similar to the provision made in the India-Mauritius treaty, it does

not provide for a transitionary period like the one provided by the India-Mauritius treaty, wherein only half the capital gains tax rate will be applicable between 2017 and 2019(Limitation of Benefit Clause). The India Mauritius Treaty provisions are also applicable to the India Singapore amended DTAA.

62. Answer (a) Explanation GAAR seeks to give the tax department powers to scrutinize transactions structured in such a way as to deliberately avoid paying tax in India. After being postponed twice, it is slated to be implemented from financial year 2017-18.

63. Answer (d) Explanation GAAR seeks to give the tax department powers to scrutinize transactions structured in such a way as to deliberately avoid paying tax in India. There are some instances when the GAAR is not invoked: 1. It will not be invoked in cases where investments are routed through tax treaties that have a sufficient

limitation of benefit (LOB) clause to address tax avoidance. 2. Alltransactions or arrangements that have been approved by courts and quasi-judicial authorities like the

authority for advance ruling and that specifically address the issue of tax avoidance will not be subject to the GAAR test.

3. These include non-applicability of GAAR to transactions where tax benefit to parties does not exceed ₹3 crore in a financial year.

To prevent misuse of GAAR provisions by the tax department, adequate safeguards have also been put in place based on which GAAR will be invoked. The proposal to apply GAAR will be vetted first by an officer at the level of the principal commissioner or commissioner of income tax and at the second stage by an approving panel headed by a high court judge.

64. Answer (c) Explanation The index of Eight core industries having a combined weight of 37.91 per centin the Index of Industrial Production (IIP), published by CSO. The weightages of them are given below: 1. Electricity generation (weight: 10.32 %) 2. Steel production (weight: 6.68 %) 3. Petroleum refinery production (weight: 5.94 %) 4. Crude oil production (weight: 5.22 %) 5. Coal production (weight: 4.38 %) 6. Cement production (weight: 2.41 %) 7. Natural gas production (weight: 1.71 %) 8. Fertilizer production (weight: 1.25 %)

65. Answer (d) Explanation Against the backdrop of robust macro-economic stability, the year 2016was marked by two major domestic policy developments, the passage of the Constitutional amendment, paving the way for implementing the transformational Goods and Services Tax (GST), and the action to demonetise the two highest denomination notes. The GST will create a common Indian market, improve tax compliance and governance, and boost investment and growth; it is also a bold new experiment in the governance of India‘s cooperative federalism.

66. Answer (a) Explanation Demonetisation has the potential to generate long-term benefits in terms of reduced corruption, greater digitalization of the economy, increased flows of financial savings, and greater formalization of the economy, all of which could eventually lead to higher GDP growth, better tax compliance and greater tax revenues.

67. Answer (b) Explanation

Page 34: PRELIMS MODEL TEST-11 ANSWER KEY AND EXPLANATION. PMT-11 to 15 Explanation.pdfTARGET PRELIMS 2017 GENERAL STUDIES PAPER - 1 PRELIMS MODEL TEST-11 ANSWER KEY AND EXPLANATION 1. Answer

NEO IAS 0484-3190310, 9446331522, 9446334122 Page 34 www.neoias.com | www.youtube.com/neoias | www.facebook.com/neoias | www.twitter.com/neoias

68. Answer (b) Explanation Demographic dividend is the ratio of the working age to non-working age population.

69. Answer (c) Explanation India became the first nation to tax digital transactions when it introduced the equalisation levy of 6%— also known as the Google tax — on the advertising revenue of multinational web companies in June 2016. According to the Budget announcement, any person or entity that makes a payment exceeding Rs. 1 lakh in a financial year to a non-resident technology company will now need to withhold 6% tax on the gross amount being paid as an equalisation levy. This tax, however, is only applicable when the payment has been made to avail certain Business to Business (B2B) services cover online advertisements, provision for digital advertising space or any other facility or service for the purpose of online advertisements. This list will expand soon. The idea of an equalization levy comes from Action 1 of the Organisation for Economic Cooperation and Development‘s (OECD‘s) base erosion and profit shifting (BEPS) Action Plan. The Action plan considers equalisation levy as

an option to tax digital transactions.

70. Answer (c) Explanation The Committee on Digital Payments constituted by the Ministry of Finance, Department of Economic Affairs under the Chairmanship of Shri. Ratan P. Watal, Principal Advisor, NITI Aayog and former Finance Secretary to the Government of India. In its Report, the Committee has recommended that a medium term strategy for accelerating growth of Digital Payments in India with a regulatory regime which is conducive to bridging the Digital divide by promoting competition, open access & interoperability in payments. The Report recommends inclusion of financially and socially excluded groups and assimilation of emerging technologies in the market, while safeguarding security of Digital Transactions and providing level playing to all stakeholders and new players who will enter this new transaction space. It has suggested inter-operability of the payments system between banks and non-banks, up-gradation of the digital payment infrastructure and institutions and a

framework to reward innovations and for leading efforts in enabling digital payments.

71. Answer (b) Explanation Australia is the first country to introduce the polymer bank notes during 1988 in coinciding with its bicentennial year. In 1996, it completely switched to polymer bank notes.

72. Answer (d) Explanation Over the past three years the RBI has implemented a number of schemes to facilitate resolution of the stressed asset problem. The 5/25 Refinancing of Infrastructure Scheme: This scheme offered a larger window for revival of stressed assets in the infrastructure sectors and eight core industry sectors. Under this scheme lenders were allowed to extend amortisation periods to 25 years with interest rates adjusted every 5 years, so as to match the funding period with the long gestation and productive life of these projects. The scheme thus aimed to improve the credit profile and liquidity position of borrowers, while allowing banks to treat these loans as standard in their balance sheets, reducing provisioning costs. However, with amortisation spread out over a longer period, this arrangement also meant that the companies faced a higher interest burden, which they found difficult to repay, forcing banks to extend additional loans (‗ever greening‘). This in turn has aggravated the initial problem. Private Asset Reconstruction Companies (ARCs): ARCs were introduced to India under the SARFAESI Act (2002), with the notion that as specialists in the task of resolving problem loans, they could relieve banks of this burden. However, ARCs have found it difficult to resolve the assets they have purchased, so they are only willing to purchase loans at low prices. Asa result, banks have

Page 35: PRELIMS MODEL TEST-11 ANSWER KEY AND EXPLANATION. PMT-11 to 15 Explanation.pdfTARGET PRELIMS 2017 GENERAL STUDIES PAPER - 1 PRELIMS MODEL TEST-11 ANSWER KEY AND EXPLANATION 1. Answer

NEO IAS 0484-3190310, 9446331522, 9446334122 Page 35 www.neoias.com | www.youtube.com/neoias | www.facebook.com/neoias | www.twitter.com/neoias

been unwilling to sell them loans on a large scale. Then, in 2014 the fee structure of the ARCs was modified, requiring ARCs to pay a greater proportion of the purchase price up-front in cash. Since then, sales have slowed to a trickle: only about 5 percent of total NPAs at book value were sold over 2014-15 and 2015-16. Strategic Debt Restructuring (SDR): The RBI came up with the SDR scheme in June 2015 to provide an opportunity to banks to convert debt of companies (whose stressed assets were restructured but which could not finally fulfil the conditions attached to such restructuring) to 51 per cent equity and sell them to the highest bidders, subject to authorization by existing shareholders. An 18-month period was envisaged for these transactions, during which the loans could be classified as performing. But as of end-December 2016, only two sales had materialized, in part because many firm‘s remained financially unviable, since only a small portion of their debt had been converted to equity. Asset Quality Review (AQR): Resolution of the problem of bad assets requires sound recognition of such assets. Therefore, the RBI emphasized AQR, to verify that banks were assessing loans in line with RBI loan classification rules. Any deviations from such rules were to be rectified by March 2016. Sustainable Structuring of Stressed Assets (S4A): Under this arrangement, introduced in June 2016, an independent agency hired by the banks will decide on how much of the stressed debt of a company is ‗sustainable‘. The rest (‗unsustainable‘) will be converted into equity and preference shares. Unlike the SDR arrangement, this involves no change in the ownership of the company.

73. Answer (b) Explanation Department of Empowerment of Persons with Disabilities (DEPwD), Ministry of Social Justice and Empowerment, has formulated the Accessible India Campaign (Sugamya Bharat Abhiyan), as a nation-wide campaign for achieving universal accessibility for PwDs. The campaign targets three separate verticals for achieving universal accessibility namely the built up environment, transportation eco-system and information & communication eco-system. The campaign has ambitious targets with defined timelines and will use IT and social media for spreading awareness about the campaign and seeking commitment / engagement of various stakeholders. The Department has asked various State Govts. to identify about 50 to 100 public buildings in big cities and also identify citizen centric public websites, which if made fully accessible would have the highest impact on the lives of PwDs.

74. Answer (c) Explanation MCLR is comprised of the following components 1. Marginal cost of funds 2. Negative carry on account of CRR 3. Operating cost 4. Tenor premium

75. Answer (b) Explanation A shadow banking system refers to the financial intermediaries involved in facilitating the creation of credit across the global financial system but whose members are not subject to regulatory oversight. The shadow banking system also refers to unregulated activities by regulated institutions. Examples of intermediaries not subject to regulation include hedge funds, unlisted derivatives and other unlisted instruments, while examples of unregulated activities by regulated institutions include credit default swaps. The shadow banking system has escaped regulation primarily because it does not accept traditional bank deposits. As a result, many of the institutions and instruments have been able to employ higher market, credit and liquidity risks, and do not have capital requirements commensurate with those risks. Subsequent to the subprime meltdown in 2008, the activities of the shadow banking system came under increasing scrutiny and regulations.

76. Answer (c) Explanation Bioequivalence studies are tests conducted on humans to evaluate any drugs dissolution and efficacy levels in blood. If two products are bioequivalent, it means they are same for all intents and purposes including quality and efficacy.

77. Answer (c) Explanation The Prime Minister launched a nation-wide ―Soil Health Card‖ (SHC) scheme in early 2015 to rejuvenate India‘s exhausted soil. Using a grid-wise approach, representative soil samples from the fields are tested for nutrient content in designated chemical laboratories. Accordingly, macro and micro nutrients needed by the soil are identified and translated into specific, measured quantities of fertilisers required. This information, printed on the SHC, is made available to the farmers in that grid through the state agricultural departments. Thirty million SHCs were issued in 2015-16 and the Ministry of Agriculture aims to cover the entire farming population by 2018-19. In addition, on a pilot basis, the soil health information is made available at fertiliser purchase points —Primary Agricultural Credit Societies (PACS) and POS devices-enabled fertiliser retail shops.

Page 36: PRELIMS MODEL TEST-11 ANSWER KEY AND EXPLANATION. PMT-11 to 15 Explanation.pdfTARGET PRELIMS 2017 GENERAL STUDIES PAPER - 1 PRELIMS MODEL TEST-11 ANSWER KEY AND EXPLANATION 1. Answer

NEO IAS 0484-3190310, 9446331522, 9446334122 Page 36 www.neoias.com | www.youtube.com/neoias | www.facebook.com/neoias | www.twitter.com/neoias

78. Answer (c) Explanation The National Gokul Mission was launched for the preservation and promotion of indigenous breeds of cows under National Programme for Bovine Breeding and Dairy Development for the first time in the country. Establishment of Integrated Indigenous Cattle Centres viz ―Gokul Gram‖ is a sub component of it. Under this component it is proposed to establish Integrated Indigenous Cattle Centres or Gokul Grams in the breeding tracts of indigenous breeds. Gokul Grams will be established in: i) the native breeding tracts and ii) near metropolitan cities for housing the urban cattle. Gokul Gram will act as Centres for development of Indigenous Breeds and a dependable source for supply of high genetic breeding stock to the farmers in the breeding tract. The Gokul Gram will be self-sustaining and will generate economic resources from sale of milk, organic manure, vermi-composting, urine distillates, and production of electricity from bio gas for in house consumption and sale of animal products.

79. Answer (d) Explanation Sugarcane is a Kharif crop. It is the main source of sugar, gur and khandsari. It also provides raw material for the manufacturing of alcohol. Bagasse, the crushed cane residue, has also multiple uses. It is used for manufacturing of paper. It is also an efficient substitute for petroleum products and a host of other chemical products. A part of it is also used as fodder.

80. Answer (d) Explanation The border method of irrigation makes use of parallel ridges to guide a sheet of flowing water as it moves down the slope. If the borders are constructed along the general slope of the field, they are known as straight borders or slope borders and if they are constructed across the general slope of the field they are called contour borders.

81. Answer (b) Explanation Tueboponics is a fairly recent development in the field of irrigation. This technique has been developed in Israel and practiced in the desert areas of that country in order to convert it into green lush forests. In tueboponics, water is provided to plants/trees through injections. The needle is inserted into the plant stem and water is delivered into the phloem. The needle and injection used for this purpose are of special shape and design prepared only for this purpose. In India, this technique has not yet arrived but it may prove very useful in Indian deserts where water is very scarce. Its use in crop plants seems to be impracticable. It is a very simple device.

82. Answer (c) Explanation Irrigation comes under Ministry of Water Resources, River Development & Ganga Rejuvenation. From time to time, Government introduces several water resources development programmes to increase cropped area under irrigation and significantly enhance water use efficiency. Command Area Development Program [CADP] in 1974-75, to bridge the gap between irrigation potential created and its actual utilization by introducing suitable cropping pattern, strengthening research, extension and training facilities, organizing field demonstrations and supplying inputs. Rural Infrastructure Development Fund [RIDF] in 1995-96 to complete hitherto incomplete irrigation projects mobilizing deposits out of the shortfall in commercial banks‘ lending targets to agriculture to complete irrigation development.

83. Answer (b) Explanation In a move that aims to address the perennial irrigation water crisis affecting rural India, Ministry of Water Resources, River Development and Ganga Rejuvenation and NABARD signed an agreement to operationalise the Long Term Irrigation Fund (LTIF) to be instituted in NABARD as part of Pradhan Mantri Krishi Sinchayee Yojana (PMKSY). The Fund, with an initial corpus of about Rs 20,000 crore, was announced in the Union Budget 2016-17. Corpus would be raised by way of budgetary resources and market borrowings to fund fast tracking of implementation of incomplete major & medium irrigation projects.

84. Answer (d) Explanation A village, wherein trained group of farmers are involved in production of seeds of various crops and cater to the needs of themselves, fellow farmers of the village and farmers of neighboring villages in appropriate time and at affordable cost is called "a seed village". CONCEPT

Organizing seed production in cluster (or) compact area

Page 37: PRELIMS MODEL TEST-11 ANSWER KEY AND EXPLANATION. PMT-11 to 15 Explanation.pdfTARGET PRELIMS 2017 GENERAL STUDIES PAPER - 1 PRELIMS MODEL TEST-11 ANSWER KEY AND EXPLANATION 1. Answer

NEO IAS 0484-3190310, 9446331522, 9446334122 Page 37 www.neoias.com | www.youtube.com/neoias | www.facebook.com/neoias | www.twitter.com/neoias

Replacing existing local varieties with new high yielding varieties.

Increasing the seed production

To meet the local demand, timely supply and reasonable cost

Self sufficiency and self reliance of the village

Increasing the seed replacement rate

85. Answer (c) Explanation With NAM in place, farmers will have access to many more buyers, including from outside a State. Thus, a farmer can choose from where and to whom he wants to sell his produce. Similarly, buyers will have more freedom to choose from where they want to buy. With middlemen mostly out of the scene, NAM is expected to create a Pan-India common market with freer inter-State movement of agri commodities. Real-time monitoring of prices and transparency in operations, such as weighing, pricing, billing and presence of a competitive marketing platform, will lead to better price discovery. It will reduce the gap between farm to fork prices (the prices farmers get and prices paid by retail consumers).It reduces the number of intermediaries and improves the scope for more income for farmers. Bulk buyers of agri commodities such as the Food Corporation of India may be able to conduct e-procurement operations across regions, at transparent prices.

86. Answer (d) Explanation There will be a uniform premium of only 2% to be paid by farmers for all Kharif crops and 1.5% for all Rabi crops. In case of annual commercial and horticultural crops, the premium to be paid by farmers will be only 5%. The premium rates to be paid by farmers are very low and balance premium will be paid by the Government to provide full insured amount to the farmers against crop loss on account of natural calamities. There is no upper limit on Government subsidy. The use of technology will be encouraged to a great extent. Smart phones will be used to capture and upload data of crop cutting to reduce the delays in claim payment to farmers. Remote sensing will be used to reduce the number of crop cutting experiments.

87. Answer (c) Explanation The National Development Council (NDC) in its 53rd meeting held on 29th May, 2007 adopted a resolution to launch a Food Security Mission comprising rice, wheat and pulses to increase the production of rice by 10 million tons, wheat by 8 million tons and pulses by 2 million tons by the end of the Eleventh Plan (2011-12). Accordingly, a Centrally Sponsored Scheme, 'National Food Security Mission' (NFSM), was launched in October 2007. The Mission is being continued during 12th Five Year Plan with new targets of additional production of food grains of 25 million tons of food grains comprising of 10 million tons rice, 8 million tons of wheat, 4 million tons of pulses and 3 million tons of coarse cereals by the end of 12th Five Year Plan. The National Food Security Mission (NFSM) during the 12th Five Year Plan will have five components (i) NFSM- Rice; (ii) NFSM-Wheat; (iii) NFSM-Pulses, (iv)NFSM-Coarse cereals/Millets and (v) NFSM-Commercial Crops.

88. Answer (b) Explanation Minimum Support Price (MSP) is a form of market intervention by the Government of India to insure agricultural producers growing major agriculture commodities (24 commodities) against any sharp fall in farm prices during bumper production years. The major objectives are to support the farmers from distress sales and to procure food grains for public distribution. MSPs have also been used as a tool by the Government to incentivise farmers to grow crops that are in short supply.

89. Answer (d) Explanation

Cropping season Major crops cultivated

Kharif June-September

Rice, Cotton, Bajra, Maize, Jowar, Tur

Rabi October-March

Wheat, Gram, Rapeseeds and Mustard, Barley

Zaid April-June

Vegetables, Fruits, Fodder

90. Answer (a)

Explanation The minimum support prices are announced by the Government of India at the beginning of the sowing season for certain crops on the basis of the recommendations of the Commission for Agricultural Costs and Prices(CACP). It is approved by Cabinet Committee on Economic affairs.

91. Answer (c)

Page 38: PRELIMS MODEL TEST-11 ANSWER KEY AND EXPLANATION. PMT-11 to 15 Explanation.pdfTARGET PRELIMS 2017 GENERAL STUDIES PAPER - 1 PRELIMS MODEL TEST-11 ANSWER KEY AND EXPLANATION 1. Answer

NEO IAS 0484-3190310, 9446331522, 9446334122 Page 38 www.neoias.com | www.youtube.com/neoias | www.facebook.com/neoias | www.twitter.com/neoias

Explanation In simple terms, Public distribution system (PDS) is a government-sponsored chain of shops entrusted with the work of distributing basic food and non-food commodities to the needy sections of the society at very cheap prices. The Central Government, through FCI, has assumed the responsibility for procurement, storage, transportation and bulk allocation of food grains to the State Governments. On an average, about 15-16 million tonnes of foodgrains are issued by the FCI to the States at a uniform Central Issue Price (CIP) which is much less than the economic cost incurred by the Central Government by way of procurement, storage, transport and distribution.

92. Answer (a) Explanation Micro Irrigation is slow application of water to plants at frequent times with low volume of water mainly under low pressure conditions. It increases water-use efficiency. Drip and sprinkler irrigation systems are two major micro-irrigation methods.

93. Answer (d) Explanation A ‗wilful default‘ would be deemed to have occurred if any of the following events is noted: a. The unit has defaulted in meeting its payment / repayment obligations to the lender even when it has the

capacity to honour the said obligations. b. The unit has defaulted in meeting its payment / repayment obligations to the lender and has not utilised the

finance from the lender for the specific purposes for which finance was availed of but has diverted the funds for other purposes.

c. The unit has defaulted in meeting its payment / repayment obligations to the lender and has siphoned off the funds so that the funds have not been utilised for the specific purpose for which finance was availed of, nor are the funds available with the unit in the form of other assets.

The unit has defaulted in meeting its payment / repayment obligations to the lender and has also disposed off or removed the movable fixed assets or immovable property given for the purpose of securing a term loan without the knowledge of the bank / lender.

94. Answer (d) Explanation Recently ‗Chettikulangara temple offerings‘ in Kerala has given the GI tag to preserve the devotional and traditional practices of the temple. Peruvian Pisco is the first foreign made product accorded the GI tag in India.

95. Answer (b) Explanation The objectives of setting up of Small Finance Banks will be to further financial inclusion by (a) provision of savings vehicles, and (b) supply of credit to small business units; small and marginal farmers; micro and small industries; and other unorganised sector entities, through high technology-low cost operations. Resident individuals/professionals with 10 years of experience in banking and finance; and companies and societies owned and controlled by residents will be eligible to set up small finance banks. Existing Non-Banking Finance Companies (NBFCs), Micro Finance Institutions (MFIs), and Local Area Banks (LABs) that are owned and controlled by residents can also opt for conversion into small finance banks. The Small Finance Bank shall primarily undertake basic banking activities of acceptance of deposits and lending to unserved and underserved sections including small business units, small and marginal farmers, micro and small industries and unorganised sector entities. The Small Finance Banks will be required to extend 75 per cent of its Adjusted Net Bank Credit (ANBC) to the sectors eligible for classification as priority sector lending (PSL) by the Reserve Bank.

96. Answer (c) Explanation ‗Place of Effective Management‘ (PoEM) is an internationally recognised test for determination of residence of a company incorporated in a foreign jurisdiction. Most of the tax treaties entered into by India recognises the concept of the place of effective management for determination of residence of a company as a tie-breaker rule for the avoidance of double taxation. Recently Government of India issued clarifying rules on determining the companies‘ place of effective management (POEM), in order to target the shell companies that are created to evade taxes through establishing overseas subsidiaries. However, it excluded from its purview firms with an annual turnover of less than ₹50 Crore and all the businesses with active business operations overseas. Thus, effectively, Indian companies with genuine business operations abroad and the MNC‘s with their units in India are not covered under the POEM guidelines.

97. Answer (a) Explanation

Page 39: PRELIMS MODEL TEST-11 ANSWER KEY AND EXPLANATION. PMT-11 to 15 Explanation.pdfTARGET PRELIMS 2017 GENERAL STUDIES PAPER - 1 PRELIMS MODEL TEST-11 ANSWER KEY AND EXPLANATION 1. Answer

NEO IAS 0484-3190310, 9446331522, 9446334122 Page 39 www.neoias.com | www.youtube.com/neoias | www.facebook.com/neoias | www.twitter.com/neoias

‗Multilateral Instrument‘ (MLI)is an agreement put out by OECD, the intergovernmental economic organisation, to stop Base Erosion and Profit Shifting (BEPS), a practice referring to tax avoidance strategies that exploit gaps and mismatches in tax rules to artificially shift profits to low or no-tax locations. MLI is thus a framework through which over 100 countries and jurisdictions are collaborating to tackle BEPS or anti-treaty shopping provisions. Tax experts say under MLI, foreign investors would have to pass a ‗principal purpose test‘ to satisfy all countries that tax avoidance was not the chief purpose. What complicates matters is that participating countries have a right to specify, at MLI‘s ratification, as to which provisions of the MLI they would opt in to and out of, leaving the field open for different interpretations.

98. Answer (c) Explanation Under a share buyback, a company repurchases its own shares from its shareholders (in this case, the promoter or government and others) using its cash reserves. The repurchased shares are killed off; as a result, the company‘s equity base shrinks. Half of the divestment proceeds in financial year 2016-17 have come from share buybacks. Financial year 2016-17 is set to go down in history as a record year for government disinvestment in public sector undertakings (PSUs). With nearly ₹37,200 Crore raised so far, the proceeds have already passed the previous best of ₹24,348 Crore for 2014-15. Market players say buybacks and bailouts defeat the basic principle of disinvestments, which is to "promote peoples ownership" in PSUs.

99. Answer (a) Explanation Annual Survey of Industries (ASI) is the main survey conducted by Central Statistics Office (CSO) Industrial Statistics (IS) wing. ASI is the principal source of industrial statistics in India. ASI is more comprehensive than Index of Industrial Production (IIP) to measure the industrial growth as the former includes small and medium industries.

100. Answer (d) Explanation The states having the special category status are Jammu and Kashmir, Uttarakhand, Himachal Pradesh, Sikkim, Assam, Arunachal Pradesh, Manipur, Nagaland, Tripura, Meghalaya and Mizoram. The criteria for granting special status were 1) a lot of hilly terrain 2) economic and social backwardness and lack of infrastructure 3) a large tribal population 4) international borders 5) non-viable nature of finances.

Page 40: PRELIMS MODEL TEST-11 ANSWER KEY AND EXPLANATION. PMT-11 to 15 Explanation.pdfTARGET PRELIMS 2017 GENERAL STUDIES PAPER - 1 PRELIMS MODEL TEST-11 ANSWER KEY AND EXPLANATION 1. Answer

NEO IAS 0484-3190310, 9446331522, 9446334122 Page 40 www.neoias.com | www.youtube.com/neoias | www.facebook.com/neoias | www.twitter.com/neoias

TARGET PRELIMS 2017 GENERAL STUDIES

PAPER - 1

PRELIMS MODEL TEST-13

ANSWER KEY AND EXPLANATION

1. Answer (b) Explanation The Council of Ministers is responsible to the Lok Sabha and not to the Rajya Sabha. It is explicitly stated in Article 75(3). As a result of it, Rajya Sabha also has no power to pass a no-confidence motion against Council of Ministers. Elected members are required to provide details of their asset and liabilities, only nominated members are exempted from it.

2. Answer (b) Explanation ―Judiciary, which is one of the three important pillars of our democracy, is the final interpreter of the Constitution and laws. It helps maintaining the social order by swiftly and effectively dealing with those on the wrong side of the law. As an upholder of the Rule of Law and enforcer of the right to liberty, the role of the Judiciary is sacrosanct.‖ (From President‘s speech)

3. Answer (d) Explanation The Swachh Bharat Abhiyan is a massive movement that seeks to create a Clean India. It ensures clean environment which is implied under the right to life, a fundamental right. Similarly, Article 47, directs the state to improve public health and raise the standard of living. It is related with the denouncing practices derogatory to the dignity of women and improving the environment, which is a fundamental duty.

4. Answer (b) Explanation The Sixth Schedule under Article 244(2) of the Constitution relates to those areas in the States of Assam, Meghalaya, Tripura and Mizoram which are declared as ‗Tribal Areas‘.

1 B 11 C 21 D 31 C 41 B 51 D 61 A 71 B 81 B 91 C

2 B 12 B 22 A 32 A 42 C 52 B 62 D 72 A 82 A 92 A

3 D 13 D 23 B 33 D 43 B 53 B 63 A 73 B 83 C 93 B

4 B 14 B 24 D 34 D 44 D 54 A 64 C 74 C 84 D 94 D

5 B 15 A 25 B 35 B 45 A 55 B 65 A 75 C 85 C 95 B

6 A 16 C 26 B 36 B 46 C 56 B 66 C 76 A 86 B 96 D

7 C 17 A 27 A 37 C 47 B 57 C 67 B 77 A 87 A 97 B

8 D 18 B 28 A 38 C 48 D 58 B 68 A 78 A 88 C 98 B

9 D 19 A 29 B 39 A 49 A 59 D 69 C 79 D 89 D 99 C

10 B 20 D 30 B 40 D 50 B 60 C 70 D 80 C 90 D 100 B

Date: 30-04-2017

Page 41: PRELIMS MODEL TEST-11 ANSWER KEY AND EXPLANATION. PMT-11 to 15 Explanation.pdfTARGET PRELIMS 2017 GENERAL STUDIES PAPER - 1 PRELIMS MODEL TEST-11 ANSWER KEY AND EXPLANATION 1. Answer

NEO IAS 0484-3190310, 9446331522, 9446334122 Page 41 www.neoias.com | www.youtube.com/neoias | www.facebook.com/neoias | www.twitter.com/neoias

5. Answer (b) Explanation President summons the joint sitting of the Parliament and Lok Sabha Speaker chair the session. Joint sessions can be summoned in all legislative deadlocks in ordinary bills except money bill. There can be no deadlock in money bill because Lok Sabha alone has the power to pass Money Bill.

6. Answer (a) Explanation It is left to the Legislative Assembly of each State to decide whether to recommend to Parliament abolition of the Council where one exists or to seek establishment of one if none exists. The procedure is simple and does not involve an amendment of the Constitution. The procedure prescribed is a resolution of the Legislative Assembly of the State concerned passed by a special majority followed by an act of Parliament.

7. Answer (c) Explanation Under Article 61, the President of India can be removed from the office by a process of impeachment for the violation of the Constitution. The impeachment is to be initiated by either House of Parliament. The charges are to be framed in the form of resolution, signed at least by 1/4 th members of the total members of the House. The President has to be given a notice of 14 days in advance. The resolution is to be passed by 2/3 rd majority of the total members of the House and then it is to be sent to other House for investigation and decision. If the other House after investigation sustains the charges and passes the identical resolution with 2/3 rd majority of the total membership, the President ceases to hold office from the date such resolution is passed. Insolvency is a condition for the removal of Chairman of UPSC, NHRC, etc.

8. Answer (d) Explanation Some cases are only to be entertained in the Supreme Court of India. Exclusive Jurisdiction of Supreme court means that the apex court is the sole authority to decide on a matter. High courts have the power to enforce fundamental rights and to accept PILs. Removing high court judges is a function of Parliament. Supreme Court original jurisdiction, therefore, is confined to federal matters and disputes regarding election of President and vice President. Removing the high court judge is the prerogative of Parliament.

9. Answer (d) Explanation In Golaknath case Supreme Court ruled that fundamental rights cannot be amended for the implementation of the directive principles of state policy. In Keshavanathi Bharati case Supreme Court upheld Parliament‘s amendments to fundamental rights to enable provisions in DPSP. Finally in Minerva Mills case, the court has clearly ruled that both Fundamental rights and DPSP are complimentary in nature and the balance need to maintained between

them.

10. Answer (b) Explanation Gram Sabha means a body consisting of all persons whose names are included in the electoral rolls for the Panchayat at the village level. The term is defined in the Constitution of India under Article 243(b). As per the article, Gram Sabha exercises such powers and performs such functions at the village level as the Legislature of a State may, by law, provide. The Gram Panchayat has a Secretary who is also the Secretary of the Gram Sabha. This person is not an elected person but is appointed by the government. The Secretary is responsible for calling the meeting of the Gram Sabha and Gram Panchayat and keeping a record of the proceedings.

11. Answer (c) Explanation The specification of ‗Scheduled Areas‘ in relation to a State is done by a notified Order of the President, after consultation with the State Governments concerned. The same applies for altering, increasing, decreasing, incorporating new areas, or rescinding any orders relating to Scheduled Areas.

12. Answer (b) Explanation Lok-Sabha Speaker presides over the joint sitting. In the absence of the Speaker, such persons as is determined by Rules of Procedure made by President shall preside. (Article 118(4)) The rules, so made, provides that, in the absence of Loksabha Speaker, Deputy Speaker will preside and if he is also absent, the deputy chairman of Rajya

Sabha will preside.

Page 42: PRELIMS MODEL TEST-11 ANSWER KEY AND EXPLANATION. PMT-11 to 15 Explanation.pdfTARGET PRELIMS 2017 GENERAL STUDIES PAPER - 1 PRELIMS MODEL TEST-11 ANSWER KEY AND EXPLANATION 1. Answer

NEO IAS 0484-3190310, 9446331522, 9446334122 Page 42 www.neoias.com | www.youtube.com/neoias | www.facebook.com/neoias | www.twitter.com/neoias

13. Answer (d) Explanation All these motions are crucial because all these involve making the executive/council of Ministers accountable to the legislature. Hence all three are exercised in Lok Sabha only. Adjournment motion helps to draw the attention of the House to a recent matter of urgent public importance having serious consequences. Censure motion is moved against an individual Minister or a group of Ministers for the failure to act or not to act for their policy. No-confidence motion is moved in order to ascertain the confidence of the Council of Ministers in the Lok Sabha.

14. Answer (b) Explanation The Parliament makes laws in skeleton form and authorises the executive to make detailed rules and regulations within the framework of the parent law. This is known as delegated legislation or subordinate legislation.

15. Answer (a) Explanation Ministers can be selected from the State Legislative Council as well. But as in the case of Parliament, Ministers are responsible to the lower house that is legislative assembly, only. About 1/3rd of members are elected by members of the Assembly, another 1/3rd by electorates consisting of members of municipalities, district boards and other local authorities in the state, 1/12th by an electorate consisting of teachers, and 1/12th by registered graduates. The remaining members are nominated by the Governor from among those who have distinguished themselves in literature, science, art, the cooperative movement, and social service.

16. Answer (c) Explanation Article 51 in Directive Principle of State Policy clearly delineate the principles of India‘s relation with foreign countries. It states that ‗To promote international peace and security and maintain just and honourable relations between nations; to foster respect for international law and treaty obligations, and to encourage settlement of international disputes by arbitration.

In Part XI of the Constitution which describes Union and State relations have given authority to the Parliament to make laws to implement International Agreements. The Parliament can make laws on any matter in the State List for implementing the international treaties, agreements or conventions. This provision enables the Central government to fulfil its international obligations and commitments. Besides, Seventh schedule of the Constitution mentions foreign relation under Union list.

17. Answer (a) Explanation While DGP heads the State Police Force, Chief Secretary heads the state administrative machinery. Both are senior civil servant officers in highest ranks. However, there is a slight difference in their service conditions as clarified by the recent Supreme Court verdict on T.P Senkumar case. As per the judgment, DGP should have two years of fixed tenure. They cannot be transferred or removed arbitrarily on the sole ground of lack of confidence in him by the Chief Minister. On the other hand, Chief Secretary has pivotal role in overseeing the general administration. Therefore, Chief Secretary must always enjoy the trust of Chief Minister for the larger interest of general administration.

18. Answer (b) Explanation Article 32 deals with right to move the supreme court for the enforcement of fundamental rights. In this context ―locus standi‖ means that the person filing the petition must show that there is a connection between her/ him and the action sought. Basically, one has to show that you will be personally effected by the action for which you have filed the writ.

19. Answer (a) Explanation Any case pending before any court or a case that is likely to be filed before the court (i.e. dispute at pre-litigative stage) are considered in Lok Adalat. Only compoundable offences are settled in Lok Adalat. There is

no court fee to be paid.

20. Answer (d) Explanation Safeguarding public property and abjure violence is a provision incorporated in the list of fundamental duties Also please note that provisions relating to protection of natural environment and preserving the heritage/monuments are covered in both DPSP and Fundamental duties.

21. Answer (d)

Page 43: PRELIMS MODEL TEST-11 ANSWER KEY AND EXPLANATION. PMT-11 to 15 Explanation.pdfTARGET PRELIMS 2017 GENERAL STUDIES PAPER - 1 PRELIMS MODEL TEST-11 ANSWER KEY AND EXPLANATION 1. Answer

NEO IAS 0484-3190310, 9446331522, 9446334122 Page 43 www.neoias.com | www.youtube.com/neoias | www.facebook.com/neoias | www.twitter.com/neoias

Explanation Whip is an official appointed to maintain discipline among and to give necessary information to, members of his party. Whip is followed in other countries such as US and UK as well. And in India, the office of whip is followed as a convention and it has no mention in rules of procedure or in the Constitution.

22. Answer (a) Explanation Public Account Committee, Estimates Committee and Committee on Public Undertaking are the 3 most important committees of the Parliament that helps in having effective financial control over executive. Some key facts about these committees are:

Members of these committees are elected

Chairpersons are appointed by the Speaker.

No Minister can be member of these committees.

The term of the office of the members is one year

Committee Purpose Members Chairman

Public Accounts Committee

Examine audit report of the CAG

22(15 from Lok Sabha and 7 from Rajya Sabha)

From its members

Estimates Committee Examine the estimates of budgets and suggest improvements

30 – All from Lok Sabha From its members

Committee on Public Undertaking

Examine reports and accounts of Public undertaking

22(15 from Lok Sabha and 7 from Rajya Sabha)

From its Lok Sabha members only

23. Answer (b) Explanation The President is elected by an indirect election, that is, by an electoral college, in accordance with the system of proportional representation by means of single transferable vote by secret ballot. The electoral college shall consists of-

The elected members of both the Houses of Parliament;

The elected members of the Legislative Assemblies of the State

The elected members of the Legislative Assemblies of Union Territories of Delhi and Pondicherry.

24. Answer (d) Explanation Gram Nyayalayas are mobile village courts in India established under Gram Nyayalayas Act, 2008 for speedy and easy access to justice system in the rural areas of India. Gram Nyayalaya is a mobile court and exercises the powers of both Criminal and Civil Courts; i.e., the seat of the Gram Nyayalaya will be located at the headquarters of the intermediate Panchayat, but they will go to villages, work there and dispose of the cases.

Gram Nyayalaya are courts of Judicial Magistrate of the first class and its presiding officer (Nyayadhikari) is appointed by the State Government in consultation with the High Court of the State concerned. The Gram Nyayalaya will not be bound by the rules of evidence provided in the Indian Evidence Act, 1872 but shall be guided by the principles of natural justice and subject to any rule made by the High Court. Appeal in criminal cases shall lie to the Court of Session, which shall be heard and disposed of within a period of six months from the date of filing of such appeal. Appeal in civil cases shall lie to the District Court.

25. Answer (b) Explanation District Planning Committee is provided under the 74

th amendment act to consolidate the plans prepared by

panchayats and municipalities in the district and to draft a district development plan.

26. Answer (b) Explanation Indian Constitution is evolved through time through amendments and judicial interpretation. Therefore, the

nature of Indian Constitution is dynamic and it is called as a living document.

27. Answer (a) Explanation Rule of Law, Parliamentary System, Law Making procedure, First Past the Post election system etc. are adapted from the British Constitution. Federal system with strong center is similar to the scheme followed in Canada. Directive Principles of State Policy is adapted from the Irish Constitution. Charter of Fundamental Rights, Judicial review and independence of Judiciary are taken from the US Constitution.

Page 44: PRELIMS MODEL TEST-11 ANSWER KEY AND EXPLANATION. PMT-11 to 15 Explanation.pdfTARGET PRELIMS 2017 GENERAL STUDIES PAPER - 1 PRELIMS MODEL TEST-11 ANSWER KEY AND EXPLANATION 1. Answer

NEO IAS 0484-3190310, 9446331522, 9446334122 Page 44 www.neoias.com | www.youtube.com/neoias | www.facebook.com/neoias | www.twitter.com/neoias

28. Answer (a) Explanation The Pradhan Mantri Ujjwala Yojan was launched on 1st May, 2016. It is aimed at providing 5 Crore LPG connections in the name of women in BPL (Below Poverty Line) households across the country. It aims to reduce the serious health hazards associated with cooking based on fossil fuel. The scheme is also for reducing the number of deaths in India due to unclean cooking fuel. The scheme will be implemented by the Ministry of Petroleum & Natural Gas (first time in the history). This Scheme would be implemented over three years, namely, the FY 2016-17, 2017-18 and 2018-19.

29. Answer (b) Explanation As per Article 3 of the Constitution, an Ordinary Legislation is to be passed to create a new state/ change the name and area of states. (Eg: Andhra Pradesh Reorganisation Act, 2014 created new state Telengana) Such legislation require permission from the President for its introduction. Besides, the President must also refer the bill to the concerned state legislature. However, the views expressed by the state legislature is not binding upon the President. These procedures are applicable in the case of Union Territories as well.

30. Answer (b) Explanation Fundamental Rights granted to the ‗citizens‘ alone are

equality of opportunity in matters of public employment (Article 15)

protection from discrimination on any ground (Article 16)

freedom of speech, peaceful assembly, expression, association, movement, residence and profession (Article 19)

cultural and educational rights (Article 29 and Article 30)

31. Answer (c) Explanation Citizenship Amendment Act 2003 provides for definition, registration, renunciation, conferment of rights on overseas citizen and cancellation of overseas citizenship. An Overseas Citizen of India is entitled to some benefits such as a multiple-entry, multi-purpose life-long visa to visit India. He is exempted from registration with local police authority for any length of stay in India. He enjoys parity with Non-Resident Indians in respect of economic, financial and educational fields except in relation to the acquisition of agricultural or plantation properties. However, he will not enjoy right to equality in public employment. He will not enjoy right to vote and elect to Parliament and hold Constitutional offices. Overseas citizenship can be cancelled on certain grounds which are same grounds on which Indian citizenship can be terminated by deprivation i.e. citizenship was obtained by fraud, showing disloyalty to the Constitution of India etc.

32. Answer (a) Explanation A section of the Preamble of the Constitution is given below: “secure to all its citizens: JUSTICE, social, economic and political; LIBERTY of thought, expression, belief, faith and worship; EQUALITY of status and of opportunity; and to promote among them all FRATERNITY assuring the dignity of the individual and the unity and integrity of the Nation”

33. Answer (d) Explanation

Judicial remedy for the violation of Fundamental Rights are available to both executive, legislative and judicial actions in India.

Fundamental Rights establish political democracy. It mainly deals with political justice and political equality. Economic and Social rights are mentioned under Directive Principles of State Policy

Fundamental Rights are amendable if the amendment does not distort the basic structure of the Constitution.

Some Important features of Fundamental Rights Most of them are available to foreigners and legal persons like corporations and companies. Most of them are available against State‘s arbitrary actions An aggrieved person can directly go to Supreme Court to reinstate his/her fundamental rights, not

necessarily by way of appeal. They can be suspended during national emergency except Article19 and 21

Page 45: PRELIMS MODEL TEST-11 ANSWER KEY AND EXPLANATION. PMT-11 to 15 Explanation.pdfTARGET PRELIMS 2017 GENERAL STUDIES PAPER - 1 PRELIMS MODEL TEST-11 ANSWER KEY AND EXPLANATION 1. Answer

NEO IAS 0484-3190310, 9446331522, 9446334122 Page 45 www.neoias.com | www.youtube.com/neoias | www.facebook.com/neoias | www.twitter.com/neoias

34. Answer (d) Explanation As per Section 123 of RPA 1951, all of these constitute corrupt practices. Section 123(3) was in news recently because of the landmark judgment by the Supreme Court that prohibit the use of religion, race, caste or language as a ground for canvassing votes in an election. Corrupt practices under section 123 are

giving bribery, undue rewards or gifts to voters

attempts to induce the voter to vote for any particular candidate or otherwise he will become an object of divine displeasure.

promotion of hatred for campaigning

usage of religion, race, caste or language for canvassing votes

appealing to national symbols for the furtherance of prospects of the candidate

usage of government vehicles for campaigning

false and purposive allegation against opponent candidates‘ personal character or conduct

spending more money than allowed Acceptance of any offer of liquor or other intoxicants or a dinner to vote for a particular candidate or not to vote for him is bribery. The provision of Corrupt Practice under section 123 (5) will cover conveyance of any elector, to or from any polling station.

35. Answer (b) Explanation As per Article 28 of the Constitution, religious instructions are allowed on a voluntary basis in institutions run by endowments, trusts etc. even though these are recognized and funded by the government. Religious instructions are not allowed in institutions that are wholly run by the state. Right to freedom of religion guaranteed as a fundamental right contains both rights for individuals and denominations. Article 25 ‗Freedom of Consciousness and Free Profession, Practice and Propagation of Religion

is granted to Individuals, while Article 26 guarantees Freedom to manage Religious Affairs to Denominations.

36. Answer (b) Explanation Prohibition and Certiorari are issued when the lower court is found to overstep its jurisdiction. The writ of prohibition means that the Supreme Court and High Courts may prohibit the lower courts such as special tribunals, magistrates, commissions, and other judiciary officers who are doing something which exceeds to their jurisdiction or acting contrary to the rule of natural justice. The writ of Certiorari orders to move a suit from an inferior court to superior court. This is generally done because superior court believes that either the inferior court had no jurisdiction or committed an error of law. While prohibition is available during the pendency of the proceedings and before the order is made, certiorari can

be issued only after the order has been made.

37. Answer (c) Explanation Article 370 in the Constitution defines special provisions enjoyed by Jammu and Kashmir. According to Article 370(d) Most of the provisions in the Constitution can be applied to the state with such exceptions and modifications as specified by the President in consultation with the state government or with the concurrence of the state government. The article also grant power to Parliament to legislate on matters in Union and Concurrent list which correspond to that specified in the Instrument of Accession. These are external affairs, defence, communication and ancillary matters.

38. Answer (c) Explanation Article 301 of the Constitution extends right to freedom of inter-State trade and commerce throughout the territory of India. The executive is incapable of imposing restrictions on the freedom of trade and commerce without the sanction of legislature by law. Both the Parliament and the State Legislature have been authorized to impose restrictions on the freedom of inter-State trade provided it is in public interest. However, the restriction so imposed by the Legislature is subject to Judicial Review and the restrictions shall be reasonable.

39. Answer (a) Explanation As per the Seventh Schedule of the Constitution, State governments are entitled to legislate on Subjects listed under State list and Concurrent List. While legislation on State subjects is an exclusive power enjoyed by states, Union government also enjoy concurrent power to legislate on subjects under concurrent list. If there is a conflict between state and centre on a legislation in a subject under concurrent list, union law shall prevail.

Among the subjects mentioned in the question, Public Order is a state subject. Only the state has the power to legislate on Public Order.

Page 46: PRELIMS MODEL TEST-11 ANSWER KEY AND EXPLANATION. PMT-11 to 15 Explanation.pdfTARGET PRELIMS 2017 GENERAL STUDIES PAPER - 1 PRELIMS MODEL TEST-11 ANSWER KEY AND EXPLANATION 1. Answer

NEO IAS 0484-3190310, 9446331522, 9446334122 Page 46 www.neoias.com | www.youtube.com/neoias | www.facebook.com/neoias | www.twitter.com/neoias

Electricity, Criminal Law and Forest come under the concurrent list where state has concurrent power with center to legislate.

Legislation on Banking is prerogative of Union government only.

40. Answer (d) Explanation Finance Commission recommend the procedures and principles guiding the distribution of financial resources between the Union and the States. Its recommendations are not binding on the government. Article 280 provides for the appointment by the President of a Finance Commission consisting of a Chairman and four members every five years. According to the Finance Commission (Miscellaneous Provisions) Act, 1951 passed to supplement the Constitutional provisions, the chairman of the commission must be a person having experience in public affairs.

A High Court Judge or one qualified to be appointed as such will serve as one of its members.

41. Answer (b) Explanation CAG‘s main function is to audit and report the expenditure from Consolidated Fund, Contingency Fund and Public Account of India and that of states. According to Article 150, the form in which the accounts of the Union and of the States are to be maintained is to be determined by the President on the advice of the CAG. CAG had earlier performed the function of compiling the accounts of the government. But the Comptroller and Auditor-General (Duties, Powers and Conditions of Service) Act, 1971, which, as amended in 1976, relieves CAG of its duty to compile the accounts of the Union. However, Comptroller and Auditor General continues to compile the accounts of all States except the State of Goa.

42. Answer (c) Explanation If a national emergency is declared on ground of external aggression, Fundamental Rights under Article 19 will get automatically suspended. Besides, the enforcement of any fundamental rights, except Article 20 and 21, can be suspended as specified in the Presidential order. In the event of emergency, state assembly will not get dissolved. They will also have the power to legislate on state subjects. But the law enacted by it shall be subject to the law made by Parliament on the same subject. It also means that the distribution of powers between Centre and States is automatically suspended and there emerges a full-fledged unitary State.

43. Answer (b) Explanation Financial Emergency is dealt under Article 360 while National emergency is declared under Article 352. These are not declared concurrently. Once approved the financial emergency shall remain in force for an indefinite period till the President revokes the financial emergency. When the financial emergency is enforced the distributions of financial resources between Centre and State is suspended. The President may also reduce the salaries and allowances of the Civil servants at the Union level and Constitutional functionaries of the Union, including the judges of the Supreme Court and High Courts. . He may direct the Governor of State to reserve all the money and the financial bills passed by the State Legislature for his consideration.

44. Answer (d) Explanation The grounds on which a member of the Parliament and State Legislature shall be disqualified from being members of the House are:

If an elected member voluntarily gives up the membership of his political party and joins any other political party.

If an elected member votes or abstains from voting in House contrary to any direction issued by the political party to which he belongs.

If an independent member of a legislature joins political party after been elected to the House.

If a nominated member of a House joins a political party six months after becoming a member of the House

45. Answer (a) Explanation According to Article 94 Speaker can be removed by the members of the house by a resolution passed by effective majority {majority of the total members of the House} of Lok Sabha. However, a 14 days advanced notice must be given for such resolution. The motion of removal can be considered and discussed only when it has the support of at least 50 members. A majority of all then members of the house is required in the removal of the Vice-President, Removal of presiding officers in Parliament and State Legislative Assembly/council. All then members of the house means effective majority which is calculated by deducting absent and vacant seats from the total strength.

Page 47: PRELIMS MODEL TEST-11 ANSWER KEY AND EXPLANATION. PMT-11 to 15 Explanation.pdfTARGET PRELIMS 2017 GENERAL STUDIES PAPER - 1 PRELIMS MODEL TEST-11 ANSWER KEY AND EXPLANATION 1. Answer

NEO IAS 0484-3190310, 9446331522, 9446334122 Page 47 www.neoias.com | www.youtube.com/neoias | www.facebook.com/neoias | www.twitter.com/neoias

A majority of members present and voting is used in case of No-Confidence Motion, Adjournment motion, cote of thanks to President or Governor‘s address, Election of presiding officers in Parliament and state legislative Assembly and passing of ordinary, money and financial bills. A majority of the total strength of house is known as absolute majority and its not used in the Constitution as a stand alone requirement. But the absolute majority is used along with special majority. A majority of not less than 2/3

rd of the members present and voting is a type of special majority used in creating

All-India Services and to have power of Parliament to legislate on state subject. Such a majority along with absolute majority is used for Constitutional Amendments, abolition or creation of State Legislative Council, removal of judges, CAG and Chief Election Commissioner. A majority of not less than 2/3rd of the total strength of the House is the highest majority needed in the Constitution. It is required for the impeachment of President.

46. Answer (c) Explanation Election commission advises to President and not to Supreme Court on matters relating to the disqualifications of MPs. Rest of the functions are performed by election commission.

47. Answer (b) Explanation Some Constitutional Bodies:

Election Commission

UPSC and State PSCs

Finance Commission

National Commission for SCs

National Commission for STs

Special Officer for Linguistic Minoritites

Comptroller and Auditor General of India

Inter State Council

Attorney General and Advocate General

48. Answer (d) Explanation Under Article 320 of the Constitution of India, the Commission is, inter-alia, required to be consulted on all matters relating to recruitment to civil services and posts. The functions of the Commission under Article 320 of the Constitution are:

Conduct examinations for appointment to the services of the Union.

Direct recruitment by selection through interviews.

Appointment of officers on promotion / deputation / absorption.

Framing and amendment of Recruitment Rules for various services and posts under the Government.

Disciplinary cases relating to different Civil Services.

Advising the Government on any matter referred to the Commission by the President of India. Cadre Management, Pay and Service Condition of civil setvants, training etc. are functions handled by Department of Personnel and Training.

49. Answer (a) Explanation Following annual reports are to be submitted to President of India

UPSC

CAG

National Commission for SCs

National Commission for STs

Central Vigilance Commission National Human Rights Commission Report is submitted to Central Government and Joint Public Service Commission reports are submitted to concerned Governors. Finance commission does not submit any annual report. It is constituted only once in five year.

50. Answer (b) Explanation According to Article 76 of the Constitution the President shall appoint a person who is qualified to be appointed a Judge of the Supreme Court to be Attorney-General for India. Article 124 (3) of the Constitution prescribes that for appointment as a judge of the Supreme Court a person must he

a citizen of India,

has been a judge of any High Court for at least 5 years, or

Page 48: PRELIMS MODEL TEST-11 ANSWER KEY AND EXPLANATION. PMT-11 to 15 Explanation.pdfTARGET PRELIMS 2017 GENERAL STUDIES PAPER - 1 PRELIMS MODEL TEST-11 ANSWER KEY AND EXPLANATION 1. Answer

NEO IAS 0484-3190310, 9446331522, 9446334122 Page 48 www.neoias.com | www.youtube.com/neoias | www.facebook.com/neoias | www.twitter.com/neoias

has been an advocate in a High Court for 10 years or is in the opinion of the President a distinguished jurist. Therefore above mentioned qualifications are required by the Attorney General as well.

51. Answer (d) Explanation The Union territories are administered in accordance with the provisions of Article 239 to 241 of the Constitution of India. Puducherry and Delhi have Legislatures with Lt. Governors and the UT of Andaman and Nicobar Islands have Lt. Governors as Administrators. The Governor of Punjab functions as the Administrator of the UT of Chandigarh and a senior IAS officer functions as Advisor to the Administrator. The remaining 3 UTs viz. Daman and Diu, Dadra and Nagar Haveli and Lakshadweep are adMinistered

through IAS officers who are appointed as Administrators.

52. Answer (b) Explanation Ministers may belong to either House of Parliament, it includes prime Minister as well. While the Ministers can participate in debates in both the house irrespective of the house they belong to, he is entitled to vote only in the House of which he is a member. With regard to powers, privileges and immunities of the Houses of Parliament, their members and its Committees, the two Houses are placed absolutely on equal footing by the Constitution.

53. Answer (b) Explanation Differences between Lok Sabha Speaker and Rajya Sabha Chairman

Speaker Chairman

Member of the house Not a member of the house

Can decide over the money bill No power to decide over money bill

Preside over joint sitting Can‘t preside over joint sitting

Allowed to present, speak and vote at first instance when a resolution for his removal is under consideration

Allowed to present, speak and not vote.

Elected among its members Election and Impeachment procedure is that of vice-President

54. Answer (a)

Explanation Special leave petition means that you take special permission to be heard in appeal against any High Court/tribunal verdict. Usually any issue decided by the State High Court is considered as final, but if there exist any Constitutional issue or legal issue which can only be clarified by the Supreme Court of India then, this leave is granted by the Supreme Court & this is heard as a Civil or Criminal appeal as the case may be. Special leave to appeal are filed before the Supreme Court under Article 136 of the Constitution.

55. Answer (b) Explanation Self sustenance or self sufficiency is not an objective of organizing village panchayats, rather self government is. Since 73

rd amendment make provisions for women reservation, it also aids women empowerment.

Local self government open up avenues for participation of local people in deciding their development needs and thereby ensuring participatory governance and bottom-up policy making.

56. Answer (b) Explanation A money bill is different with respect to an ordinary bill in following grounds:

It is introduced in Lok Sabha only

It is introduced with the prior permission of the President

Rajya Sabha can recommend changes in the bill, but the Lok Sabha may or may not accept the changes.

President can‘t return the bill for reconsideration

57. Answer (c) Explanation Following are the charged expenditures: Salary and Allowances of the President, Speaker / Deputy Speaker of Lok Sabha, Chairman/ Deputy chairman of Rajya Sabha, Salaries and Allowances of Supreme Court judges, Pensions of Supreme Court as well as High Court Judges, Salaries and Allowances of CAG, Lok Pal and Debt charges of Government of India. Salary of high court judges are charged from the consolidated fund of the state.

Page 49: PRELIMS MODEL TEST-11 ANSWER KEY AND EXPLANATION. PMT-11 to 15 Explanation.pdfTARGET PRELIMS 2017 GENERAL STUDIES PAPER - 1 PRELIMS MODEL TEST-11 ANSWER KEY AND EXPLANATION 1. Answer

NEO IAS 0484-3190310, 9446331522, 9446334122 Page 49 www.neoias.com | www.youtube.com/neoias | www.facebook.com/neoias | www.twitter.com/neoias

The above expenditures cannot be voted because these payments are deemed to be guaranteed by the state. Although voting does not take place on such charges, discussion can take place in any house of the Parliament. The demand for grant for these charges is also made on recommendation of the President.

58. Answer (b) Explanation Parliament has the cardinal functions of legislation, control of executive and ventilation of people grievances. Other functions are

Providing the cabinet

Control of the cabinet: cabinet remains in power as long as they enjoy confidence of the majority in the house – Article 75(3)

Criticism of Individual members and cabinet: policies come into wider deliberation and discussions

An organ of information: Ministers provides answers authoritatively through question and answer section in the Parliament.

Highlighting people‘s grievances. Eg: Question hour, half-an-hour discussions, various types of motions and resolutions.

Legislation: several right based legislation in the recent past

Financial Control – Article 109

Agent of Social change – right based social reforms

59. Answer (d) Explanation

Pardon: It annuls both the sentence and the conviction and absolves the offender from all punishment and disqualifications

Commutation: It merely substitutes one form of punishment for another of a lighter character. For example death sentence can be commuted to rigorous imprisonment, rigorous imprisonment to simple imprisonment and simple imprisonment to fine.

Remission: It reduces the amount of sentence without changing its character. For example a sentence of imprisonment for one year may be remitted to six months.

Respite: It means awarding a lesser sentence instead of the penalty prescribed, in view of some special fact, e.g., the pregnancy of a woman offender.

Reprieve: It means stay of execution of a sentence, e.g., pending a proceeding for pardon or commutation.

60. Answer (c) Explanation Meaning of

Cognizable: Police officer can arrest without warrant.

Bailable: Whether bail is possible or not.

Compoundable: Any committed crime, which can be settled by making up for damages.

61. Answer (a) Explanation Shyama Prasad Mukherji Rurban Mission (SPMRM) was launched on 21

st February, 2016. It aims at development

of 300 rural growth clusters called ‗Rurban Clusters‘ (cluster of Smart Villages). It also aims to stop rising rural-urban migration. Rurban areas refer to a cluster of 15-20 villages having about 30 to 40 lakh population. The clusters will be geographically contiguous Gram Panchayats with a population of about 25000 to 50000 in plain and coastal areas and a population of 5000 to 15000 in desert, hilly or tribal areas. Clusters would be strengthened by providing for economic activities, developing skills, local entrepreneurship and infrastructure development.

62. Answer (d) Explanation Article 25, 26 and 27 in the Fundamental rights guarantees freedom of religion and is the foundation of ‗secularism‘. These provisions does not subscribes to western notions of secularism which advocate neat separation between religion and the state. Instead, these provisions come closer to Gandian notion of ‗sarva dharma sambhava‘, that is equal treatment of all religions. Fundamental Duties such as preserving the rich heritage of the country‘s composite culture reflects Gandhian ideals of taking pride in one‘s own value system. Provisions such as prohibition of alcohol, setting up of cooperatives, prohibition of slaughtering cows in the DPSP etc are distinctively Gandhian. Part IX of the Constitution deals, village panchayats which concur with Gandhian ideals of self-governance.

63. Answer (a) Explanation

Page 50: PRELIMS MODEL TEST-11 ANSWER KEY AND EXPLANATION. PMT-11 to 15 Explanation.pdfTARGET PRELIMS 2017 GENERAL STUDIES PAPER - 1 PRELIMS MODEL TEST-11 ANSWER KEY AND EXPLANATION 1. Answer

NEO IAS 0484-3190310, 9446331522, 9446334122 Page 50 www.neoias.com | www.youtube.com/neoias | www.facebook.com/neoias | www.twitter.com/neoias

Various restrictions have been imposed upon the State Legislatures while enacting law relating to Co-operative Societies which was earlier unfettered prior to the incorporation of Part IXB. Article 243ZJ provides a definite restriction which has been imposed upon the State Legislatures regarding fixation of maximum number of Directors of a Co-operative Society which shall not exceed twenty-one. The Constitution does not provide for minimum number of directors. As per the amendment the changes done to Constitution are:- o In Part III of the Constitution, after words ―or unions‖ the words ―Cooperative Societies‖ was added. o In Part IV a new Article 43B was inserted, which says: The state shall endeavour to promote voluntary

formation, autonomous functioning, democratic control and professional management of the co-operative societies‖.

o After Part IXA of the Constitution, a Part IXB was inserted to accommodate state vs centre role

64. Answer (c) Explanation Operation Muskaan-II Campaign (Launched from 1st July, 2016 to 31st July, 2016) is the second phase of ‗Operation Muskaan‘. Under this campaign, missing children and those found abandoned would be rescued and reunited with their families. District children protection office, social welfare department, police department, differently abled welfare office, labour department, Childline 1098 and NGOs has been constituted for ―Operation Muskaan-II‖. Thousands of children have been rescued under the campaign so far.

65. Answer (a) Explanation There are five conditions under which Parliament can make laws on state list.

If Rajya Sabha approves a resolution with special majority and declares that a subject is of national interest. (under Article-249)

During times of national Emergency (under Article 250)

If the legislature of 2 or more state request Parliament to make law for an item in state list (under Article 252)

For implementation of international treaties

During President's rule.

66. Answer (c) Explanation Removing a high court/Supreme Court judge, impeaching the President and electing vice-President are exclusive powers of Parliament. State legislative assembly members do not take part in these.

67. Answer (b) Explanation NIDHI (National Initiative for Development and Harnessing Innovations) was launched in May 2016. It aims to provide technological solutions to the pressing needs of the society and create new avenues for wealth and job creation. It focuses on building a seamless and innovation driven entrepreneurial ecosystem especially by channelizing youth towards it and thereby bringing in the positive impact on the socio-economic development

of the country.

68. Answer (a) Explanation While Constitution grants freedom of expression in Article 19(1) (a), Article 19(2) limit this including in the case of contempt of court. It is mentioned in the Constitution as a restriction to freedom of speech. However, a Contempt of Court Act passed in 1971 to limit courts power in determining what constitute contempt of court. This Act recognize two common form of contempt. Civil contempt and Criminal Contempt. The former refers to willful disobedience of court order and the latter refers to interfering, scandalizing or downgrading the court activities or its stature.

69. Answer (c) Explanation Democratic decentralisation in India attained the present status through continues evolution across decades, starting from the colonial rule. Many attempts have been made in Independent India to set up a vibrant local democracy. Many committees have appointed for this purpose. Some important committees are

Balwant Rai Mehta Committee (1957): appointed to study community development program and national extension service program, the committee recommended a 3-tier Panchayati Raj system composed of electoral representatives having enough independence and autonomy.

Santhanam Committee (1963) constituted to study financial relations with Panchayati Raj institutions and gave some practical recommendations to matters of financial relevance.

Ashok Mehta Committee (1977) recommended that 3-tier should be replaced by the 2-tier system and Mandal panchayats/group of villages must be the base of panchayati raj system.

GVK Rao Committee (1985) recommended and emphasised on the district level Zila Parishad which should be the basic unit for policy planning and programme implementation.

Page 51: PRELIMS MODEL TEST-11 ANSWER KEY AND EXPLANATION. PMT-11 to 15 Explanation.pdfTARGET PRELIMS 2017 GENERAL STUDIES PAPER - 1 PRELIMS MODEL TEST-11 ANSWER KEY AND EXPLANATION 1. Answer

NEO IAS 0484-3190310, 9446331522, 9446334122 Page 51 www.neoias.com | www.youtube.com/neoias | www.facebook.com/neoias | www.twitter.com/neoias

L. M Singvi Committee (1986) recommended Constitutional status for Panchayats and emphasized on the role of Gram Sabhas.

70. Answer (d) Explanation Inter-State Council established via Article 263 is the first Constitutional body having a representation from both union and state governments. Inter-State Council include prime Minister, six cabinet Ministers and chief Ministers/administrators of states/UTs. However, one can rightly say that GST council would be the first truly federal Constitutional body having substantial federal powers. Decisions in the council is to be taken with a majority of three-fourth of the weighted votes. Union government has one-third of the weighted votes and states have the rest. However, decisions are mainly taken with consensus. The Constitution (One Hundred and First Amendment) Act 2016 inserted Article 279A into the Constitution which provided for the establishment of GST Council. The composition of the council include representatives from both union government and state governments (Finance Minister, MoS in charge of finance/revenue and Minister in charge of finance/taxation from states or Ministers recommended by the states).

71. Answer (b) Explanation Cooperative federalism refers to participation of both Union and Federal units in the governance of the country in a cooperative manner. In the recently established GST Council, decisions are taken jointly by Union Government and State Governments. NITI Aayog also enables participation and support to state governments in the development and governance activities. It brings States together in national interest, and thereby fosters Cooperative Federalism. On the other hand, Monetary Policy Committee is a committee of the Central Bank in India which is entrusted with the task of fixing the benchmark policy interest rate. It has participation from central government and RBI and does not involve state governments.

72. Answer (a) Explanation 7

th Schedule divides the legislative powers of union and state governments. Therein, ‗preservation, protection and

improvement of stock and prevention of animal disease, veterinary training and practice’ come under the state list. Even though fundamental duties mention protection of wild life, it does not mention anything specifically about cattle. In the Directive Principles of State Policy, Article 48 urges the state to take steps for preserving and improving the breeds, and prohibiting the slaughter, of cows and calves and other milch and draught cattle. Ninth Schedule does not mention anything about cattle.

73. Answer (b) Explanation Under Article 80(3) of the Constitution, the Council of States (Rajya Sabha) includes 12 members nominated by the President of India from amongst persons who have special knowledge or practical experience in literature, science, art and social service. The rationale behind such system is to not only recognizes their merit and confers honour on them, but also enables them to enrich the debates by their expertise and knowledge. Nominated members enjoy all powers, privileges and immunities available to an elected member of Parliament. They take part in the proceedings of the House as any other member. They, however, are not entitled to vote in the election of the President of India. But in the election of the Vice-President of India, they have a right to vote. So far, none from them has been inducted into the Council of Ministers. A nominated member is allowed six months, should he decide to join a political party after he has taken his seat in the House in terms of Article 99 of the Constitution. A nominated member has also been exempted from filing his assets and liabilities under Section 75A of the Representation of the Peoples Act, 1951 which requires the elected member to do so within 90 days of his making or subscribing oath/affirmation.

74. Answer (c) Explanation The Pradhan Mantri Awas Yojana - Urban (PMAY-U) aims to building affordable pucca houses with water facility, sanitation and electricity supply. It aims to transform slum areas by building homes for slum dwellers in collaboration with private developers. It is meant to cover people in the economically weaker sections, low-income group and mid-income group. Further specifications of people are: Indian women of all religions and castes, Scheduled tribes and scheduled castes. This ‗Housing For All - Urban‘ scheme to be achieved by the year 2022. Under the scheme, construction developers should strictly follow eco-friendly development technologies.

Around 305 cities were identified by the government for the implementation of this scheme.

75. Answer (c) Explanation

Page 52: PRELIMS MODEL TEST-11 ANSWER KEY AND EXPLANATION. PMT-11 to 15 Explanation.pdfTARGET PRELIMS 2017 GENERAL STUDIES PAPER - 1 PRELIMS MODEL TEST-11 ANSWER KEY AND EXPLANATION 1. Answer

NEO IAS 0484-3190310, 9446331522, 9446334122 Page 52 www.neoias.com | www.youtube.com/neoias | www.facebook.com/neoias | www.twitter.com/neoias

National Commission for Backward Classes is set up through the enactment of National Commission for Backward Classes Act, 1993 to determine and recommend both addition and omission of different classes under OBC list. It is not a Constitutional body as of now. The Commission consists of five Members, comprising of a Chairperson who is or has been a judge of the Supreme Court or of a High Court; a social scientist; two persons, who have special knowledge in matters relating to backward classes; and a Member-Secretary, who is or has been an officer of the Central Government in the rank of a Secretary to the Government of India. The Commission have all the powers of a civil court. And the advice of the Commission shall ordinarily be binding upon the Central Government. The commission was on news because Ministry of Social Justice has put forward a new Constitutional Amendment bill to replace existing commission with more powers and to grant Constitutional status.

76. Answer (a) Explanation The National Commission for Women was set up as statutory body in January 1992 under the National Commission for Women Act, 1990 (Act No. 20 of 1990 of Govt. of India) to:

review the Constitutional and Legal safeguards for women;

recommend remedial legislative measures;

facilitate redressal of grievances and

advise the Government on all policy matters affecting women. One Stop Center Scheme is a Centrally Sponsored Scheme run by Ministry of Women and Child Development (MWCD), for setting up One Stop Centre to provide integrated support and assistance under one roof to women affected by violence, both in private and public spaces in phased manner. Parivarik Mahila Lok Adalat (PMLA) is the alternative justice delivery system organized by National Commission for Women (NCW) to supplement the efforts of district legal services authority (DLSA) for redressal and speedy disposal of matters related to family and marriage issues.

77. Answer (a) Explanation The Returning Officer is responsible for the conduct of elections in the Parliamentary or assembly constituency concerned as per section 21 of the Representation of the People Act 1951. The Election Commission of India nominates or designates an officer of the Government or a local authority as the Returning Officer for each of the assembly and Parliamentary constituencies in consultation with the State Government/Union Territory Administration.

78. Answer (a) Explanation Various departments in the Finance Ministry are

Economic Affairs

Expenditure

Revenue

Financial Services

Department of Investment and Public Asset Management

79. Answer (d) Explanation NHRC can take up a case on its own initiative (suo moto) or on a petition presented to it by a victim or any person on his behalf. It can also take up a case when courts directs.

80. Answer (c) Explanation Tax on income on agriculture is levied by the state government. Tax on professions, trades, callings and employments is also included in the state list. Land Revenue, Tolls, Stamp duties except on financial documents, capitation taxation, mineral rights, taxes on land and buildings are some of the important entries in the state list. Tax on income, other than agriculture income, corporate tax, custom duties, excise duty, terminal taxes, Taxes on the sale or purchase of goods in the course of inter-state trade etc. are prerogative of union. Recently passed 101

st Constitution Amendment Act to introduce GST omitted the item service tax from the union

list.

81. Answer (b) Explanation Article 323A empowers the Parliament by law for the establishment of administrative tribunals and Article 323B for the establishment of tribunals such as income tax, land dispute tribunal etc. Since they are established through Law rather than a Constitutional Amendment Act, they are statutory bodies. It is also confirmed by Supreme Court saying that the administrative tribunals are statutory bodies.

Page 53: PRELIMS MODEL TEST-11 ANSWER KEY AND EXPLANATION. PMT-11 to 15 Explanation.pdfTARGET PRELIMS 2017 GENERAL STUDIES PAPER - 1 PRELIMS MODEL TEST-11 ANSWER KEY AND EXPLANATION 1. Answer

NEO IAS 0484-3190310, 9446331522, 9446334122 Page 53 www.neoias.com | www.youtube.com/neoias | www.facebook.com/neoias | www.twitter.com/neoias

The Administrative Tribunals are not bound by the technical rules of Civil Procedure Code and Evidence Act. They have the power to adapt their own rules of procedure. They enjoy some of the powers of civil court in certain matters and proceedings are constrained to be judicial in nature.

82. Answer (a) Explanation English is not included in the Eighth Schedule. The Eighth Schedule to the Constitution consists of the following 22 languages:- (1) Assamese, (2) Bengali, (3) Gujarati, (4) Hindi, (5) Kannada, (6) Kashmiri, (7) Konkani, (8) Malayalam, (9) Manipuri, (10) Marathi, (11) Nepali, (12) Oriya, (13) Punjabi, (14) Sanskrit, (15) Sindhi, (16) Tamil, (17) Telugu, (18) Urdu (19) Bodo, (20) Santhali, (21) Maithili and (22) Dogri.

83. Answer (c) Explanation NITY Aayog is the premier policy ‗Think Tank‘ of the Government of India, providing both directional and policy inputs. While designing strategic and long-term policies and programmes for the Government of India, NITY Aayog also provides relevant technical advice to the Central and States. The centre-to-state one-way flow of policy, that was the hallmark of the Planning Commission era, is now sought to be replaced by a genuine and continuing partnership of states.

84. Answer (d) Explanation According to Article 329, the election disputes in case of both Houses of Parliament and the State Legislature can be challenged only in the form of filing the election petition before the High Court concerned and election petition can be filed only after the completion of polls and declaration of official results. The High Court is prohibited for entertaining elections petition at any stage before the announcement of the official result. This is to ensure that the elections are held as scheduled and the democratic process is not derailed. The decision of the High Court is subject to the appellate jurisdiction of the Supreme Court.

85. Answer (c) Explanation Cabinet Secretary‘s main function is to coordinate between different ministries and departments for the smooth transaction of functions. An Apex Committee headed by Cabinet Secretary is also responsible for overseeing the Digital India programme and providing policy and strategic directions for its implementation and resolving inter-Ministerial issues.

86. Answer (b) Explanation Department of Space is an independent Ministry. Department of Atomic research is also an Independent Ministry. Department of Justice is a department under Ministry of Law and Justice. Department of States is under the Ministry of Home Affairs.

87. Answer (a) Explanation Ministry of Home Affairs is the nodal ministry. NDMA was established through the Disaster Management Act 2005. The vision of NDMA is "To build a safer and disaster resilient India by a holistic, pro-active, technology driven and sustainable development strategy that involves all stakeholders and fosters a culture of prevention, preparedness and mitigation."

88. Answer (c) Explanation Drafting legislation is mainly the responsibility of Ministry of Law and Justice. Ministry of Parliamentary Affairs keeps a watch over the progress of Bills from the stage of approval by the Cabinet till the Bill is passed by both Houses of the Parliament. In order to ensure smooth passage of Bills, in Parliament, officials of the Ministry have to be in constant touch with the officials of Ministries/Departments sponsoring the Bills and the Ministry of Law, Justice and Company Affairs which drafts the Bills.

89. Answer (d) Explanation Various States have been appointing Parliamentary Secretaries to assist cabinet miniters. They are usually have the same status as Minister of States. It is argued that these Parliamentary Secretaries hold the Office of Profit and must be disqualified. However, it is yet to emerge a clear picture on the exact nature of the position. Various Courts, however, seen the practice as unconstitutional because it will contravene the Constitutional provision on maximum number of Ministers. Also since they work as part of executive, these legislators may obliged to executive. Therefore, the practice need to be discouraged.

Page 54: PRELIMS MODEL TEST-11 ANSWER KEY AND EXPLANATION. PMT-11 to 15 Explanation.pdfTARGET PRELIMS 2017 GENERAL STUDIES PAPER - 1 PRELIMS MODEL TEST-11 ANSWER KEY AND EXPLANATION 1. Answer

NEO IAS 0484-3190310, 9446331522, 9446334122 Page 54 www.neoias.com | www.youtube.com/neoias | www.facebook.com/neoias | www.twitter.com/neoias

90. Answer (d) Explanation Prime Minister‘s Office [PMO] assists the Prime Minister in the efficient discharge of his role, functions and responsibilities. It is headed by a Principal Secretary who has always been a senior civil servant or a retired bureaucrat. Inter-Ministerial coordination and framing rules of business are functions of Cabinet Secretariat, not the

responsibility of PMO. But PMO may engage in any inter-Ministerial conflict so as to resolve it.

91. Answer (c) Explanation Preamble of Indian Constitution reads ―We, the People of India having solemnly resolved to constitute India into a Sovereign, Socialist, Secular, Democratic, Republic‖

92. Answer (a) Explanation Pradhan Mantri Kaushal Vikas Yojana (PMKVY) was launched on 16th July, 2015. This will be the flagship scheme for skill training of youth to be implemented by the new Ministry of Skill Development and Entrepreneurship through the National Skill Development Corporation (NSDC). The scheme will cover 24 lakh persons. It aims to enable a large number of Indian youth to take up industry-relevant skill training that will help them in securing a better livelihood. Skill training would be done based on the National Skill Qualification Framework (NSQF) and

industry led standards.

93. Answer (b) Explanation Some of the Important changes brought by 42

nd Constitutional Amendment Act, 1976

The preamble has been amended to substitute the words ―SOVEREIGN DEMOCRATIC REPUBLIC‖, with the words ―SOVEREIGN SOCIALIST SECULAR DEMOCRATIC REPUBLIC‖ and the words ―unity of the Nation‖ was substituted with ―unity and integrity of the Nation.‖

New directives was added by new articles 39A, 43A, 48A which, respectively, provide for equal justice and free legal aid to economically backward classes, participation of workers in the management of industries, and protection and improvement of environment and safeguarding of forests and wildlife.

New Part IVA containing article 51A was added to provide lists of fundamental duties of citizens.

Article 74(1) was amended to make the President to act in accordance with the advice of the Council of Ministers.

It curtailed the power of the Supreme Court and High Court with regard to the issue of writs and judicial review.

It transferred subjects like forests, education, weights and measures except establishments of standards, protection of wild animals and birds from the State List to the Concurrent List. New entry 20A was added in Concurrent List which is ―Population control and family planning‖.

94. Answer (d) Explanation The basic structure doctrine is an Indian judicial principle that the Constitution of India has certain basic features that cannot be altered or destroyed through amendments by the Parliament. The doctrine thus forms the basis of power of the Supreme Court to review and strike down Constitutional amendments enacted by the Parliament which conflict with or seek to alter this "basic structure" of the Constitution. The sovereign, democratic and secular character of the polity, rule of law, independence of the judiciary, fundamental rights of citizens etc. are some of the essential features of the Constitution that have appeared time and again in the apex court's pronouncements.

95. Answer (b) Explanation List of Fundamental Duties: Art. 51A, Part IVA of the Indian Constitution, specifies the list of fundamental duties of the citizens. It says ―it shall be the duty of every citizen of India:

to abide by the Constitution and respect its ideal and institutions;

to cherish and follow the noble ideals which inspired our national struggle for freedom;

to uphold and protect the sovereignty, unity and integrity of India;

to defend the country and render national service when called upon to do so;

to promote harmony and the spirit of common brotherhood amongst all the people of India transcending religious, linguistic and regional diversities, to renounce practices derogatory to the dignity of women;

to value and preserve the rich heritage of our composite culture;

to protect and improve the natural environment including forests, lakes, rivers, and wild-life and to have compassion for living creatures;

to develop the scientific temper, humanism and the spirit of inquiry and reform;

to safeguard public property and to abjure violence;

Page 55: PRELIMS MODEL TEST-11 ANSWER KEY AND EXPLANATION. PMT-11 to 15 Explanation.pdfTARGET PRELIMS 2017 GENERAL STUDIES PAPER - 1 PRELIMS MODEL TEST-11 ANSWER KEY AND EXPLANATION 1. Answer

NEO IAS 0484-3190310, 9446331522, 9446334122 Page 55 www.neoias.com | www.youtube.com/neoias | www.facebook.com/neoias | www.twitter.com/neoias

to strive towards excellence in all spheres of individual and collective activity, so that the nation constantly rises to higher levels of endeavor and achievement.

who is a parent or guardian, to provide opportunities for education to his child, or as the case may be, ward between the age of six and fourteen years.

96. Answer (d) Explanation Residuary powers include subjects not mentioned in either of the lists that is not mentioned in Union, state and concurrent lists. Cyber Security can said to be a residuary power. These powers are vested with Union Government.

97. Answer (b) Explanation In order to get formal recognition for Leader of Opposition, the concerned party must have at least 10% of the total strength of the House (55 seats in the Lok Sabha). If any party fails to get 10% seats in opposition, the House will not have recognized leader of the opposition. A single party has to meet the 10% seat criteria, not an alliance. In the present Lok Sabha, Congress, the largest opposition party has only 44 seats. Therefore its leader does not have the formal recognition as ‗Leader of Opposition‘

98. Answer (b) Explanation A bill that is pending in Lok Sabha, despite where it has originated, lapses when the house is dissolved. Also a bill originated in the Lok Sabha but pending in the Rajya Sabha also lapses. No other bills will lapse on dissolution.

99. Answer (c) Explanation

President of India submits his/her resignation letter to the Vice-President.

100. Answer (b) Explanation Sansad Adarsh Gram Yojana (SAGY) is a village development project launched by Government of India in October 2014, under which each Member of Parliament will take the responsibility of developing physical and institutional infrastructure in three villages by 2019. It is aimed at instilling certain value in the villages and their people so that they get transformed into models for others. The goal is to develop three Adarsh Grams by March 2019, of which one would be achieved by 2016. Thereafter, five such Adarsh Grams (one per year) will be selected and developed by 2024.

Page 56: PRELIMS MODEL TEST-11 ANSWER KEY AND EXPLANATION. PMT-11 to 15 Explanation.pdfTARGET PRELIMS 2017 GENERAL STUDIES PAPER - 1 PRELIMS MODEL TEST-11 ANSWER KEY AND EXPLANATION 1. Answer

NEO IAS 0484-3190310, 9446331522, 9446334122 Page 56 www.neoias.com | www.youtube.com/neoias | www.facebook.com/neoias | www.twitter.com/neoias

TARGET PRELIMS 2017 GENERAL STUDIES

PAPER - 1

PRELIMS MODEL TEST-14

ANSWER KEY AND EXPLANATION

1. Answer (d) Explanation Aquila is the code name of the unmanned solar-powered drone developed by Facebook. The company plans to use a linked network of the drones to provide internet access to large rural areas. Recently Facebook completed the first successful flight of Aquila. However, as with its Internet.org project, Facebook will not be dealing with customers directly, instead partnering with local ISPs to offer the services.

2. Answer (a) Explanation Cyber Physical Systems (CPS) is an interdisciplinary field that deals with the deployment of computer-based systems that do things in the physical world. It deals with self-driven cars, autonomous unmanned vehicles and aircraft navigation systems etc. Department of Science and Technology (DST) has recently launched a Cyber Physical Systems (CPS) program.

3. Answer (d) Explanation VoLTE stands for Voice-over-LTE, and essentially, it is a technology that makes it possible for voice calls to be carried over LTE (Long Term Evolution) networks. Right now, LTE networks only carry data, leaving voice calls to be carried by older 3G networks. With VoLTE, voice and data will travel through the same network. The clearest benefit for consumers will be higher-quality voice calls, otherwise known as HD Voice. Calls made between VoLTE devices will have clearer audio with less background noise.

4. Answer (b) Explanation The ―Cyber Swachhta Kendra " (Botnet Cleaning and Malware Analysis Centre) is a part of the Government of India's Digital India initiative under the Ministry of Electronics and Information Technology (MeitY) to create a secure cyber space by detecting botnet infections in India and to notify, enable cleaning and securing systems of end users so as to prevent further infections. The ―Cyber Swachhta Kendra " (Botnet Cleaning and Malware Analysis Centre)

1 D 11 C 21 B 31 B 41 D 51 B 61 C 71 D 81 D 91 D

2 A 12 B 22 C 32 C 42 A 52 A 62 B 72 D 82 D 92 C

3 D 13 C 23 B 33 A 43 A 53 D 63 C 73 D 83 C 93 A

4 B 14 B 24 C 34 A 44 C 54 B 64 C 74 D 84 D 94 D

5 C 15 C 25 B 35 B 45 A 55 B 65 C 75 C 85 B 95 D

6 D 16 D 26 A 36 B 46 C 56 C 66 B 76 C 86 A 96 C

7 A 17 A 27 B 37 A 47 B 57 D 67 A 77 D 87 B 97 C

8 C 18 B 28 C 38 C 48 A 58 D 68 D 78 D 88 B 98 D

9 C 19 B 29 D 39 C 49 C 59 D 69 B 79 D 89 D 99 B

10 D 20 C 30 B 40 C 50 A 60 A 70 B 80 D 90 C 100 D

Date: 07-05-2017

Page 57: PRELIMS MODEL TEST-11 ANSWER KEY AND EXPLANATION. PMT-11 to 15 Explanation.pdfTARGET PRELIMS 2017 GENERAL STUDIES PAPER - 1 PRELIMS MODEL TEST-11 ANSWER KEY AND EXPLANATION 1. Answer

NEO IAS 0484-3190310, 9446331522, 9446334122 Page 57 www.neoias.com | www.youtube.com/neoias | www.facebook.com/neoias | www.twitter.com/neoias

is set up in accordance with the objectives of the "National Cyber Security Policy", which envisages creating a secure cyber eco system in the country. This centre operates in close coordination and collaboration with Internet Service Providers and Product/Antivirus companies.

5. Answer (c) Explanation Aadhaar Enabled Payment System (AEPS) is a new payment service offered by the National Payments Corporation of India to banks, financial institutions using ‗Aadhaar‘ number and online UIDAI authentication through their respective Business correspondent service centre. AEPS functions through the National Financial Switch of NPCI, which is also used for routing inter bank ATM transactions and Interbank Mobile Payment Service (IMPS). AEPS is a bank led model which allows online interoperable financial inclusion transaction at PoS (Point of Sale Devices/MicroATM) through the Business correspondent of any bank using the Aadhaar authentication.

6. Answer (d) Explanation DigiSevak is an online volunteering platform for interested citizens who want to contribute to the success of Digital India program. Various government department and agencies can create volunteering tasks and volunteers can choose tasks based on their skills and interest areas. The platform provides means for end-to-end execution of a volunteering task, right from registration of volunteers to creation of tasks, evaluation of tasks and rewards & recognition of contribution by volunteers.

7. Answer (a) Explanation Long Term Evolution (LTE) is a fourth generation mobile wireless technology. 4G network is currently deployed using LTE (Long Term Evolution) and WiMAX (Worldwide Interoperability for Microwave Access) technologies. EDGE is a second generation mobile wireless technology. Data connection on EDGE networks is faster than GPRS (General Packet Radio Service) but is slower than 3G.

8. Answer (c) Explanation In line with the Government's Digital India initiative, eBASTA project has created a framework to make school books accessible in digital form as e-books to be read and used on tablets and laptops. The main idea is to bring various publishers (free as well as commercial) and schools together on one platform. In addition to the portal, a back-end framework to facilitate the organization and easy management of such resources has been developed, along with the web-based applications that can be installed on tablets for navigating the framework.

9. Answer (c) Explanation Quantum communication lines boasts ultra-high security. It is impossible to wiretap, intercept or crack the information transmitted through them. In November 2016, China launched a 712-km quantum communication line, stated to be the world‘s longest secure telecommunications network, which boasts of ultra-high security making it impossible to wiretap, intercept or crack the information transmitted through them. The new quantum communication line links Hefei, capital of Anhui province, to Shanghai, the country‘s financial hub. It is part of a 2,000-km quantum communication line connecting Beijing and Shanghai.

10. Answer (d) Explanation Digitize India Platform (DIP) is an initiative of the Government of India under the Digital India Programme to provide digitization services for scanned document images or physical documents for any organization. The aim is to digitize and make usable all the existing content in different formats and media, languages, digitize and create data extracts for document management, IT applications and records management.

11. Answer (c) Explanation Internet of Things generally refers to scenarios where network connectivity and computing capability extends to objects, sensors and everyday items not normally considered computers. The Centre of Excellence for IoT was announced as a part of the Digital India Initiative to jump start the IoT ecosystem taking advantage of India's IT strengths and help the country attain a leadership role in the convergent area of hardware and software. The main objective of the center is to create innovative applications and domain capability. Additionally, the center will help build industry capable talent, start-up community and an entrepreneurial ecosystem for IoT.

12. Answer (b) Explanation One of the core principles governing the internet is Net Neutrality. It is the principle that Internet service providers (ISP) and governments should treat all data on the Internet the same, not discriminating or charging differentially by

Page 58: PRELIMS MODEL TEST-11 ANSWER KEY AND EXPLANATION. PMT-11 to 15 Explanation.pdfTARGET PRELIMS 2017 GENERAL STUDIES PAPER - 1 PRELIMS MODEL TEST-11 ANSWER KEY AND EXPLANATION 1. Answer

NEO IAS 0484-3190310, 9446331522, 9446334122 Page 58 www.neoias.com | www.youtube.com/neoias | www.facebook.com/neoias | www.twitter.com/neoias

user, content, site, platform, application, type of attached equipment, or mode of communication. In other words, it is the principle that the company that connects you to the Internet does not get to control what you do on it.

13. Answer (c) Explanation The Origins Spectral Interpretation Resource Identification Security - Regolith Explorer (OSIRIS-REX) is a spacecraft to study on near-Earth asteroid, called Bennu (formerly 1999 RQ36). It was launched on Sept. 8, 2016. As planned, the spacecraft will reach its asteroid target in 2018 and return a sample to Earth in 2023. It was developed by NASA. The mission will help scientists investigate how planets formed and how life began, as well as improve our understanding of asteroids that could impact Earth. This will be NASA‘s first asteroid sampling mission.

14. Answer (b) Explanation Gaia was launched by European Space Agency. Gaia will monitor each of its target stars about 70 times over a five-year period. It will precisely chart their positions, distances, movements, and changes in brightness. It is expected to discover hundreds of thousands of new celestial objects, such as extra-solar planets and brown dwarfs, and observe hundreds of thousands of asteroids within our own Solar System. The mission will also study about 500 000 distant quasars and will provide stringent new tests of Albert Einstein‘s General Theory of Relativity. Recently, it has mapped more than a billion stars in the Milky Way, vastly expanding the inventory of known stars in our galaxy.

15. Answer (c) Explanation Light utility helicopter (LUH) is a light, single-engine helicopter that is powered by the Safran HE Ardiden-1U engine. It can fly six passengers at sea level, with load capacity reducing with altitude. It was developed by Hindustan Aeronautics Limited (HAL). The light utility helicopter (LUH) will replace the military‘s obsolescent fleet of Chetak and Cheetah helicopters that have flown for over three decades. Apart from replacing ageing fleet of Cheetah/Chetak, LUH is expected to capture a sizeable share both in domestic and international market. It can operate from sea levels to the high altitudes of the Himalayas.

16. Answer (d) Explanation INSAT-3DR similar to INSAT-3D, is an advanced meteorological satellite of India configured with an imaging System and an Atmospheric Sounder. It was launched by GSLV-F05. It will provide service continuity to earlier meteorological missions of ISRO and further augment the capability to provide various meteorological as well as search and rescue services. INSAT-3DR has a lift-off mass of 2211 kg, which includes about 1255 kg of propellant. The propellant carried by INSAT-3DR is mainly required to raise the satellite from the Geosynchronous Transfer Orbit (GTO) to its final Geostationary Orbit and to maintain the satellite in its orbital slot during its life.

17. Answer (a) Explanation INS Mormugao is the second ship of the Visakhapatnam-class stealth guided-missile destroyers of the Indian Navy. She is being constructed at Mazagon Dock Limited (MDL), and has been launched on 17 September 2016. The ship is expected to get commissioned by 2020.

18. Answer (b) Explanation US space agency NASA has developed a high-tech electroactive bandage that creates an electric charge to help promote the healing process of wounds. The bandage is made of an electroactive material that is stimulated by pressure of cell growth and body heat. The electroactive material is made using a polyvinylidene fluoride (PVDF). An electroactive device is applied to an external wound site, which utilises low level electrical stimulation to promote wound healing. The bandage minimises infection and related complications such as illness or amputation.It also speeds the wound‘s healing process.

19. Answer (b) Explanation Mining Surveillance System (MSS) was developed by the Indian Bureau of Mines (IBM), Bhaskaracharya Institute for Space Applications and Geo-informatics (BISAG), Gandhinagar, and the ministry of electronics and information technology (MEITY). It aims to curb illegal mining without human intervention and subjectivity. It will receive tip offs in the form of pictures taken from a mobile phone, automatically generate complaints to state authorities and publish the outcome of a probe in a mobile application. It detects suspicious activities within a range of 500 metres outside the mining lease boundary.

20. Answer (c)

Page 59: PRELIMS MODEL TEST-11 ANSWER KEY AND EXPLANATION. PMT-11 to 15 Explanation.pdfTARGET PRELIMS 2017 GENERAL STUDIES PAPER - 1 PRELIMS MODEL TEST-11 ANSWER KEY AND EXPLANATION 1. Answer

NEO IAS 0484-3190310, 9446331522, 9446334122 Page 59 www.neoias.com | www.youtube.com/neoias | www.facebook.com/neoias | www.twitter.com/neoias

Explanation The Great Balls of Fire (GBF) are mysterious, super-hot blobs of gas. They were detected by the Hubble Space Telescope. The GBF have continued once every 8.5 years for at least the past 400 years. The gas balls were observed near a red giant called V Hydrae, about 1,200 light years away from earth. (Red giants are stars that are nearing the end of their fuel supplies and have begun to puff up and expand).

21. Answer (b) Explanation CYSVAX is the world's first vaccine to fight against Tapeworms in pigs. Aim of the recombinant porcine (pigs) cysticercosis vaccine - Cysvax is to fight against tapeworms in pigs and this may also help reduce incidence of epilepsy in human beings. It was developed by Indian Immunologicals Ltd (IIL). (It was set up by the National Dairy Development Board (NDDB) in 1982, as its unit, with the objective of making Foot and Mouth Disease (FMD) vaccine available to farmers at an affordable price.)

22. Answer (c) Explanation Deep brain stimulation (DBS) is a surgical procedure used to treat a variety of disabling neurological symptoms—most commonly the debilitating symptoms of Parkinson‘s disease (PD), such as tremor, rigidity, stiffness, slowed movement, and walking problems. It is popularly known as the pacemaker for the brain. It is only for patients who can‘t control their movements. It does not damage healthy brain tissue by destroying nerve cells. Instead the procedure blocks electrical signals from targeted areas in the brain.

23. Answer (b) Explanation Druzhba 2016 is joint military training exercise between Pakistan and Russia. It aims to develop coordination in carrying out practice combat tasks in a mountainous environment, including operations against illegal armed groups. The two sides (Russia and pakisthan) have taken several steps to improve defence relations as India‘s military procurements moved away from Russia to the US and Israel. The drill reflects the growing military-to-military ties between Pakistan and Russia, whose relations were strained for decades following the Soviet intervention in Afghanistan.

24. Answer (c) Explanation Magnetar is an extremely dense type of neutron star. Neutron stars are formed when stars more massive than our sun explode as supernovae. Magnetar can produce magnetic fields trillions of times stronger than our sun‘s. They are 1,000 times more powerful than that (i.e. Just a regular neutron star has a magnetic field of a trillion gauss), with a magnetic field of a quadrillion gauss. The magnetar releases a blast of radiation that can be seen clear across the Milky Way. It can cause starquakes, or earthquakes on stars. The most powerful starquake ever recorded came from a magnetar called SGR 1806-20, located about 50,000 light years away (In a tenth of a second, one of these starquakes released more energy than the sun gives off in 1,00,000 years).

25. Answer (b) Explanation A proteome is the set of proteins thought to be expressed by an organism. The majority of the UniProt proteomes are based on the translation of a completely sequenced genome, and will normally include sequences that derive from extra-chromosomal elements such as plasmids or organellar genomes in organisms where these occur. Some proteomes may also include protein sequences based on high quality cDNAs that cannot be mapped to the current genome assembly due to sequencing errors or gaps. These are only included in the proteome following manual review of the supporting evidence, including careful analysis of homologous sequences from closely related organisms.

26. Answer (a) Explanation A scramjet (supersonic combusting ramjet) is a variant of a ramjet airbreathing jet engine in which combustion takes place in supersonic airflow. As in ramjets, a scramjet relies on high vehicle speed to forcefully compress the incoming air before combustion (hence ramjet), but a ramjet decelerates the air to subsonic velocities before combustion, while airflow in a scramjet is supersonic throughout the entire engine. It was developed by Indian Space Research Organisation (ISRO). It uses Hydrogen as fuel and the Oxygen from the atmospheric air as the oxidiser. ISRO‘s Advanced Technology Vehicle (ATV), which is an advanced sounding rocket, was the solid rocket booster used for test of Scramjet engines at supersonic conditions. ATV carrying Scramjet engines weighed 3277 kg at lift-off. The Scramjet engine will also be used eventually to power ISRO‘s reusable launch vehicle at hypersonic speed. India is the fourth country to demonstrate the flight testing of Scramjet Engine after the United States, Russia and European Space Agency.

27. Answer (b) Explanation

Page 60: PRELIMS MODEL TEST-11 ANSWER KEY AND EXPLANATION. PMT-11 to 15 Explanation.pdfTARGET PRELIMS 2017 GENERAL STUDIES PAPER - 1 PRELIMS MODEL TEST-11 ANSWER KEY AND EXPLANATION 1. Answer

NEO IAS 0484-3190310, 9446331522, 9446334122 Page 60 www.neoias.com | www.youtube.com/neoias | www.facebook.com/neoias | www.twitter.com/neoias

DMH (Dhara Mustard Hybrid)-11 is a genetically modified (GM) mustard hybrid. DMH-11 was developed by a team of scientists at Delhi University led by former vice-chancellor Deepak Pental under a government-funded project. Hybrids are normally obtained by crossing two genetically diverse plants from the same species. The first-generation offspring resulting from it has higher yields than what either of the parents is individually capable of giving. But there is no natural hybridisation system in mustard, unlike in, say, cotton, maize or tomato. This is because its flowers contain both the female (pistil) and male (stamen) reproductive organs, making the plant naturally self-pollinating. To the extent that the egg cells of one plant cannot be fertilised by the pollen discharged from the stamen of another, it restricts the scope for developing hybrids.

28. Answer (c) Explanation Tianntong-01 is China‘s satellite for mobile communication. It was designed by China Academy of Space Technology and its ground service will be operated by China Telecom. It is the first satellite of China's home-made satellite mobile telecom system, and a key part of the country's space information infrastructure. It will establish a mobile network with ground facilities, providing services for China, the Middle East, Africa and other areas.

29. Answer (d) Explanation Agni-V is India's most potent missile yet and can cover a range of over 5000 kilometres in any direction. It is also a nuclear-capable missile, which means that it can carry a nuclear warhead and cover a long stretch equipped with that nuclear devise. Geographically, the missile's cover space ranges from all of China in the East to all of Europe in the West. The missile is able to travel faster than a bullet and can carry with it at least a 1000 kilogramme of nuclear weapon. However, the missile can only be launched on the direct orders of the Prime Minister of India and no one else. India has now joined an elite club with only Russia, China, US and France currently possessing the capability to operate an intercontinental ballistic missile. The indigenously developed, intercontinental, surface to surface, nuclear capable ballistic missile is unlike other missiles in the Agni series. Agni-V is the most advanced system so far, which has various new technologies incorporated in it in terms of navigation and guidance, warhead and engine. Agni-V was tested on January 31, 2015 in Odisha. Being successful in the previous two test launches in 2011 and 2012 Agni-V has now made India a missile superpower.

30. Answer (b) Explanation Proxima b is an exoplanet orbiting within the habitable zone of the closest star to the Sun—the red dwarf star Proxima Centauri, which is in a triple star system. It is located about 4.2 light-years (1.3 parsecs, 40 trillion km, or 25 trillion miles) from Earth in the constellation of Centaurus, making it the closest known exoplanet to the Solar System.

31. Answer (b) Explanation DNA index system uses the latest DNA technology tool developed by integen X, USA known as RapidHIT DNA system. It allows generation of DNA profiles from live samples such as buccal swabs, blood strains, saliva etc., within less than two hours as the currently available technologies takes at least two days or more. It aims to make Andhra Pradesh a crime free state in India through the usage of latest technology in forensic labs.

32. Answer (c) Explanation Andrographolid drug which has ability to inhibit tumor growth. It was developed by national center for cell science (NCCS). To test the efficacy of Andrographolide drug candidate, which is being tested in several clinical trials, in inhibiting tumour growth, the researchers injected the compound into the tumour bearing mice. Two different drug dosages were used. The compound at 50 mg/kg body weight dosage was able to inhibit primary tumour (melanoma) growth by about two-third at the end of 24 days. In the case of a higher dosage of 150 mg/kg body weight, the compound was found to inhibit melanoma growth by more than two-third at the end of 24 days.

33. Answer (a) Explanation PHYSICAL CHANGE A physical change is any change in matter that involves the substance going from one physical state to another. The reference to a physical state involves solids, liquids, and gases. When a substance undergoes a physical change it does not lose its original properties when changing from one phase to another. The types of physical changes can vary. A substance can go from a solid to a liquid, a liquid to a gas, a gas to a liquid, a liquid to a solid, a solid to a gas, or a gas to a solid. Examples of Physical changes:

Melting of icecube

Breaking a glass

Crushing a can

Boiling water etc

Page 61: PRELIMS MODEL TEST-11 ANSWER KEY AND EXPLANATION. PMT-11 to 15 Explanation.pdfTARGET PRELIMS 2017 GENERAL STUDIES PAPER - 1 PRELIMS MODEL TEST-11 ANSWER KEY AND EXPLANATION 1. Answer

NEO IAS 0484-3190310, 9446331522, 9446334122 Page 61 www.neoias.com | www.youtube.com/neoias | www.facebook.com/neoias | www.twitter.com/neoias

CHEMICAL CHANGE Chemical changes involve chemical reactions and the creation of new products. Typically, a chemical change is irreversible. In contrast, physical changes do not form new products and are reversible. Examples of chemical change

burning wood

souring milk

mixing acid and base

digesting food

cooking an egg etc

34. Answer (a) Explanation Xmu-Mp-1 is a small molecule which can regenerate the tissue and could make organ transplant much easier in future. It was developed by Xiamen University in China. It can promote repair and regeneration in the liver, intestines and skin. In the future, the pills may do away the need for organ transplant or complex biomaterial and cell therapies. (The XMU-MP-1 has proven to inhibit the activity of MST1/2, the central component of this pathway and promote cell growth in four different mouse models of acute and chronic injuries, including acetaminophen-induced injury, which is a common cause of liver failure worldwide).

35. Answer (b) Explanation Lymphatic filariasis Lymphatic filariasis is infection with the filarial worms, Wuchereria bancrofti, Brugia malayi or B. timori. These parasites are transmitted to humans through the bite of an infected mosquito and develop into adult worms in the lymphatic vessels, causing severe damage and swelling (lymphoedema). It is not an eradicated disease from India. Hepatitis B It is a viral infection that attacks the liver and can cause both acute and chronic disease. It is transmitted through exposure to infective blood, semen, and other body fluids. It is not an eradicated disease from India. Smallpox Smallpox is a very serious illness caused by a virus called the variola virus. It is one of the eradicated diseases from India. Smallpox gets its name from the pus-filled blisters (or pocks) that form during the illness.

36. Answer (b) Explanation Virtual black hole is an experiment based on sound, rather than light. It was developed by Jeff Steinhauer, a physicist at the Israel Institute of Technology built a virtual black hole in the lab. It aims to prove that Hawking‘s theory of radiation emanating from black holes is correct. The quantum effects of Hawking radiation in his lab as part of a virtual black hole-which, if proven to be true, will be the first time it has ever been achieved. For many years, scientists believed that nothing could ever escape from a black hole-not even light. However, in 1974, Stephen Hawking suggested particles, that are now called Hawking radiation, could escape black holes.

37. Answer (a) Explanation Maglev, which is derived from the term 'magnetic levitation', a transport method that employs magnetic levitation to move the vehicle without touching the ground. With maglev technology, a vehicle travels along a 'guideway' using magnets to create both lift and propulsion. It can run at a top speed of 500 km an hour. They are less expensive to operate and maintain, because the absence of rolling friction means that parts do not wear out quickly. Maglev trains produce no air pollution during operation, because no fuel is being burned. As there is no friction, the train produce almost no sound making the ride smoother for the passengers. Maglev systems can operate on higher ascending grades compared to traditional railroads, reducing the need of land modification in order to accommodate the tracks. Magnets made from ferrite (an iron compound) or alnico (alloys of iron, aluminium, nickel, cobalt, and copper) produce a stronger magnetic field than ordinary magnets which helps lift and channelize the train cars over the directed 'guideway'. Maglev trains are in operation in China, Japan, Germany and South Korea.

38. Answer (c) Explanation DNA sequencing is the process of determining the sequence of nucleotides in a piece of DNA. It is used to identify the criminals by finding some proof in the crime scene in the form of hair, nail, skin or blood samples. It is used to determine the paternity of the child. It can also be used to detect the gene which are associated with heredity or acquired disease.

39. Answer (c) Explanation Regenerative medicine is that branch of medicine that develops methods to regrow, repair or replace damaged or diseased cells, organs or tissues that have been damaged by disease, trauma, or congenital issues, vs. the current

Page 62: PRELIMS MODEL TEST-11 ANSWER KEY AND EXPLANATION. PMT-11 to 15 Explanation.pdfTARGET PRELIMS 2017 GENERAL STUDIES PAPER - 1 PRELIMS MODEL TEST-11 ANSWER KEY AND EXPLANATION 1. Answer

NEO IAS 0484-3190310, 9446331522, 9446334122 Page 62 www.neoias.com | www.youtube.com/neoias | www.facebook.com/neoias | www.twitter.com/neoias

clinical strategy that focuses primarily on treating the symptoms. Regenerative medicine includes the generation and use of therapeutic stem cells, tissue engineering and the production of artificial organs.

40. Answer (c) Explanation Psyche is the largest metallic asteroid in our solar system. Psyche is about 300 kilometre across and is made of almost pure nickel-iron metal. It is located in the asteroid belt. It is thought to be the remnant core of a budding planet that was mostly destroyed by impacts billions of years ago. Recently, scientists have detected the presence of water on Psyche.

41. Answer (d) Explanation Halophyte is a salt tolerant plant. Plants such as these are important in the context of increasing salinisation of land, one of the adverse effects of climate change. Halophytes tolerate salinity and could grow in saline affected regions. Their seeds contain high oil (30 per cent) and protein (35 per cent) like soyabeans and other oilseed crops and the salt content is less than three per cent. This is probably the first such garden in the world and could be called a global garden - very important in the aspect of anticipatory research. The potential of halophyte as food for people, fodder for livestock, bio fuel and for ornamental purposes was being explored and "could reap rich rewards in research for climate change.

42. Answer (a) Explanation It is an exoplanet, is not within the star‘s habitable zone, but its short orbital period of 8.7 days and the luminosity of its star make it an attractive candidate for investigating its atmospheric composition. (The star, GJ 536, is a red dwarf which is quite cool and near to our Sun.)

43. Answer (a) Explanation Japanese encephalitis virus (JEV) is a flavivirus related to dengue, yellow fever and West Nile viruses, and is spread by mosquitoes. Athlete's foot, known medically as tinea pedis, is a common skin infection of the feet caused by fungus. Signs and symptoms often include itching, scaling, and redness. Leprosy is a chronic disease caused by a bacillus, Mycobacterium leprae. The disease mainly affects the skin, the peripheral nerves, mucosa of the upper respiratory tract, and also the eyes. Dengue is a mosquito-borne viral infection. The infection causes flu-like illness, and occasionally develops into a potentially lethal complication called severe dengue. Measles, also known as rubeola or morbilli, is a highly infectious illness caused by the rubeola virus.

44. Answer (c) Explanation The EM Drive, or Electromagnetic Drive, is a propulsion system first proposed by British inventor Roger Shawyer back in 1999. It was tested and developed by NASA‘s Advanced Propulsion Physics Research Laboratory, informally known as Eagleworks. Em Drive generates thrust by bouncing around electromagnetic energy (in this case, microwave photons) in a closed, cone-shaped chamber. As those photons collide with the chamber‘s walls, they somehow propel the device forward, despite the fact that nothing is released from the chamber. By contrast, ion drives now in use on some NASA spacecraft create thrust by ionizing a propellant, often xenon gas, and shooting out beams of charged atoms. It defies Newton's third law, which states that every action has an equal and opposite reaction. According to the law, for a system to produce thrust, it has to push something out the other way. The EM Drive doesn't do this.

45. Answer (a) Explanation DOTS or Directly Observed Treatment Short course is the internationally recommended strategy for TB control that has been recognized as a highly efficient and cost-effective strategy. It was launched by world health organization. DOTS comprises five components. 1. Sustained political and financial commitment. TB can be cured and the epidemic reversed if adequate

resources and administrative support for TB control are provided 2. Diagnosis by quality ensured sputum-smear microscopy. Chest symptomatics examined this way helps to

reliably find infectious patients 3. Standardized short-course anti-TB treatment (SCC) given under direct and supportive observation (DOT).

Helps to ensure the right drugs are taken at the right time for the full duration of treatment. 4. A regular, uninterrupted supply of high quality anti-TB drugs. Ensures that a credible national TB programme

does not have to turn anyone away. 5. Standardized recording and reporting. Helps to keep track of each individual patient and to monitor overall

programme performance

Page 63: PRELIMS MODEL TEST-11 ANSWER KEY AND EXPLANATION. PMT-11 to 15 Explanation.pdfTARGET PRELIMS 2017 GENERAL STUDIES PAPER - 1 PRELIMS MODEL TEST-11 ANSWER KEY AND EXPLANATION 1. Answer

NEO IAS 0484-3190310, 9446331522, 9446334122 Page 63 www.neoias.com | www.youtube.com/neoias | www.facebook.com/neoias | www.twitter.com/neoias

46. Answer (c) Explanation The Bio-Indicator Lidar Instrument, or BILI is a fluorescence-based lidar, a type of remote-sensing instrument similar to radar in principle and operation. Blagojevic, who formerly worked for Science and Engineering Services, LLC, that developed the sensor, has applied the technology to create an instrument prototype, proving in testing that the same remote-sensing technology used to identify bio-hazards in public places also could be effective at detecting organic bio-signatures on Mars. Instead of using radio waves, however, lidar instruments use light to detect and ultimately analyse the composition of particles in the atmosphere.

47. Answer (b) Explanation INS Chennai is the third indigenously designed guided missile destroyer in a kolkata class. It is armed with supersonic surface-to-surface BrahMos missiles and Barak-8 long range surface-to-air missiles.

48. Answer (a) Explanation Robot lakshmi is India's first banking robot. It was launched by Kumbakonam-based City Union Bank. It is the artificial intelligence powered robot. If a customer wants to know his bank account details or transaction history, the robot can flash the answer on its display. Sensitive financial information like account details are displayed discreetly on the robot's screen and not voiced. Lakshmi, who currently speaks in English, gestures, turns around and engages in a very life-like manner in conversations. Since its artificial intelligence, the robot is constantly learning from customers - the more interactions it has with customers the better it gets. It can give real time updates of foreign exchange movement, current interest rates at banks for different asset classes like personal, educational, two-wheeler and home loans, possible charges on withdrawals or deposits.

49. Answer (c) Explanation The European Organization for Nuclear Research, known as CERN is a European research organization that operates the largest particle physics laboratory in the world. Recently, India got associate membership in CERN. As an Associate member, India can participate in all experiments. India can choose the experiments where India wants to participate. When India was not an Associate member, India could data only from those experiments where India was participating. Currently, CERN has 22 member states. Besides India, Turkey, Pakistan, Ukraine are Associate members and Serbia and Cyprus are associate members in the pre-stage to membership.

50. Answer (a) Explanation Coradia iLint is the world's first zero-emission hydrogen passenger train. It was developed by Alstom (a French company). The train operates using a hydrogen fuel tank, stored on the roof of the vehicle that in turn powers a fuel cell to produce electrical energy, its only emission being steam and condensed water while operating with a low level of noise. Running on hydrogen power supply technology, the train has no carbon footprints and does not pollute the air with harmful emissions. It can reduce greenhouse gas emissions from transportation. Because of its much larger fuel capacity, the train does not have to confront the same problem that hydro-cars face with refueling.

51. Answer (b) Explanation Red soil develops on crystalline igneous rocks in areas of low rainfall in the eastern and southern part of the Deccan Plateau. By and large, the Red soils are poor in lime, magnesia, phosphates, nitrogen and humus and potash. The red soils are porous and they have friable structure. They have low water holding capacity. The soil develops a reddish colour due to the presence of ferric oxide.

52. Answer (a) Explanation Wind breaks also reduces wind velocity and protects arid regions from wind erosion. Shifting cultivation is not a soil conservation method, rather it results in soil degradation and erosion. If ploughing is done at right angles to the hill slope, following the natural contours of the hill, the ridges and furrows break the flow of water down the hill. This prevents excessive soil loss as gullies are less likely to develop and also reduce run-off so that plants receive more water. Thus by growing crops in contour pattern, plants can absorb much of the rain water and erosion is minimized. Shifting cultivation is not a soil conservation method rather it results in soil erosion and should be checked.

53. Answer (d) Explanation

Page 64: PRELIMS MODEL TEST-11 ANSWER KEY AND EXPLANATION. PMT-11 to 15 Explanation.pdfTARGET PRELIMS 2017 GENERAL STUDIES PAPER - 1 PRELIMS MODEL TEST-11 ANSWER KEY AND EXPLANATION 1. Answer

NEO IAS 0484-3190310, 9446331522, 9446334122 Page 64 www.neoias.com | www.youtube.com/neoias | www.facebook.com/neoias | www.twitter.com/neoias

54. Answer (b) Explanation Bellandur Lake is a lake in the suburb of Bellandur in the southeast of the city of Bangalore and is the largest lake in the city. It is a part of Bellandur drainage system that drains the southern and the south-eastern parts of the city. It is currently highly polluted, and was frequently in news.

55. Answer (b) Explanation Kolleru Lake is one of the largest freshwater lakes in India located in state of Andhra Pradesh. With the onset of 2017 summer, a major portion of the Kolleru Lake dried up. Chilika Lake - Odisha - Nalaban Bird Sanctuary. Pulicat Lake is the second largest brackish water lake or lagoon in India, after Chilika Lake. It straddles the border of Andhra Pradesh and Tamil Nadu states with over 40% of it in Andhra Pradesh and 60% in Tamil Nadu situated on the Coromandal Coast in South India. The lake encompasses the Pulicat Lake Bird Sanctuary. The barrier island of Sriharikota separates the lake from the Bay of Bengal and is home to the Satish Dhawan Space Centre.

56. Answer (c) Explanation Réunion is an island and region of France in the Indian Ocean, east of Madagascar. Thengar Char is an island in Bangladesh which is located on the Bay of Bengal. The island was formed with Himalayan silt in 2006. It is under water from June to September annually because of the monsoon, and it has no flood fences. On January 2017, the Bangladeshi government ordered the resettlement of Rohingya refugees to the island, which is seen as a " human rights and humanitarian disaster in the making". Havelock is the largest of the islands that comprise Ritchie's Archipelago, a chain of islands to the east of Great Andaman in the Andaman Islands. It belongs to the South Andaman administrative district, part of the Indian

union territory of Andaman and Nicobar Islands.

57. Answer (d) Explanation Western most – Gujarat Southern most – Tamil Nadu

58. Answer (d) Explanation Myanmar is situated in Southeast Asia and is bordered on the north and north-east by China, on the east and south-east by Laos and Thailand, on the south by the Andaman Sea and the Bay of Bengal and on the west by

Bangladesh and India.

59. Answer (d) Explanation Indian standard Meridian passes through UP, MP, Chhattisgarh, Orissa and Andhra Pradesh. With the forest cover of 77,700 square kilometre area, Madhya Pradesh is one of the states with largest forest cover (according to Indian

State of Forest Report 2011).

60. Answer (a) Explanation

Page 65: PRELIMS MODEL TEST-11 ANSWER KEY AND EXPLANATION. PMT-11 to 15 Explanation.pdfTARGET PRELIMS 2017 GENERAL STUDIES PAPER - 1 PRELIMS MODEL TEST-11 ANSWER KEY AND EXPLANATION 1. Answer

NEO IAS 0484-3190310, 9446331522, 9446334122 Page 65 www.neoias.com | www.youtube.com/neoias | www.facebook.com/neoias | www.twitter.com/neoias

The Reunion Island is above a hotspot in the Earth's crust. The Piton de la Fournaise, a shield volcano on the eastern end of Réunion Island, rises more than 2,631 metres (8,632 ft) above sea level and is sometimes called a sister to Hawaiian volcanoes because of the similarity of climate and volcanic nature. It has erupted more than 100 times since 1640, most recently erupting on September 2016. The collision of Indo-Australian Plate with Eurasia lifted the Himalayas and most of the Indonesian islands, and created a long arc of highlands and islands, which includes the Arakan Yoma range of Burma, the Andaman and Nicobar Islands, and the islands off the west coast of Sumatra. Lakshadweep is an archipelago of twelve atolls (made of corals), three reefs and five submerged banks, with a total of about thirty-nine islands and islets. The Maldives consists of 1,192 coral islands grouped in a double chain of 26 atolls.

61. Answer (c) Explanation The creation and destruction of ozone gas is a gradual and continuous natural process. The oxygen molecules are broken up or separated in the atmospheric layer between the altitudes of 80 to 100 km. Oxygen breaks up into two separate oxygen atoms.

O2 O + O These separated oxygen atoms (O) are combined with oxygen molecules (O2) and thus ozone (O3) is formed.

O2 + O O3 (ozone) The ozone is transformed back into oxygen by further collision of ozone with monoatomic oxygen.

Ozone (O3) + O Oxygen (O2) + Oxygen (O2) The process of the formation of ozone may be described as the production of ozone by the actions of ultraviolet rays on ordinary oxygen atoms by the photo chemical process which is responsible for the constant transformation of oxygen to ozone and from ozone back to oxygen. Thus, the constant formation of production and destruction of ozone is a natural process which maintains an approximate equilibrium level of ozone in the atmosphere if not inferred by human activities. The controlling and phasing out ozone depleting substances is associated with Montreal protocol.

62. Answer (b) Explanation During the day, the earth is heated by the sun. If skies are clear, more heat reaches the earth's surface. This leads to warmer temperatures. However, if skies are cloudy, some of the sun's rays are reflected off the cloud droplets back into space. Therefore, less of the sun's energy will reach the earth's surface, which causes the earth to heat up more slowly. This leads to cooler temperatures. Thus, due to clouds reflecting more intense solar energy, winter would be much cooler. Further, if clouds are present, some of the heat emitted from the earth's surface is trapped by the clouds and reemitted back towards the earth. As a result, temperatures decrease more slowly than if the skies were clear. Thus, cloud cover reduces diurnal range of the temperature.

63. Answer (c) Explanation Stalactites and stalagmites are depositional landforms created by groundwater. Typical landforms produced by the action of groundwater through the processes of solution and deposition is called Karst topography

64. Answer (c) Explanation During its revolution around the sun, the earth is farthest from the sun (152 million km) on 4th July. This position of the earth is called aphelion. On 3rd January, the earth is the nearest to the sun (147 million km). This position is called perihelion.

65. Answer (c) Explanation

Page 66: PRELIMS MODEL TEST-11 ANSWER KEY AND EXPLANATION. PMT-11 to 15 Explanation.pdfTARGET PRELIMS 2017 GENERAL STUDIES PAPER - 1 PRELIMS MODEL TEST-11 ANSWER KEY AND EXPLANATION 1. Answer

NEO IAS 0484-3190310, 9446331522, 9446334122 Page 66 www.neoias.com | www.youtube.com/neoias | www.facebook.com/neoias | www.twitter.com/neoias

66. Answer (b) Explanation In troposphere, temperature decreases with altitude at normal lapse rate.

67. Answer (a) Explanation Bordering countries of Turkey are Greece, Bulgaria, Georgia, Armenia, Azerbaijan, Iran, Iraq, and Syria.

68. Answer (d) Explanation Loktak Lake is the largest freshwater lake in Northeast India, and is famous for the phumdis (heterogeneous mass of vegetation, soil, and organic matter at various stages of decomposition) floating over it. Keibul Lamjao is the only floating national park in the world. It is located near Moirang in Manipur state, India. The Keibul Lamjao National Park is the last natural refuge of the endangered Sangai (state animal of Manipur) or Manipur brown-antlered deer.

69. Answer (b) Explanation The Circum–Pacific belt, also known as the ‗volcanic zones of the convergent oceanic plate margins‘, includes the volcanoes of the eastern and western coastal areas of the Pacific Ocean (or the western coastal margins of the North and South Americas and the eastern coastal margins of Asia), of island arcs and festoons off the east coast of Asia and of the volcanic islands scattered over the Pacific Ocean. This volcanic belt is also called as the fire girdle of the Pacific or the fire ring of the Pacific. Moreover, world‘s distribution of earthquakes coincides very closely with that of volcanoes. As many as 70 per cent of earthquakes occur in the Circum-Pacific belt. Due

to the occurrence of undersea earthquakes as well as volcanoes, Circum-Pacific belt is also prone to Tsunamis

Page 67: PRELIMS MODEL TEST-11 ANSWER KEY AND EXPLANATION. PMT-11 to 15 Explanation.pdfTARGET PRELIMS 2017 GENERAL STUDIES PAPER - 1 PRELIMS MODEL TEST-11 ANSWER KEY AND EXPLANATION 1. Answer

NEO IAS 0484-3190310, 9446331522, 9446334122 Page 67 www.neoias.com | www.youtube.com/neoias | www.facebook.com/neoias | www.twitter.com/neoias

70. Answer (b) Explanation Due to revolution of Earth on its tilted axis, North pole and South Pole experiences 6 months of daylight and darkness alternatively.

71. Answer (d) Explanation The salinity of water in the surface layer of oceans depend mainly on evaporation and precipitation. The lower latitudinal water is always identified with higher salinity levels due to evaporation. And in the regions with higher amount of precipitation salinity level are less. Salinity is high in Red Sea as river water influx is low.

72. Answer (d) Explanation Earthquake prediction is concerned with forecasting the occurrence of an earthquake of a particular intensity over a specific locality within a specific time limit. However, no flawless technique has been developed to accurately predict the earthquakes till date.

73. Answer (d) Explanation In the course of a year, the earth‘s revolution round the sun with its axis inclined at 66½

0 to the plane of the

ecliptic changes the apparent altitude of the midday sun.

74. Answer (d) Explanation A polar vortex is an upper level low-pressure area lying near the Earth's pole. There are two polar vortices in the Earth's atmosphere, which overlie the North, and South Poles. Each polar vortex is a persistent, large-scale, low pressure zone that rotates counter-clockwise at the North Pole (called a cyclone), and clockwise at the South Pole. The bases of the two polar vortices are located in the middle and upper troposphere and extend into the stratosphere. Ozone depletion occurs within the polar vortices – particularly over the Southern Hemisphere – reaching a maximum depletion in the spring.

75. Answer (c) Explanation Ram Sethu is a chain of limestone shoals, between Pamban Island, also known as Rameswaram Island, off the south-eastern coast of Tamil Nadu, India, and Mannar Island, off the north-western coast of Srilanka. The bridge is 50 km (30 mi) long and separates the Gulf of Mannar from the Palk Strait.

76. Answer (c) Explanation The Paris Agreement‘s central aim is to strengthen the global response to the threat of climate change by keeping a global temperature rise this century well below 2 degrees Celsius above pre-industrial levels and to pursue efforts to limit the temperature increase even further to 1.5 degrees Celsius. Additionally, the agreement aims to strengthen the ability of countries to deal with the impacts of climate change. On 5 October 2016, the threshold for entry into force of the Paris Agreement was achieved. The Paris Agreement entered into force on 4 November 2016. the Paris Agreement doesn‘t deal with emissions from aviation sector rather it is dealt by Carbon Offset and Reduction Scheme for International Aviation (CORSIA).

77. Answer (d) Explanation The agreement establishes a framework for global stocktake to assess the collective action towards achieving the long-term goals mentioned in the Agreement. Every five years there will be an assessment of how the emission reduction actions of all countries collectively are stacking up against the goal of keeping temperatures under control. As per article 4 of Paris Agreement, ‗Each Party shall prepare, communicate and maintain successive nationally determined contributions that it intends to achieve. Parties shall pursue domestic mitigation measures, with the aim of achieving the objectives of such contributions. Each Party's successive nationally determined contribution will represent a progression beyond the Party's then current nationally determined contribution and reflect its highest possible ambition, reflecting its common but differentiated responsibilities and respective capabilities, in the light of different national circumstances‘. The major commitments India made through its NDC are as following: - 1. To reduce the emissions intensity of its GDP by 33 to 35 percent by 2030 from 2005 level. 2. To achieve about 40 percent cumulative electric power installed capacity from non-fossil fuel based

energy resources by 2030 with the help of transfer of technology and low cost international finance including from Green Climate Fund (GCF).

3. To create an additional carbon sink of 2.5 to 3 billion tonnes of CO2 equivalent through additional forest and tree cover by 2030.

78. Answer (d)

Page 68: PRELIMS MODEL TEST-11 ANSWER KEY AND EXPLANATION. PMT-11 to 15 Explanation.pdfTARGET PRELIMS 2017 GENERAL STUDIES PAPER - 1 PRELIMS MODEL TEST-11 ANSWER KEY AND EXPLANATION 1. Answer

NEO IAS 0484-3190310, 9446331522, 9446334122 Page 68 www.neoias.com | www.youtube.com/neoias | www.facebook.com/neoias | www.twitter.com/neoias

Explanation The decisions on REDD+ enumerate five "eligible activities" that developing countries may implement to reduce emissions and enhance removals of greenhouse gases: (a) Reducing emissions from deforestation. (b) Reducing emissions from forest degradation. (c) Conservation of forest carbon stocks. (d) Sustainable management of forests. (e) Enhancement of forest carbon stocks.

79. Answer (d) Explanation Terracing across the hill slopes is a very effective and one of the oldest methods of soil conservation. Hill slopes are cut into a number of terraces having horizontal top and steep slopes on the back and front. It divides the hill slope into numerous small slopes, checks the flow of water, promotes absorption of water by soil and saves soil from erosion.

80. Answer (d) Explanation Soil formation depends upon the texture and structure as well as the mineral and chemical composition of the parent materials. The influence of topography is felt through the amount of exposure of a surface covered by parent materials to sunlight and the amount of surface and sub-surface drainage over and through the parent materials. Climate is an important active factor in soil formation. The climatic elements involved in soil development are: (i) moisture in terms of its intensity, frequency and duration of precipitation - evaporation and humidity; (ii) temperature in terms of seasonal and diurnal variations. Geomorphic agents (running water, wind etc.) transport materials and thus influences soil formation.

81. Answer (d) Explanation Coastal Regulation Zone is a tool used for regulating the development and human activities along the coasts. The Ministry of Environment and Forests had issued the Coastal Regulation Zone (CRZ) Notification on 19.2.1991 under the Environment (Protection) Act, 1986, with the aim to provide comprehensive measures for the protection and conservation of coastal environment. The Coastal Regulation Zone (CRZ) notification of 1991 has been replaced recently in January 2011 by the latest CRZ notification of the Ministry of Environment and Forests (MoEF). The main objectives of the Coastal Regulation Zone Notification, 2011:

To ensure livelihood security to the fishing communities and other local communities living in the coastal areas;

To conserve and protect coastal stretches and;

To promote development in a sustainable manner based on scientific principles, taking into account the dangers of natural hazards in the coastal areas and sea level rise due to global warming.

As per the notification, the coastal land up to 500m from the High Tide Line (HTL) and a stage of 100m along banks of creeks, estuaries, backwater and rivers subject to tidal fluctuations, is called the Coastal Regulation Zone(CRZ). CRZ along the country has been placed in four categories.

82. Answer (d) Explanation The country is endowed with vast resources of a variety of minerals and has favourable geological settings for many others. However, vast geographical areas are still to be explored to the desired levels. The NMEP primarily aims at accelerating the exploration activity in the country through enhanced participation of the private sector. There is a need for comprehensive mineral exploration of the country to uncover its full mineral potential so as to put the nation's mineral resources (non-fuel and non-coal) to best use and thereby maximize sectoral contribution to the Indian economy. The policy emphasizes on making available baseline geoscientific data of world standards in the public domain, quality research in a public-private partnership, special initiatives for search of deep-seated and concealed deposits, quick aero-geophysical surveys of the country, and creation of a dedicated geoscience database etc. SALIENT FEATURES

The Ministry of Mines will carry out auctioning of identified exploration blocks for exploration by private sector on revenue sharing basis in case their exploration leads to auctionable resources. The revenue will be borne by the successful bidder of those auctionable blocks. If the explorer agencies do not discover any auctionable resources, their exploration expenditure will be reimbursed on normative cost basis.

Creation of baseline geoscientific data as a public good for open dissemination free of charge.

Government will carry out a National Aerogeophysical Program for acquiring state-of-the-art baseline data for targeting concealed mineral deposits.

Page 69: PRELIMS MODEL TEST-11 ANSWER KEY AND EXPLANATION. PMT-11 to 15 Explanation.pdfTARGET PRELIMS 2017 GENERAL STUDIES PAPER - 1 PRELIMS MODEL TEST-11 ANSWER KEY AND EXPLANATION 1. Answer

NEO IAS 0484-3190310, 9446331522, 9446334122 Page 69 www.neoias.com | www.youtube.com/neoias | www.facebook.com/neoias | www.twitter.com/neoias

A National Geoscientific Data Repository is proposed to be set up to collate all baseline and mineral exploration information generated by various central & state government agencies and mineral concession holders, and to maintain these on geospatial database.

Government proposes to establish a not-for-profit autonomous institution that will be known as the National Centre for Mineral Targeting (NCMT) in collaboration with scientific and research bodies, universities and industry for scientific and technological research to address the mineral exploration challenges in the country.

Provisions for inviting private investment in exploration through attractive revenue sharing models.

On the lines of UNCOVER project of Australia, the government intends to launch a special initiative to probe deep-seated/ concealed minerals deposits in the country in collaboration with National Geophysical Research Institute and the proposed NCMT and Geoscience Australia.

83. Answer (c) Explanation Indian standard Meridian (82

030' E meridian) passes through UP, MP, Chhattisgarh, Orissa and Andhra Pradesh.

84. Answer (d) Explanation A cloudburst is a sudden aggressive downpour within the radius of a couple of kilometres. Though, cloudbursts usually do not last for more than few minutes, they are capable of flooding the entire area. Rainfall from a cloudburst is usually equal to or greater than 100 mm per hour. Cloudbursts are generally associated with thunderstorms. However, the above definition has been given by a particular school of thought. In reality, there is no specific amount of rain associated with a cloudburst, either in time or duration. How does a cloudburst happen? Whenever vertically formed clouds fully develop, moving very slow, most of the water content available in the clouds come down as a downpour at a particular area. Cloudbursts generally descend from very high clouds, sometimes with tops above 15 kilometers. The windward side of mountains are generally conducive for generating thunderclouds with huge updrafts. Effect of cloudbursts on hills and plains The catastrophic nature of cloudbursts differ on the virtue of terrain. In the hills, large volume of water keeps getting momentum as it flows in gushes. On its way, it demolishes everything and gravity of the situation increases due to landslides, mudslides, etc. On the other hand, cloudbursts in the plains only leads to waterlogging and inundation.

85. Answer (b) Explanation Perched on the Himalayas at an altitude of nearly 13,500 feet, Himansh, the glaciological research facility is considered to be the highest point from where an Indian glacier research facility is functioning. The research lab, established by the National Centre for Antarctic and Ocean Research (NCAOR), has automatic weather stations, ground penetrating radars, geodetic GPS systems and other sophisticated facilities to quantify glaciers in the upper Indus basin in Himachal Pradesh and their contribution to river discharge.

86. Answer (a) Explanation A Supermoon is a new or full moon closely coinciding with perigee – the moon‘s closest point to Earth in its monthly orbit.

87. Answer (b) Explanation SOME FAMOUS LOCAL STORMS OF HOT WEATHER SEASON (i) Mango Shower: Towards the end of summer, there are pre-monsoon showers which are a common

phenomenon in Kerala and coastal areas of Karnataka. Locally, they are known as mango showers since they help in the early ripening of mangoes.

(ii) Blossom Shower: With this shower, coffee flowers blossom in Kerala and nearby areas. (iii) Nor Westers: These are dreaded evening thunderstorms in Bengal and Assam. Their notorious nature can

be understood from the local nomenclature of ‗Kalbaisakhi‘, a calamity of the month of Baisakh. These showers are useful for tea, jute and rice cultivation. In Assam, these storms are known as ―Bardoli Chheerha‖.

(iv) Loo: Hot, dry and oppressing winds blowing in the Northern plains from Punjab to Bihar with higher intensity between Delhi and Patna.

88. Answer (b) Explanation Reasons for landslides in Himalayas

Himalayas are tectonically active

They are mostly made up of sedimentary rock and unconsolidated and semi-consolidated deposits

The slopes are very steep

Presence of several big thrusts, viz., main Boundary thrust, main central thrust etc.

Page 70: PRELIMS MODEL TEST-11 ANSWER KEY AND EXPLANATION. PMT-11 to 15 Explanation.pdfTARGET PRELIMS 2017 GENERAL STUDIES PAPER - 1 PRELIMS MODEL TEST-11 ANSWER KEY AND EXPLANATION 1. Answer

NEO IAS 0484-3190310, 9446331522, 9446334122 Page 70 www.neoias.com | www.youtube.com/neoias | www.facebook.com/neoias | www.twitter.com/neoias

Monsoon

89. Answer (d) Explanation Compared to the Himalayas, the Nilgiris bordering Tamil Nadu, Karnataka, Kerala and the Western Ghats along the west coast are relatively tectonically stable and are mostly made up of very hard rocks; but, still, debris avalanches and landslides occur though not as frequently as in the Himalayas, in these hills. Because, many slopes are steeper with almost vertical cliffs and escarpments in the Western Ghats and Nilgiris. Mechanical weathering due to temperature changes and ranges is pronounced. They receive heavy amounts of rainfall over short periods. So, there is almost direct rock fall quite frequently in these places along with landslides and debris avalanches. Presence of hard rocks doesn‘t favour landslides.

90. Answer (c) Explanation Tropic of Cancer passes through following Countries: 1. Mexico, 2. Bahamas, 3. Western Sahara, 4. Mauritania, 5. Mali, 6. Algeria, 7. Nigeria, 8. Libya, 9. Egypt, 10. Saudi Arabia, 11. U.A.E, 12. Oman, 13. India, 14. Bangladesh, 15. Myanmar, 16. China, 17. Taiwan. The mid-day Sun is exactly overhead only once a year, along tropic of Cancer and tropic of Capricorn. Along equator, mid-day sun will be overhead twice a year. Brazil life both on equator and tropic of Capricorn.

91. Answer (d) Explanation

92. Answer (c) Explanation The southwest monsoon is characterized by the presence of strong westerly winds in the lower troposphere (below 5 km) and very strong easterly winds in the upper troposphere (above 9 km). This results in large vertical wind shear. Strong vertical wind shear inhibits cyclone development.

93. Answer (a) Explanation The original relief of the Himalayas has been extensively modified by the incessant work of the running water and the glaciers. Besides the awe inspiring gorges of the major rivers, which bear testimony to their erosive power, evidence of the vigorous glacial action is scattered all over the Himalayas. This evidence is found in the features noted below: (a) Large blocks of rocks transported from the high summits of the Himalayan range over long distances; (b) Karewas of the Kashmir Valley comprising thick deposits of glacial clay and other materials embedded with

moraines; (c) Numerous U-shaped and hanging valleys found at elevations much lower than the position of the existing

glaciers; and (d) Extensive moraine deposits along the river valleys which in many areas form well developed terraces.

Particularly on the flanks of the surrounding mountain ranges in the Kashmir Valley.

Page 71: PRELIMS MODEL TEST-11 ANSWER KEY AND EXPLANATION. PMT-11 to 15 Explanation.pdfTARGET PRELIMS 2017 GENERAL STUDIES PAPER - 1 PRELIMS MODEL TEST-11 ANSWER KEY AND EXPLANATION 1. Answer

NEO IAS 0484-3190310, 9446331522, 9446334122 Page 71 www.neoias.com | www.youtube.com/neoias | www.facebook.com/neoias | www.twitter.com/neoias

94. Answer (d) Explanation A thin, whitish sheet of cloud covering the whole sky and giving it a milky appearance is called cirrostratus. These clouds commonly produce a halo around the sun and the moon. Thunderstorms are well developed cumulonimbus clouds producing thunder and lightening. Asperitas (from the Latin meaning wave-like and roughness) – a dramatic formation that looks like an upturned roughened sea surface.

95. Answer (d) Explanation DIFFERENCE BETWEEN MONSOON AND PRE-MONSOON RAIN Pre-Monsoon rain commences and withdraws from Peninsular India earlier than the northern parts. June is very much a pre-Monsoon season for North India but the Southwest Monsoon generally strikes Kerala by June 1. Simultaneously, Monsoon reaches parts of Northeast India. Listed below are few characteristics and differences between the Monsoon showers and pre-Monsoon rain in Peninsular India: TEMPERATURE Pre-Monsoon season is synonymous with heat and humidity with uncomfortable conditions throughout the day and night. However, fairly stronger winds bring down temperatures to a comfortable level during Monsoon in India. TYPES OF CLOUDS Clouds are more of vertical in nature during the pre-Monsoon season and mostly come up during late afternoon and early evening hours. They are triggered by high temperatures and towering clouds build up. On the other hand, the Monsoon season is known for stratiform clouds, mainly sheet-like continuous layers of clouds. Depths of these clouds are less but the layers are thick and moisture laden. NATURE OF PRECIPITATION Pre-Monsoon rain is sharp and intense and gets over for the day, after just one spell. But, the Southwest Monsoon brings longer spells of rain which are also recurring in nature. TIME OF OCCURRENCE In the Monsoon season, rain can commence in Peninsular India during any time of the day, though the preferred time is generally late evening. On the other hand, pre-Monsoon rain occurs in the late afternoon and early evening hours. NATURE OF WINDS Pre-Monsoon showers are accompanied by squally winds (A squall is a strong, sudden wind which generally lasts a few minutes then quickly decreases in speed) leading to dust storms but during Monsoon, winds are persistently strong. PROMINENT SEA AND LAND BREEZE Due to differential heating and huge diurnal variation of temperatures, the sea and land breeze remains prominent before the arrival of Southwest Monsoon in India. But, with the available humidity and overcast skies, the breezes are not that marked during Monsoons. Moreover, pre-Monsoon rain is patchy in nature but Southwest Monsoon covers large areas and the weather remains similar in vast pockets. Chill in the air persists irrespective of the terrain, be it Mumbai, Pune, Hyderabad or Bangalore.

96. Answer (c) Explanation Shoreline changes induced by erosion and accretion are natural processes that take place over a range of time scales. They may occur in response to smaller-scale (short-term) events, such as storms, regular wave action, tides and winds, or in response to large-scale (long-term) events such as glaciation or orogenic cycles that may significantly alter sea levels (rise/fall) and tectonic activities that cause coastal land subsidence or emergence. The south west coast of India consists of beaches and cliff as it is submerged coastal plain. Thus, erosion predominates in west coast over deposition.

97. Answer (c) Explanation The objective of ‗Ude Desh ka Aam Nagrik‘ (Regional Connectivity Scheme) is to make flying affordable for the masses, to promote tourism, increase employment and promote balanced regional growth. It also intends to revive un-served and under-served airports. Once the scheme is implemented, passengers will be able to fly an hour‘s journey (of about 500 km) for an all-inclusive fare of Rs 2,500. But, only routes covering a distance of 200-800 km connecting an ‗unserved or under- served‘ airport will qualify for the regional connectivity scheme. In order to support the operators for reducing the airfare, concessions as well as financial support will be given to them. Financial support will be through Viability Gap Funding, for which a cess will be charged to airlines flying on metro or trunk routes for each departure (a model of cross subsidisation).

98. Answer (d) Explanation

Page 72: PRELIMS MODEL TEST-11 ANSWER KEY AND EXPLANATION. PMT-11 to 15 Explanation.pdfTARGET PRELIMS 2017 GENERAL STUDIES PAPER - 1 PRELIMS MODEL TEST-11 ANSWER KEY AND EXPLANATION 1. Answer

NEO IAS 0484-3190310, 9446331522, 9446334122 Page 72 www.neoias.com | www.youtube.com/neoias | www.facebook.com/neoias | www.twitter.com/neoias

Humanizing landscapes (deforestation, largescale irrigation, quarrying) negatively influence monsoon. Some of the important factors influencing Indian monsoon are: Easterly waves Cyclonic depression Thermal contrast Apparent movement of sun (Revolution of earth on its tilted axis) resulting in migration of ITCZ.

99. Answer (b) Explanation Potholes – Running water Drumlines – Glacier Playas – wind Stalagmite – Ground water

100. Answer (d) Explanation States and territories with borders on the sea (clockwise from north) include: the People's Republic of China (including Macau and Hong Kong), the Republic of China (Taiwan), the Philippines, Malaysia, Brunei, Indonesia, Singapore, and Vietnam.

Page 73: PRELIMS MODEL TEST-11 ANSWER KEY AND EXPLANATION. PMT-11 to 15 Explanation.pdfTARGET PRELIMS 2017 GENERAL STUDIES PAPER - 1 PRELIMS MODEL TEST-11 ANSWER KEY AND EXPLANATION 1. Answer

NEO IAS 0484-3190310, 9446331522, 9446334122 Page 73 www.neoias.com | www.youtube.com/neoias | www.facebook.com/neoias | www.twitter.com/neoias

TARGET PRELIMS 2017 GENERAL STUDIES

PAPER - 1

PRELIMS MODEL TEST-15

ANSWER KEY AND EXPLANATION

1. Answer (b) Explanation Amalaka is a ribbed, lenticular or globoid part resembling the amalaka (Indian gooseberry fruit) crowning the top of the North Indian style shikhara.

2. Answer (c) Explanation Viceroy Linlithgow declared India at war with Germany on 3 September 1939. The Congress objected strongly to the declaration of war without prior consultation with Indians. The Congress Working Committee suggested that it would cooperate if there were a Central Indian National Government formed, and a commitment made to India‘s independence after the war. Linlithgow refused the demands of the Congress. On 22 October 1939, Congress ministries tendered their resignations.

3. Answer (c) Explanation Chaitya: Place of congregation and worship. Panchayatana: Main temple surrounded by four sub-shrines in each corner. Squinch is an arch spanning the corners of a square chamber and acting as support for a dome.

1 B 11 C 21 D 31 B 41 B 51 D 61 A 71 A 81 C 91 C

2 C 12 C 22 C 32 C 42 D 52 A 62 A 72 C 82 A 92 A

3 C 13 D 23 B 33 C 43 D 53 C 63 A 73 D 83 B 93 C

4 D 14 C 24 A 34 B 44 C 54 D 64 D 74 C 84 C 94 C

5 D 15 C 25 D 35 C 45 D 55 A 65 B 75 B 85 C 95 A

6 B 16 A 26 D 36 D 46 C 56 B 66 B 76 B 86 B 96 A

7 B 17 B 27 B 37 D 47 B 57 A 67 B 77 D 87 A 97 A

8 D 18 D 28 A 38 C 48 D 58 C 68 C 78 D 88 C 98 B

9 A 19 A 29 D 39 A 49 A 59 C 69 A 79 A 89 C 99 C

10 D 20 C 30 A 40 B 50 C 60 B 70 D 80 C 90 B 100 A

Date: 14-05-2017

Page 74: PRELIMS MODEL TEST-11 ANSWER KEY AND EXPLANATION. PMT-11 to 15 Explanation.pdfTARGET PRELIMS 2017 GENERAL STUDIES PAPER - 1 PRELIMS MODEL TEST-11 ANSWER KEY AND EXPLANATION 1. Answer

NEO IAS 0484-3190310, 9446331522, 9446334122 Page 74 www.neoias.com | www.youtube.com/neoias | www.facebook.com/neoias | www.twitter.com/neoias

4. Answer (d) Explanation The Cabinet Mission proposals included Provisions for three groups of provinces to possess their separate constitutions. It proposed the formation of a Union of India, comprising both the British India and the Princely States. Union would remain in charge of only foreign affairs, defence and communications leaving the residuary powers to be vested in the provinces. It proposed setting up an Interim Government, which would remain in office till a new government was elected on the basis of the new Constitution framed by the Constituent Assembly. Rejection of the demand for a full-fledged Pakistan. It also provided for formation of the constituent assembly on democratic principle of population.

5. Answer (d) Explanation In 1931, the Congress met at Karachi. It approved the Gandhi-Irwin Pact. The most significant contribution of the Karachi session was a resolution it passed on Fundamental Rights and Economic Policy. It outlined the plan for the reconstruction of Indian society after Independence and to furnish many aims and ideals for the Constitution of India and the social and economic policy of the Indian Republic.

6. Answer (b) Explanation The term "Rihla" was especially attributed to the written account of the adventures of the Moroccan traveller and scholar, Ibn Battuta. Ibn Battuta traveled throughout most of the Islamic world during 1304-1377 C.E., later dictating his account as the Rihla.

7. Answer (b) Explanation The Jury Act of 1827 introduced religious discrimination into the structure of the judiciary. Under the provision of the Act, Hindus and Muslims could not sit on the jury when a Christian was being tried. Ram Mohan Roy protested against this because it introduced the pernicious element of ―religious distinction into the judicial system of the land‖. Roy initiated a petition on August 17, 1829 against the Act, which was signed by distinguished Hindus and Muslims and the same was submitted to the British Parliament. Though the petition failed, the matter was taken up by Mr. Grant, the then President of the Board of Control. Grant‘s Bill was passed in 1832, making Hindus and Muslims eligible not only to sit both the petty and grant juries, but also for appointment as justices of the peace. Roy always fought for the cause of right of the people.

8. Answer (d) Explanation Visakhadatta‘s Devichandraguptam is about Chandragupta II‘s accession to the throne after murdering his elder brother Ramagupta and marriage to Dhruvadevi, his brother‘s widow, Vajjika‘s Kaumudimahotsava dealt with the accession of Chandragupta I to the throne and Sudraka‘s Mrichchhakatika was a love story between Vasantasena and Charudatta.

9. Answer (a) Explanation Swarajists wanted an end to the boycott of legislative councils so that the nationalists could enter them to expose the basic weaknesses of these assemblies and use these councils as an arena of political struggle to arouse popular enthusiasm. It was not to gain experience of the functioning of Legislative Council.

10. Answer (d) Explanation Gandhi initiated a limited satyagraha on an individual basis in every locality. The carefully -chosen satyagrahis would preach against participation in the war, after informing the district magistrate of the time and place of the anti war speech. Vinoba Bhave was first satyagrahi on 17 October 1940 and Jawarharlal Nehru, the second. If the government did not arrest them, they planned to start moving towards Delhi (the Delhi Chalo movement). The individual satyagraha not only expressed the Indian people's strong political feeling but gave the British the opportunity to accept Indians demands peacefully.

Page 75: PRELIMS MODEL TEST-11 ANSWER KEY AND EXPLANATION. PMT-11 to 15 Explanation.pdfTARGET PRELIMS 2017 GENERAL STUDIES PAPER - 1 PRELIMS MODEL TEST-11 ANSWER KEY AND EXPLANATION 1. Answer

NEO IAS 0484-3190310, 9446331522, 9446334122 Page 75 www.neoias.com | www.youtube.com/neoias | www.facebook.com/neoias | www.twitter.com/neoias

11. Answer (c) Explanation

12. Answer (c)

Explanation The Champaran Movement of Bihar in 1917 was a result of the long colonial oppressive "teenkathiya system" where the planters forced cultivators to grow indigo (a blue dye) in 3/20th parts of their lands. It addressed the economic demands of the peasants. Champaran satyagraha also featured for the first time that bridges had been built between the peasants and the other sections, especially the middle-class intelligentsia, such as Rajendra Prasad, J.B.Kriplani etc.

13. Answer (d) Explanation The Tolkappiyam is a work on the grammar of the Tamil language and the earliest extant work of Tamil literature and linguistics. It is written in the form of noorpaa or short formulaic compositions and comprises three books–the Ezhuttadikaram, the Solladikaram and the Poruladikaram. Tolkappiyam deals with orthography, phonology, morphology, semantics, prosody and the subject matter of literature.

14. Answer (c) Explanation With the exception of the Governor-General and the Commander-in-chief, all other member of the Executive Council would be nominated from amongst leaders of Indian Political life. All portfolios except the Defence would be held by the Indian Members. Wavell Plan is also known as Breakdown Plan and was not accepted by the British, for whom, leaving without a universally agreed agreement was dishonorable. It also said that in case of a disagreement, the British should withdraw to the 6 Pakistan Provinces, and leaving the Congress to deal with rest of India.

15. Answer (c) Explanation Sir Muhammad Iqbal widely known as Allama Iqbal was a poet, philosopher, and politician, as well as an academic, barrister and scholar in British India who is widely regarded as having inspired the Pakistan Movement. In 1931 and 1932, in recognition of his status as an important Muslim cultural leader, Iqbal was invited in the second and third Round Table Conferences in London. Like the first, these ended in deadlock, with the delegates unable to reach any agreement due to communal differences.

16. Answer (a) Explanation Jallianwala Bagh massacre-mass killing of people in Punjab forced people like Tagore to react and restricted them to enjoy the titles awarded by a brutal repressive power. The inactivity of the Government to punish the culprit

Page 76: PRELIMS MODEL TEST-11 ANSWER KEY AND EXPLANATION. PMT-11 to 15 Explanation.pdfTARGET PRELIMS 2017 GENERAL STUDIES PAPER - 1 PRELIMS MODEL TEST-11 ANSWER KEY AND EXPLANATION 1. Answer

NEO IAS 0484-3190310, 9446331522, 9446334122 Page 76 www.neoias.com | www.youtube.com/neoias | www.facebook.com/neoias | www.twitter.com/neoias

also shown the very intentions of Britishers and also Tagore considered returning award as means not entry to protest Britishers but also to awaken country people against this brutal incident.

17. Answer (b) Explanation Walchand Hirachand Doshi was an Indian industrialist and the founder of the Walchand group. He established India's first modern shipyard, first aircraft factory and first car factory; he also established construction companies, sugar plantations, sugar factories, confectioneries, engineering companies and many other businesses. Ambalal Sarabhai (1890–1967) was a leading industrialist of Ahmedabad and also played an important role in India's freedom struggle. He was founder of Sarabhai group of Companies, like Sarabhai Textiles, Calico Textile Mills, Sarabhai Chemicals etc. He was the President of the Ahmedabad Mill Owner's Association in the year 1918-1919 and also a member of the Ahmedabad Municipal Corporation. He was awarded the Kaisar-i-Hind Gold Medal by the British Government which he renounced.

18. Answer (d) Explanation Hind Swaraj or Indian Home Rule is a book written by Mohandas K. Gandhi in 1909. In it he expresses his views on social ideals such as Swaraj, civilization, modern education, mechanization. Gandhi's Hind Swaraj takes the form of a dialogue between two characters, the Reader and the Editor. The Reader essentially serves as the typical Indian countryman whom Gandhi would have been addressing with Hind Swaraj. The Reader voices the common beliefs and arguments of the time concerning Indian Independence. Gandhi, The Editor, explains why those arguments are flawed and interjects his own arguments. As 'The Editor' Gandhi puts it, "it is my duty patiently to try to remove your prejudice."

19. Answer (a) Explanation In September 1916, Annie Besant started the Home Rule League as an independent political organization. A few months earlier (April) Tilak had established an Indian Home Rule League with the object of attaining Home Rule or self-government. Home Rule Movement was able to combine and balance all the three trends viz. moderates, extremists, and revolutionary terrorists. Home Rule Movement continued extremist‘s idea of passive resistance. Home Rule Movement dropped the idea of mendicancy of the moderates but continued their concept of patriotism. Through his writings in the Mahratta and Kesari, Tilak made the concept of Home Rule the popular catchword. Annie carried on her campaign through the medium of New India and The Commonweal.

20. Answer (c) Explanation The findings of the Bardoli Inquiry committee came to the conclusion that the increase in tax had been unjustified. The Government agreed to restore the confiscated lands and properties, as well as to reduce the rate of increase in tax to 6% as against the earliest rate of 30%.

21. Answer (d) Explanation It is a tribal martial art dance performed mainly in the states of Odisha, Jharkahand and West Bengal. There are three subgenres of this dance based on the place of their origin and development, Purulia Chhau (West Bengal), Seraikella Chhau (Jharkahand) and Mayurbhanj Chhau (Odisha). The dance is mainly performed during the spring festival and last for 13 days. The whole community participates in it. The dance is performed by the male dancers during night time at an open space. It is a blend of both dance and martial practices employing mock combat techniques.

22. Answer (c) Explanation Cabinet mission plan was proposed to achieve the dual objective of coming up with a mechanism for Independent India & of making an interim Government. The proposals can be enumerated as- (1) No division of India & restriction on communal representation. (2) Interim cabinet to be made of Indians. (3) Constituent assembly to be formed on democratic principles. It is to be formed of the representatives from

provinces & princely states. (4) Union Govt. to have limited powers i.e. Finance, foreign affairs & communications. (5) All residuary powers in the hand of provinces & princely states. (6) Representation of provincial legislature to be divided into 3 different groups of provinces.

23. Answer (b) Explanation Durgeshnandini is a Bengali historical romance novel written by Indian writer Bankim Chandra Chattopadhyay in 1865. Durgeshnandini is a story of the love triangle between Jagat Singh, a Mughal General, Tilottama, the daughter of a Bengali feudal lord and Ayesha, the daughter of a rebel Pathan leader against

Page 77: PRELIMS MODEL TEST-11 ANSWER KEY AND EXPLANATION. PMT-11 to 15 Explanation.pdfTARGET PRELIMS 2017 GENERAL STUDIES PAPER - 1 PRELIMS MODEL TEST-11 ANSWER KEY AND EXPLANATION 1. Answer

NEO IAS 0484-3190310, 9446331522, 9446334122 Page 77 www.neoias.com | www.youtube.com/neoias | www.facebook.com/neoias | www.twitter.com/neoias

whom Jagat Singh was fighting. The story is set in the backdrop of Pathan-Mughal conflicts that took place in south-western region of modern-day Indian state of Paschimbanga (West Bengal) during the reign of Akbar. Durgeshnandini is the first Bengali novel written by Bankim Chandra as well as the first major Bengali novel in the history of Bengali literature.

24. Answer (a) Explanation In champaran, the local young educated Congress leaders like Rajendra Prasad or J.B. Kripalani played their role in mobilizing the peasants. The kheda revolt gave India a robust leader in Sardar Vallabhbhai Patel. Sardar Vallabhbhai Patel and his colleagues such as Narhari Parikh, Mohanlal Pandya and Ravi Shankar Vyas organized this major tax revolt. The first individual Satyagrahi selected was Acharya Vinoba Bhave, who was sent to Jail when he spoke against the war. Second Satyagrahi was Jawahar Lal Nehru. Third was Brahma Datt, one of the inmates of the Gandhi‘s Ashram. They all were sent to jails for violating the Defence of India Act.

25. Answer (d) Explanation According to the proposals of the Cabinet Mission announced in May, 1946, the country could have a Federal government consisting of the British and the Indian India. The Federation was naturally to look after the national and international problems like the Defence and communication, the foreign affairs and the international trade while the provinces were to enjoy complete autonomy in Provincial subjects like police, agriculture, industries, educational and many other local problems.

26. Answer (d) Explanation Self-help and constructive work at the village level was envisaged as a means of bringing about the social and economic regeneration of the villages and of reaching the rural masses. This meant social reform and campaigns against evils such as caste oppression, early marriage, the dowry system, consumption of alcohol, etc. The economic boycott movement seemed to be a suitable opportunity for reviving Indian small scale industries. The boycott of foreign goods led to the increase in demand of indigenous goods especially clothes which fell short of supply. The mill-owners of Bombay and Ahmadabad came to its rescue. The Boycott movement in Bengal supplied a momentum and driving force to the cotton mills in India and the opportunity thus presented was exploited by the mill-owners. The economic boycott movement seemed to be a suitable opportunity for reviving weaving industry. The clothes produce were accepted by the Bengalis in the true spirit of the Swadeshi Movement. Further, J N Tata founded Tata Iron and Steel. Prafulla Chandra Roy set up Bengal Chemicals Factory. Aravindo Ghosh, Syed Haidar Raza and Bipin Chandra Pal took part in Swadeshi.

27. Answer (b) Explanation The copper plate inscriptions such as the Srirangam copper plates of Devaraya II provide the genealogy and achievements of Vijayanagar rulers. Italian traveller Nicolo de Conti visited Vijaynagar empire during Devaraya II. From the account of the contemporary European explorer Nicolo Conti, the Devaraya II levied tribute on Ceylon, Quilon, Pegu, Pulicat and Tenasserim. But he could not win any clear victory over the Bahmani Sultans.

28. Answer (a) Explanation Krishna himself was the author of one Telugu work Amuktamalyada, and an accomplished Sanskrit scholar and wrote one Sanskrit work Madalasa Charita, Satyavadu Parinaya, Rasamanjari and Jambavati Kalyanam. Ramarajas enemies ultimately combined to inflict a crushing defeat on Vijayanagar at Bennaihatti, near Talikota, in 1565. This battle is also known as the battle of Rakasa Tangadi. Rama Raja was imprisoned and immediately executed. This battle is generally considered to mark the end of the great age of Vijayanagar. Although the kingdom lingered on for almost one hundred years under the Aravidu dynasty, its territories shrank steadily and the Raya no longer counted in the political affairs of South India.

29. Answer (d) Explanation Yaoshang is the premier festival of Manipur which is celebrated for five days commencing from the full moon day of Phalguna (February/March). The Thabal Chongba a kind of Manipuri folk dance is particularly associated with this festival.

30. Answer(a) Explanation Fahien visited India during the reign of Chandragupta II. Hiuen Tsang visited India during the reign of Harsha, and he gives us information about the popularity of Buddhism during Gupta period, and Itsing visited India in about 672-5 AD, but refers to the donation of some villages to Chinese Buddhists at Nalanda monastery by the first Gupta, Srigupta.

Page 78: PRELIMS MODEL TEST-11 ANSWER KEY AND EXPLANATION. PMT-11 to 15 Explanation.pdfTARGET PRELIMS 2017 GENERAL STUDIES PAPER - 1 PRELIMS MODEL TEST-11 ANSWER KEY AND EXPLANATION 1. Answer

NEO IAS 0484-3190310, 9446331522, 9446334122 Page 78 www.neoias.com | www.youtube.com/neoias | www.facebook.com/neoias | www.twitter.com/neoias

31. Answer (b) Explanation Both of them were opposed to animal sacrifices. Both came as a protest movement against Hinduism. The Jaina conception of soul differed from that of the Buddhists. The Jainas ascribe life to plants, stone and water, which the Buddhists reject. Their concept of Jiva (soul) and Ajiva (matter) is entirely different from the Buddhist concept of soul. The Jainas practice rigorous asceticism and self- mortification. Mahavira himself practiced tremendous physical hardships to realize the truth. He advised his followers to starve and undergo physical suffering. But Buddha was opposed to extreme penance and privations. He advised a ―Middle Path‖.

32. Answer (c) Explanation Mṛicchakaṭika (The Little Clay Cart) is a ten-act Sanskrit drama attributed to Śūdraka , an ancient playwright generally thought to have lived sometime between the third century BC and the fifth century AD whom the prologue identifies as a Kshatriya king and a devotee of Siva who lived for 100 years. The play is set in the ancient city of Ujjayini during the reign of the King Palaka, near the end of the Pradyota dynasty that made up the first quarter of the fifth century BC. The central story is that of noble but impoverished young Brahmin, Chārudatta, who falls in love with a wealthy courtesan, Vasantasenā.

33. Answer(c) Explanation Land trade and sea trade were in vogue. They did not use metal money. Most probably they carried on all exchanges through barter. The Mesopotamian records from about 2350 B.C. onwards refer to trade relations with Meluha, which was the ancient name given to the Indus region, and they also speak of two intermediate stations called ‗Dilmun‘ (identified with Bahrain on the Persian Gulf) and Makan (Makran Coast).

34. Answer (b) Explanation The recommendations of the Simon Commission were published in May 1930. They were as follows: Dyarchy in the provinces should be abolished and ministers should be made responsible to the provincial legislatures in all departments, including the department of law and order. The Governor was to retain the special powers for the safety and tranquillity of the province and for the protection of the minorities. The franchise was to be extended and legislatures were to be enlarged. At the Centre, a Federal assembly would be constituted on the basis of representation of the provinces and other areas as per the population. No change in the Central Executive. Burma should be separated from the British India and should be provided a constitution of its own.

35. Answer (c) Explanation The Radcliffe Committee chaired by Sir Cyril Radcliffe was appointed to delimit the boundaries between India and Pakistan.

36. Answer (d) Explanation Constituent assembly to be partly elected by provincial assemblies by proportional representation with and partly nominated by princes. The constituent assembly will decide the Union Constitution along with provincial constitutions.

37. Answer (d) Explanation John Mathai, an economist headed Industries and Supplies and C. Rajagopalachari was head of the Department of Education and Arts. The Department of Railways and Transport was headed by Congress leader Asaf Ali, Jagjivan Ram leader of the Scheduled Castes headed the Labour Department and Rajendra Prasad headed the Food and Agriculture Department.

38. Answer (c) Explanation It is a combined dance drama conducted by the Chakkiyars (a sub-caste among Hindus) who play the male caste traditionally in the state of Kerala. The women of the Nangiar caste play the female roles. The performance lasts from 6 to 20 days. They are mainly enacted inside the temples and the theme is based on Hindu mythology. Mizhavu is the major musical instrument used here. Kutiyattam is traditionally performed in theatres called Kuttampalams, which are located in Hindu temples. In its stylized and codified theatrical language, netra abhinaya (eye expression) and hasta abhinaya (the language of gestures) are prominent.

Page 79: PRELIMS MODEL TEST-11 ANSWER KEY AND EXPLANATION. PMT-11 to 15 Explanation.pdfTARGET PRELIMS 2017 GENERAL STUDIES PAPER - 1 PRELIMS MODEL TEST-11 ANSWER KEY AND EXPLANATION 1. Answer

NEO IAS 0484-3190310, 9446331522, 9446334122 Page 79 www.neoias.com | www.youtube.com/neoias | www.facebook.com/neoias | www.twitter.com/neoias

39. Answer (a) Explanation The failure of the Cripps Mission to solve the constitutional deadlock exposed Britain‘s unchanged attitude on constitutional advance and made it clear that any more silence would enable British to decide the fate of future India by themselves.

40. Answer (b) Explanation It was not supported by Hindu Mahasabha. The women actively took part in Quit India Movement of 1942.There were no communal clashes during the movement and formation of parallel governments were highlights of this movement.

41. Answer (b) Explanation The Muslim League should cooperate with the Congress in the formation of provisional interim government for the transitional period. After the war, a commission shall be appointed to demarcate the boundaries of the Muslim-dominated districts in the north-west and east of India. The people of these districts shall decide, by plebiscite, the issue of separation from India. In the event of separation, mutual agreement shall be entered into between the two governments for jointly safeguarding defence, commerce, and communication and for other essential purposes.

42. Answer (d) Explanation Allan Octavian Hume (A.O. Hume), a restricted British member of Indian Civil Service who had settled at Simla, founded the Indian National Union, the forerunner of Indian Nation Congress in 1884. After touring Calcutta, Bombay and Madras, Hume announced that a conference of the Indian National Union would be held at Pune in December, which would be composed of educated delegates of all parts of the three presidencies. But cholera having broken out at Pune, the venue of the conference was shifted to Tejpal Sanskrit Pathshal, Bombay. The first session of the Indian National Congress was held on December 28, 1885. At the suggestion of Dadhabai Naoroji the name of the organization was changed to Indian Nation Congress (INC). Womesh Chandra Bannerjee, a leading lawyer of Calcutta was elected its president.

43. Answer (d) Explanation The contribution of Matangini Hazra is a perfect example of the involvement of rural folk in the nationalist struggle of 1942. Aruna Asaf Ali is remembered for the daring act of raising the Tricolour at the Gowalia tank in the midst of police brutality following the Bombay Resolution. She was proclaimed an offender by the police and her property was seized. Subsequently, she got involved in the Royal Indian Navy revolt of 1946. Usha Mehta was a Gandhian and freedom fighter of India. She is also remembered for organizing the Congress Radio, also called the Secret Congress Radio, an underground radio station, which functioned for few months during the Quit India Movement of 1942.

44. Answer (c) Explanation A traditional painting of Odisha, the name Pattachitra comes from a Sanskrit word Patta which means Canvas/Cloth and Chitra means picture. The paintings show a mix of classic and folk elements, with a bias towards the latter. The base of the painting is treated cloth while the colours used come from natural sources including burnt coconut shells. No pencil or charcoal is used, rather brush is used to draw the outlines in red or yellow after which the colours are filled. They are inspired from Jagannath and Vaishnava cult.

45. Answer (d) Explanation Suvarnabhumi, which means ―The Land of Gold‖, is an ancient term for Southeast Asia, found in early Buddhist and Hindu literature. There were also legends, religious accounts, and foreign traders‘ written accounts dating to the first millennium AD that mentioned the name ―Suvarnabhumi‖. These accounts have convinced scholars that the Suvarnabhumi of the past is the Southeast Asia of the present.

46. Answer (c) Explanation Indian National Association was the first declared Nationalist Organization founded in British India by Surendranath Banerjee and Anand Mohan Bose in 1876. Abul Kalam Azad Prominent Nationalist leader

Became youngest president of INC in 1923

President of INC during Quit India Movement

Page 80: PRELIMS MODEL TEST-11 ANSWER KEY AND EXPLANATION. PMT-11 to 15 Explanation.pdfTARGET PRELIMS 2017 GENERAL STUDIES PAPER - 1 PRELIMS MODEL TEST-11 ANSWER KEY AND EXPLANATION 1. Answer

NEO IAS 0484-3190310, 9446331522, 9446334122 Page 80 www.neoias.com | www.youtube.com/neoias | www.facebook.com/neoias | www.twitter.com/neoias

Published weekly ‗Al-Hilal‘

47. Answer (b) Explanation Results of Munda Ulgulan: Government enacted Chotanagpur Tenancy Act, 1908. Government recognized Khuntkatti rights Government banned Beth Begari (forced labour) Birsa Munda became a legend to the tribals of Chota Nagpur, and a symbol of the anti-feudal, anti-colonial

struggle of that time.

48. Answer (d) Explanation T R Holmes: A conflict between civilization and barbarism. V D Savarkar: India‘s planned war of Independence. Karl Marx: Struggle of soldiers, peasants and democratic combine against the foreign and feudal bondage. R C Mazumadar: ―Neither first, nor national not war of independence.‖

49. Answer (a) Explanation This school of philosophy was developed by Patanjali sage. According to the Yoga school, a person can attain salvation through meditation and physical activity. Practice of control over pleasure senses and bodily organs is central to this system. In order to obtain salvation, physical exercises in various postures called asana are prescribed, and the breathing exercise called pranayama is recommended. It is thought that through these methods the mind gets diverted from worldly matters and achieves concentration.

50. Answer (c) Explanation British labour reforms were not a reason for tribal uprisings, instead land reforms, influx of outsiders, Christian missionaries, forced labour, forest laws etc were reasons.

51. Answer (d) Explanation Ethnic ties were a basic feature of the tribal rebellions. The rebels saw themselves not as discreet class but as having a tribal identity. At this level the solidarity shown was of a very high order. Fellow tribals were never attacked unless they had collaborated with the enemy. At the same time not all outsiders were attacked as enemies. Tribal uprisings were not supported by Indian intellectuals.

52. Answer (a) Explanation Tattvabodhini Sabha was a particular reform movement organization, aiming to popularize Brahmodharma or Brahmo faith. When Brahmo Samaj became weak after the death of its founder Ram Mohan Roy, Debendranath Tagore took every possible initiative and established the 'Tattvaranjini Sabha' on 6 October 1839. This Sabha was later renamed as 'Tattvabodhini Sabha'. It encouraged religious inquiries and disseminate the essence of Upanishads.

53. Answer (c) Explanation Mir Saman, under Mughal administration was the in charge of the imperial department

54. Answer (d) Explanation The Prarthana Samaj was established in Bombay by Dr. Atma Ram Pandurang (1825-1898) in 1867 with the objective of rational worship and social reform. The two great members of this Samaj were Shri R.C. Bhadarkar and Justice Mahadev Govind Ranade. They devoted themselves to the work of social reform such as inter-caste dining, inter-caste marriage, widow remarriage and improvement of the lot of women and depressed classes.

55. Answer (a) Explanation Theosophy subscribed to the spiritual philosophy of Hinduism and its doctrine of Karma and transmigration of the soul. The beliefs of the Theosophical Society were a mixture of religion, philosophy and occultism. It promoted a spirit of national pride among the Indians.

56. Answer (b) Explanation The movement started by Derozio was called the Young Bengal Movement and his followers were known as the Derozians. They condemned religious rites and the rituals, and pleaded for eradication of social evils, female education and improvement in the condition of women. They carried on public agitation on public questions such as the revision of the Company‘s Charter, the freedom of the Press, better treatment for Indian labour in

Page 81: PRELIMS MODEL TEST-11 ANSWER KEY AND EXPLANATION. PMT-11 to 15 Explanation.pdfTARGET PRELIMS 2017 GENERAL STUDIES PAPER - 1 PRELIMS MODEL TEST-11 ANSWER KEY AND EXPLANATION 1. Answer

NEO IAS 0484-3190310, 9446331522, 9446334122 Page 81 www.neoias.com | www.youtube.com/neoias | www.facebook.com/neoias | www.twitter.com/neoias

British colonies abroad, trial by jury, protection of the riots from oppressive zamindars, and employment of Indians in the higher grades of government services. But the extreme radical ideology of Derozians resulted in lack of support and gradual decline of movement.

57. Answer (a) Explanation The meaning of Dipavamsa is ―Chronicle of Island‖. It is the oldest historical record of Sri Lanka. It is believed to have been compiled around 3rd or 4th century BC somewhere in Anuradhapura in Sri Lanka during the reign of King Dhatusena of Sri Lanka, the first Mauryan King of Sri Lanka. The Avukana Buddha statue was erected by King Dhatusena in Sri Lanka. Dipavamsa is one of the most important works in Pali Literature. It details the tooth relic and Bodhi Tree‘s arrival in Sri Lanka. It also deals with the arrival of Buddha‘s teaching and preachers in Sri Lanka. It mentions that Buddha visited Kelaniya and Dighavapi in Sri Lanka. Mahavamsa is the most important Pali epic poem. Mahavamsa means ―Great Chronicle‖. It‘s a historical poem in Pali Language which deals about the Kings of Sri Lanka. The first version of Mahavamsa dates back to 3-4th century BC during the reign of King Vijaya.

58. Answer (c) Explanation Under permanent settlement system, large scale land were transferred from the cultivating to non-cultivating class. Thus the new zamindari class became owners of estates. For East India Company, this system secured the British Dominion in India. In Zamindars, it created a faithful class of Indians who proved to be a great instrument for the security of the British interests in India. This settlement was the reason that during the 1857 mutiny, the Zamindars remained loyal to the company and did not provide any help to the rebels.

59. Answer (c) Explanation The servants of the East India Company amassed great wealth by abusing the system. It separated power from responsibility; Under this system, the administration was divided between the Company and the Nawab but the whole power was actually concentrated in the hands of the Company. Nizamat function (police and judicial) was with Nawab but it was also undertaken by the Company. For Nizamat functions, the British gave the additional responsibility of deputy Nazim to Mohammad Raza Khan. The deputy Nazim could not be removed without the consent of the company.

60. Answer (b) Explanation Mudiyettu is a ritual dance drama from Kerala based on the mythological tale of a battle between the goddess Kali and the demon Darika. The dance is performed in the village temples, called Bhagavati Kavus, in between February and May after the harvesting season. The performers do heavy make-up and wear gorgeous attire with conventional facial painting, tall headgear etc to give the touch of the supernatural. Mutual cooperation and collective participation of each caste in the rituals strengthens their common identity and bonding.

61. Answer: (a) Explanation The 13th EU-India Summit was held in Brussels on 30

th March 2016. The EU was represented by Mr. Donald

Tusk, President of the European Council, and Mr. Jean-Claude Juncker, President of the European Commission. The Republic of India was represented by Prime Minister Shri Narendra Modi.

62. Answer (a) Explanation In May 2016, the Kenyan government decided to close Dadaab refugee camp which has hundreds of thousands of Somali refugees and is often referred to as the world's largest refugee camp. The decision was condemned by domestic and international rights groups and organizations dealing with refugees. A Kenyan court ruled on February 2017 that the government must not close the world‘s largest refugee camp and send more than 2,00,000 people back to war-torn Somalia.

63. Answer (a) Explanation The 457 visa programme in Australia, allows business to employ foreign workers for a period up to four years in skilled jobs where there is a shortage of Australian workers. Recently Australia abolished the 457 visa programme used by over 95,000 temporary foreign workers, to tackle the growing unemployment in the country. The majority of the visa holders were from India, followed by the United Kingdom and China. The programme will be replaced by another visa programme, with new restrictions.

64. Answer (d) Explanation The Arctic Council is the leading intergovernmental forum promoting cooperation, coordination and interaction among the Arctic States, Arctic indigenous communities and other Arctic inhabitants on common Arctic issues, in particular on issues of sustainable development and environmental protection in the Arctic.

Page 82: PRELIMS MODEL TEST-11 ANSWER KEY AND EXPLANATION. PMT-11 to 15 Explanation.pdfTARGET PRELIMS 2017 GENERAL STUDIES PAPER - 1 PRELIMS MODEL TEST-11 ANSWER KEY AND EXPLANATION 1. Answer

NEO IAS 0484-3190310, 9446331522, 9446334122 Page 82 www.neoias.com | www.youtube.com/neoias | www.facebook.com/neoias | www.twitter.com/neoias

Eight member countries are: Canada, Denmark, Finland, Iceland, Norway, Russia, Sweden, and the United States as these are the eight countries with sovereignty over the lands within the Arctic Circle. The Arctic Council‘s mandate explicitly excludes military security. India is an observer in Arctic Council.

65. Answer (b) Explanation The Great Repeal Bill will repeal the 1972 European Communities Act, which took Britain into the EU and meant that European law took precedence over laws passed in the British parliament. It will also end the jurisdiction of the European Court of Justice. All existing EU legislation will be copied across into domestic UK law to ensure a smooth transition after Brexit.

66. Answer (b) Explanation North Korea is located in East Asia on the northern half of the Korean Peninsula. North Korea shares a border with three countries; China along the Amnok River, Russia along the Tumen River, and South Korea along the Korean Demilitarized Zone. (DMZ is an area in which treaties or agreements between nations, military powers or contending groups forbid military installations, activities or personnel.)

67. Answer (b) Explanation Kuril Islands stretches north across the Pacific Ocean from the Japanese island of Hokkaido to the southern tip of Russia's Kamchatka Peninsula. Four islands - which Russia calls the Southern Kurils and Japan calls the Northern Territories - are the subject of a dispute between the two nations. Because of the dispute, Russia and Japan have not yet signed a peace treaty to end World War II. All the islands are currently under Russian jurisdiction.

68. Answer (c) Explanation The Organization of the Petroleum Exporting Countries (OPEC) was founded in Baghdad, Iraq, with the signing of an agreement in September 1960 by five countries namely Islamic Republic of Iran, Iraq, Kuwait, Saudi Arabia and Venezuela. They were to become the Founder Members of the Organization. These countries were later joined by Qatar (1961), Indonesia (1962), Libya (1962), the United Arab Emirates (1967), Algeria (1969), Nigeria (1971), Ecuador (1973), Gabon (1975) and Angola (2007). Oman is not an OPEC member. Oman is the largest producer of crude oil in the Middle East that is not a member of the Organization of the Petroleum Exporting Countries (OPEC).

69. Answer (a) Explanation The Strait of Gibraltar is a narrow strait that connects the Atlantic Ocean to the Mediterranean Sea. Gibraltar, a British Overseas Territory, located on the southern end of the Iberian Peninsula, is the subject of an irredentist territorial claim by Spain.

70. Answer (d) Explanation Rakhine is a state in Myanmar, situated on the western coast. Rakhine like many parts of Burma, has a diverse ethnic population. The ethnic Rakhine make up the majority, followed by a considerable population of Rohingya Muslims. The Rohingya conflict is one of the longest conflicts between the majority Buddhist Burmese and the minority Muslims in Myanmar. In 2015 a refugee crisis involving the Rohingyas spread panic across the region, after boats carrying hundreds of refugees were found floating in the sea. Many of these refugees also fled to India. There are an estimated 36,000 Rohingya refugees presently located in India. For India, the Rohingya problem is made further significant by the fact that many of them occupy Rakhine province – home to Sittwe Port.

71. Answer (a) Explanation The World Happiness Report is a measure of happiness published by the United Nations Sustainable Development Solutions Network. The first World Happiness Report was released in 2012. Norway is the overall happiest country in the world and India ranked 122 according to the 2017 report. Bhutan is the first and so far only country to officially adopt Gross National Happiness (GNH) instead of Gross Domestic Product (GDP) as their main development indicator.

72. Answer (c) Explanation The Wassenaar Arrangement has been established in order to contribute to regional and international security and stability, by promoting transparency and greater responsibility in transfers of conventional arms and dual-use goods and technologies, thus preventing destabilising accumulations. Participating States seek, through their national policies, to ensure that transfers of these items do not contribute to the development or enhancement of military capabilities which undermine these goals, and are not diverted to support such capabilities.

Page 83: PRELIMS MODEL TEST-11 ANSWER KEY AND EXPLANATION. PMT-11 to 15 Explanation.pdfTARGET PRELIMS 2017 GENERAL STUDIES PAPER - 1 PRELIMS MODEL TEST-11 ANSWER KEY AND EXPLANATION 1. Answer

NEO IAS 0484-3190310, 9446331522, 9446334122 Page 83 www.neoias.com | www.youtube.com/neoias | www.facebook.com/neoias | www.twitter.com/neoias

The aim is also to prevent the acquisition of these items by terrorists. Representatives of Participating States meet regularly in Vienna where the Wassenaar Arrangement has established its headquarters and a small Secretariat.

73. Answer (d) Explanation The Australia Group is an informal arrangement which aims to allow exporting or transshipping countries to minimise the risk of assisting chemical and biological weapon (CBW) proliferation. Participants in the Australia Group do not undertake any legally binding obligations: the effectiveness of their cooperation depends solely on a shared commitment to CBW non-proliferation goals and the strength of their respective national measures. All states participating in the Australia Group are parties to the Chemical Weapons Convention (CWC) and the Biological Weapons Convention (BWC).

74. Answer (c) Explanation The Pelindaba Treaty signed in 1996, also known as the African Nuclear Weapon Free Zone Treaty, aims at preventing nuclear proliferation and preventing strategic minerals of Africa from being exported freely. The treaty establishes a Nuclear-Weapon-Free Zone in Africa.

75. Answer (b) Explanation The iconic Teen Murti Chowk and the Teen Murti Marg in New Delhi will now be known after the Israeli city Haifa with the NDMC (New Delhi Municipal Council) approving their renaming ahead of Prime Minister Narendra Modi's visit to Israel. (Every year, September 23 is celebrated as Haifa Day in India and Israel as a mark of respect to the soldiers who lost their lives in the battle and to mark the end to 400 years of the Turkish control over the city.)

76. Answer (b) Explanation The Hague Code of Conduct against Ballistic Missile Proliferation (HCoC) is a voluntary legally non-binding multilateral body aimed at preventing the spread of ballistic missiles that can deliver weapons of mass destruction. India has recently joined the Hague Code of Conduct against Ballistic Missile Proliferation (HCoC). The HCoC was formed in November 2002 with 137 members. China, Pakistan, Israel and Iran have not yet joined the voluntary regime.

77. Answer (d) Explanation Organization of American States (OAS) Intergovernmental organisation of American Republics. OAS brings together all 35 independent states of the Americas and constitutes the main political, juridical, and social governmental forum in the Hemisphere. In addition, it has granted permanent observer status to 69 states, as well as to the European Union (EU).

78. Answer (d) Explanation The Prime Minister Narendra Modi has been conferred with Amir Amanullah Khan Award, Afghanistan‘s highest civilian honour. He was bestowed the honour by President Ashraf Ghani after the inauguration of the landmark Afghan-India Friendship Dam in Herat. It is the Highest Civilian honour bestowed by Afghan government to Afghan national as well as foreigners in appreciation of their services.

79. Answer (a) Explanation The Mekong-Ganga Cooperation (MGC) is an initiative by six countries – India and five ASEAN countries, namely, Cambodia, Lao PDR, Myanmar, Thailand and Vietnam for cooperation in tourism, culture, education, as well as transport and communications. Commonwealth of Independent States (CIS) was created in December 1991 during the dissolution of the Soviet Union. At present the CIS unites: Azerbaijan, Armenia, Belarus, Georgia, Kazakhstan, Kyrgyzstan, Moldova, Russia, Tajikistan, Turkmenistan, Uzbekistan and Ukraine. India has been a member of International Atomic Energy Agency (IAEA) from 1957 onwards.

80. Answer (c) Explanation The Nuclear Suppliers Group (NSG) is a group of nuclear supplier countries that seeks to contribute to the non-proliferation of nuclear weapons through the implementation of two sets of Guidelines for nuclear exports and nuclear-related exports. NSG was created following the explosion in 1974 of a nuclear device by India. NSG works on the basis of consensus.

81. Answer (c) Explanation

Page 84: PRELIMS MODEL TEST-11 ANSWER KEY AND EXPLANATION. PMT-11 to 15 Explanation.pdfTARGET PRELIMS 2017 GENERAL STUDIES PAPER - 1 PRELIMS MODEL TEST-11 ANSWER KEY AND EXPLANATION 1. Answer

NEO IAS 0484-3190310, 9446331522, 9446334122 Page 84 www.neoias.com | www.youtube.com/neoias | www.facebook.com/neoias | www.twitter.com/neoias

The Missile Technology Control Regime (MTCR) is an informal political understanding among states that seek to limit the proliferation of missiles and missile technology. The MTCR is not a treaty and does not impose any legally binding obligations on Partners (members). While there is no formal linkage, the activities of the MTCR are consistent with the UN‘s non-proliferation and export control efforts.

82. Answer (a) Explanation Scarborough Shoal is a territory disputed between Philippines and China in the South China Sea.

83. Answer (b) Explanation The Asia-Pacific Economic Cooperation (APEC) is a regional economic forum established in 1989 to leverage the growing interdependence of the Asia-Pacific. APEC's 21 member economies are Australia; Brunei Darussalam; Canada; Chile; People's Republic of China; Hong Kong, China; Indonesia; Japan; Republic of Korea; Malaysia; Mexico; New Zealand; Papua New Guinea; Peru; The Philippines; The Russian Federation; Singapore; Chinese Taipei; Thailand; United States of America; Viet Nam.

84. Answer (c) Explanation Malabar Naval Exercise is a trilateral naval exercise involving the United States, Japan and India as permanent partners. Originally a bilateral exercise between India and the U.S., Japan became a permanent partner in 2015. Past non-permanent participants are Australia and Singapore.

85. Answer (c) Explanation Kigali Agreement to amend the Montreal Protocol, was agreed to at the 28th Meeting of Parties at Kigali, Rwanda on October 2016. 14th ASEAN-India Summit and the 11th East Asia Summit was held in Vientiane, Laos on September 2016. The northern Syrian city of Aleppo was a key battleground in the Syrian civil war.

86. Answer(b) Explanation Balochistan is one of the four provinces of Pakistan, located in the southwestern region of the country. It was not a part of the former princely state of Jammu and Kashmir. It was previously ruled by the Khan of Kalat and was forcefully annexed by Pakistan in 1948. Gwadar Port is a deep-sea port situated on the Arabian Sea at Gwadar in Balochistan province of Pakistan.

87. Answer (a) Explanation

Page 85: PRELIMS MODEL TEST-11 ANSWER KEY AND EXPLANATION. PMT-11 to 15 Explanation.pdfTARGET PRELIMS 2017 GENERAL STUDIES PAPER - 1 PRELIMS MODEL TEST-11 ANSWER KEY AND EXPLANATION 1. Answer

NEO IAS 0484-3190310, 9446331522, 9446334122 Page 85 www.neoias.com | www.youtube.com/neoias | www.facebook.com/neoias | www.twitter.com/neoias

To strengthen sub-regional cooperation, four South Asian countries– Bangladesh, Bhutan, India and Nepal (BBIN) signed the historic Motor Vehicle Agreement (BBIN-MVA) for the ‗Regulation of Passenger, Personal and Cargo Vehicular Traffic‘ amongst them on June 2015. But Bhutan is finding it difficult to get parliamentary approval for the pact.

88. Answer (c) Explanation Logistics Exchange Memorandum of Agreement (LEMOA), is a tweaked India-specific version of the Logistics Support Agreement (LSA). LEMOA gives both Indian and United States of America access to designated military facilities on either side for the purpose of refueling and replenishment. This is not a basing agreement. There will be no basing of the U.S. troops or assets on Indian soil. This is purely a logistical agreement.

89. Answer (c) Explanation The Asia-Europe Meeting (ASEM) was established as a dialogue forum in 1996 to foster dialogue and cooperation between Asia and Europe. Its 53 partners include 51 member countries and 2 regional organisations (ASEAN Secretariat and European Commission).

90. Answer (b) Explanation India is one of the founding members of the Non Aligned Movement. Vice President M. Hamid Ansari led the Indian delegation at the 17th Summit of the Non Aligned Movement (NAM) held at Margarita Island in Venezuela. This is the first NAM summit that will not be attended by an Indian PM, except for 1979 when caretaker PM Charan Singh skipped the summit in Havana.

91. Answer (c) Explanation Raisina Dialogue is India‘s flagship geo-political conference. It is held annually in New Delhi from 2016 onwards. The conference is held jointly by Ministry of External Affairs and the Observer Research Foundation (ORF), an independent think tank based in India. 2nd Edition of Raisina Dialogue was held from 17-19 January, 2017. The Theme of the second edition of the Dialogue, was ―The New Normal: Multilateralism with Multi-Polarity‖.

92. Answer (a) Explanation Islamic Military Alliance to Fight Terrorism (IMAFT) is a military alliance led by Saudi Arabia to fight terrorism, with a joint operations centre in Riyadh. The 39 member military alliance include most Arab League states, a number of mainly Muslim states mostly in Africa, and Asian countries including Pakistan, Bangladesh and Malaysia. Notable omissions are Iran, Iraq and Syria.

93. Answer (c) Explanation The Group of Twenty (G20) is the premier forum for its members‘ international economic cooperation and decision-making. Its membership comprises 19 countries plus the European Union. The members of the G20 are Argentina, Australia, Brazil, Canada, China, France, Germany, India, Indonesia, Italy, Japan, Republic of Korea, Mexico, Russia, Saudi Arabia, South Africa, Turkey, the United Kingdom, the United States and the European Union.

94. Answer (c) Explanation India, Afghanistan and Iran signed the trilateral Chabahar Agreement among the three countries on Transport and Transit Corridors. The Agreement envisions trilateral cooperation for providing alternative access to seas to Afghanistan, inter alia for Afghanistan‘s trade with India. When the Agreement comes into force it will significantly enhance utilization of Chabahar Port, contribute to economic growth of Afghanistan, and facilitate better regional connectivity, including between India and connections to Afghanistan and Central Asia.

95. Answer (a) Explanation The South Asia Subregional Economic Cooperation (SASEC) Program brings together Bangladesh, Bhutan, India, Maldives, Myanmar, Nepal, and Sri Lanka in a project-based partnership that aims to promote regional prosperity, improve economic opportunities, and build a better quality of life for the people of the subregion. SASEC seeks to strengthen multimodal cross-border transport networks that boost intraregional trade and open up trade opportunities with East and Southeast Asia. Asian Development Bank (ADB) serves as the SASEC Secretariat.

96. Answer (a) Explanation

Page 86: PRELIMS MODEL TEST-11 ANSWER KEY AND EXPLANATION. PMT-11 to 15 Explanation.pdfTARGET PRELIMS 2017 GENERAL STUDIES PAPER - 1 PRELIMS MODEL TEST-11 ANSWER KEY AND EXPLANATION 1. Answer

NEO IAS 0484-3190310, 9446331522, 9446334122 Page 86 www.neoias.com | www.youtube.com/neoias | www.facebook.com/neoias | www.twitter.com/neoias

Organisation of Islamic Cooperation (OIC) endeavours to safeguard and protect the interests of the Muslim world in the spirit of promoting international peace and harmony among various people of the world. India is not a member of OIC. It has membership of 57 states spread over four continents.

97. Answer (a) Explanation The Shanghai Cooperation Organisation (SCO) is a permanent intergovernmental international organisation, the creation of which was announced on 15 June 2001 in Shanghai (China) by the Republic of Kazakhstan, the People's Republic of China, the Kyrgyz Republic, the Russian Federation, the Republic of Tajikistan, and the Republic of Uzbekistan. It was preceded by the Shanghai Five mechanism. The SCO Secretariat, based in Beijing, is the main permanent executive body of the SCO.

98. Answer (b) Explanation The Bay of Bengal Initiative for Multi-Sectoral Technical and Economic Cooperation (BIMSTEC) is a sub-regional organization comprising seven Member States lying in the littoral and adjacent areas of the Bay of Bengal. It constitutes seven Member States: five deriving from South Asia, including Bangladesh, Bhutan, India, Nepal, Sri Lanka, and two from Southeast Asia, including Myanmar and Thailand. Unlike many other regional groupings, BIMSTEC is a sector-driven cooperative organization. BIMSTEC Summit is the highest policy making body in the BIMSTEC process.

99. Answer (c) Explanation The South Asian Association for Regional Cooperation (SAARC) was established with the signing of the SAARC Charter in Dhaka on 8 December 1985. SAARC comprises of eight Member States: Afghanistan, Bangladesh, Bhutan, India, Maldives, Nepal, Pakistan and Sri Lanka. The Secretariat of the Association was set up in Kathmandu on 17 January 1987.

100. Answer (a) Explanation BRICS brings together five major emerging economies Brazil, Russia, India, China and South Africa who are also members of G20. BRIC started after the meeting of the Leaders of Russia, India and China in St. Petersburg on the margins of G8 Outreach Summit in 2006. The 1st BRIC Summit was held in Yekaterinburg, Russia, on 16 June 2009. It was agreed to expand BRIC into BRICS with the inclusion of South Africa at the BRIC Foreign Ministers‘ meeting in New York in September 2010. Accordingly, South Africa attended the 3rd BRICS Summit in Sanya, China on 14 April 2011.

Page 87: PRELIMS MODEL TEST-11 ANSWER KEY AND EXPLANATION. PMT-11 to 15 Explanation.pdfTARGET PRELIMS 2017 GENERAL STUDIES PAPER - 1 PRELIMS MODEL TEST-11 ANSWER KEY AND EXPLANATION 1. Answer

NEO IAS 0484-3190310, 9446331522, 9446334122 Page 87 www.neoias.com | www.youtube.com/neoias | www.facebook.com/neoias | www.twitter.com/neoias